8 и 6 наименьшее общее кратное: НОД и НОК для 6 и 8 (с решением)

2

Число 24

Свойства и характеристики одного числа
Все делители числа, сумма и произведение цифр, двоичный вид, разложение на простые множители…

Свойства пары чисел
Наименьшее общее кратное, наибольший общий делитель, сумма, разность и произведение чисел…

Сейчас изучают числа:

8 и 6 2011 2022 24742000 277353 31 8743 6929 5713 339 2247 110010100101 1043 1027 1019 1003 320 1543 36 и 45 444888 20449 594 и 60 500 391

Двадцать четыре

Описание числа 24

Неотрицательное рациональное двузначное число 24 является составным числом. Произведение цифр числа: 8. Число имеет следующие делители: 1, 2, 3, 4, 6, 8, 12, 24. Сумма делителей: 60. Обратным числом является 0.041666666666666664.
Данное число представляется произведением: 2 * 2 * 2 * 3.

Другие представления числа 24: двоичная система: 11000, троичная система: 220, восьмеричная система: 30, шестнадцатеричная система: 18. 24 байта представляет из себя число байт 24.

В виде кода азбуки Морзе: ..— ….-

Косинус 24: 0.4242, синус 24: -0.9056, тангенс 24: -2.1349. Натуральный логарифм числа равен 3.1781. У числа 24 есть десятичный логарифм: 1.3802. Квадратный корень числа: 4.8990, а кубический: 2.8845. Возведение в квадрат: 576.00.

24 в секундах это 24 секунды . В нумерологии число 24 означает цифру 6.

  • ← 23
  • 25 →

Вычисление наименьшего общего кратного

Введите цифры


  • Три автобуса
    Три автобуса общественного транспорта отправляются вместе с автовокзала утром. Первый автобус возвращается на станцию ​​через 18 минут, второй – через 12 минут, а третий – через 24 минуты. Как долго снова будем вместе на вокзале? Пожалуйста, экспресс
  • Портниха
    Портниха оставила кусок холста короче 5 метров. Она решает, сшить ли ей юбку или платье. Холста было ровно столько, сколько они израсходовали, разрезав юбку до 120 см, или 180 сантиметров. Какой кусок холста оставил ей?
  • LCM двух чисел
    Найдите наименьшее кратное 63 и 147
  • Различные 6975
    Три разных автобусных маршрута, 80, 81 и 82, отправляются с конечной станции в 5 ч 20 мин. Маршрут 80 отправляется каждые 30 минут, маршрут 81 — каждые 20 минут, а маршрут 82 — каждые 40 минут. Во сколько они снова уйдут?
  • Напоминание и частное
    Даны числа A = 135, B = 315. Найдите наименьшее натуральное число R, большее единицы, чтобы отношения R:A, R:B были с остатком 1.
  • Бакалейная лавка
    Сьюзен решила сделать продуктовые наборы для своего магазина. Оптовый торговец, у которого она покупает, продает сахар в упаковках по 20 штук в коробке, муку в упаковках по 12 штук в коробке и 15 мешков риса в коробке. Сколько штук каждого предмета она должна купить, чтобы их было одинаковое количество
  • Вокруг клумбы
    Вокруг прямоугольной клумбы размерами 5,25 м и 3,5 м нужно посадить розы через равные промежутки так, чтобы розы находились в каждом углу клумбы и потреблять как можно меньше. а) На каком расстоянии посажены розы? б) Сколько роз
  • Автобусы
    На остановке в 10 часов встретились автобусы №2 и №9. Автобус №2 ходит с интервалом 4 минуты, а автобус №9 с интервалом 9 минут. Сколько раз автобус встречается в 18:00 по местному времени?
  • Зубчатая передача
    Зубчатая передача состоит из двух колес. У одного 88, а у второго 56 зубов. Сколько раз поверните меньшее колесо, чтобы попасть в те же зубья, что и в начале? Сколько раз мы повернём самое большое колесо?
  • Автобусы 4
    Интервалы: 1-й автобус 40 мин. 2-й автобус 2 часа 3-й бутон 20 минут Через какое время они встретятся — как можно скорее?
  • Четыре класса
    Учащиеся всех 7, 8 и 9 классов одной школы могут занимать 4, 5, 6 и 7 ряд подряд, и никого не останется. Сколько в среднем учеников в одном классе, если в каждом классе всегда четыре класса?
  • Gcd и lcm
    Вычислить наибольший общий делитель и наименьшее общее кратное чисел. a) 16 и 18 b) 24 и 22 c) 45 и 60 d) 36 и 30
  • Вычислить 2976
    Вычислить наименьшее общее кратное чисел 120, 660 и 210.
  • Уточните: 4001
    Укажите: a = D (240,320) b = n (40,64)
  • Pardubická 4651
    Йирка решил разделить выигрыш от пари в Velká Pardubická между собой и тремя своими младшими братьями по возрасту в соотношении 2:3:5:7. Каждую сумму они платили целыми кронами. Одна из сумм составила 679 чешских крон. Насколько велик был выигрыш?
  • Веревка
    Пол может разрезать веревку на равные части, не оставив ни одной веревки. Длина может быть 15 см, 18 см или 25 см. Какова наименьшая возможная длина веревки?

другие математические задачи »

Что такое наибольший общий делитель и наименьшее общее кратное?

Привет! Добро пожаловать в это видео, посвященное наименьшему общему кратному и наибольшему общему делителю!

Как вы знаете, бывают случаи, когда нам приходится алгебраически «настраивать» то, как выглядит число или уравнение, чтобы продолжить нашу математическую работу. Для этого мы можем использовать наибольший общий делитель и наименьшее общее кратное. Наибольший общий делитель (GCF) — это наибольшее число, которое является множителем двух или более чисел, а наименьшее общее кратное (НОК) — это наименьшее число, кратное двум или более числам.

Чтобы понять, насколько полезны эти понятия, давайте рассмотрим сложение дробей. Прежде чем мы сможем складывать дроби, мы должны убедиться, что знаменатели совпадают, создав эквивалентную дробь:

\(\frac{2}{3}+\frac{1}{6} \rightarrow \frac{2 }{3} \times \frac{2}{2}\)\(+\frac{1}{6} \rightarrow \frac{4}{6} +\frac{1}{6}=\frac{ 5}{6}\)

 

В этом примере необходимо определить наименьшее общее кратное 3 и 6. Другими словами, «Каково наименьшее число, на которое можно разделить без остатка и 3, и 6?» Немного подумав, мы понимаем, что 6 является наименьшим общим кратным, потому что 6, деленное на 3, равно 2, а 6, деленное на 6, равно 1. Затем дробь \(\frac{2}{3}\) приводится к эквиваленту дробь \(\frac{4}{6}\) путем умножения числителя и знаменателя на 2. Теперь можно сложить две дроби с общими знаменателями, чтобы получить окончательное значение \(\frac{5}{6}\) .

Найдите наименьшее общее кратное

В контексте сложения или вычитания дробей наименьшее общее кратное называется наименьшим общим знаменателем .

Как правило, вам нужно определить число, большее или равное двум или более числам, чтобы найти их наименьшее общее кратное.

Важно отметить, что существует несколько способов определения наименьшего общего кратного. Один из способов — просто перечислить все кратные рассматриваемых значений и выбрать наименьшее общее значение, как показано здесь:

Наименьшее общее кратное 8, 4, 6

\(8\rightarrow 8,16,24,32,40,48\)
\(4\rightarrow 4,8,12,16,20,24, 28,32\)
\(6\стрелка вправо 6,12,18,24,30,36\)

 

Это показывает, что наименьшее общее кратное чисел 8, 4 и 6 равно 24, поскольку оно является наименьшим число, на которое можно поровну разделить 8, 4 и 6.

Другой распространенный метод включает первичную факторизацию каждого значения. Помните, что простое число делится только на 1 и само на себя.

После определения простых множителей перечислите общие множители один раз, а затем умножьте их на остальные оставшиеся простые множители. Результатом является наименьшее общее кратное:

\(30=2\умножить на 2\умножить на 3\умножить на 3\)
\(90=2\умножить на 3\умножить на 3\умножить на 5\)

\(\text {НОК}=2\умножить на 3\умножить на 3\умножить на 2\умножить на 5\)

 

Наименьшее общее кратное также можно найти путем общего (или повторного) деления. Этот метод иногда считают более быстрым и эффективным, чем листинг 9.0062 умножить на и найти простые множители. Вот пример нахождения наименьшего общего кратного чисел 3, 6 и 9 с помощью этого метода:

Разделите числа на множители любого из трех чисел. 6 имеет коэффициент 2, поэтому давайте использовать 2. Девять и 3 не могут делиться на 2, поэтому мы просто перепишем здесь 9 и 3. Повторяйте этот процесс, пока все числа не будут уменьшены до 1. Затем перемножьте все множители вместе, чтобы получить наименьшее общее кратное.

2 3 6 9
3 3 3 9
3 1 1 3
1 1 1

НОК \(=2\умножить на 3\умножить на 3=18\)

Теперь, когда были введены методы нахождения наименьших общих кратных, нам нужно изменить свое мышление, чтобы найти наибольший общий множитель двух или более числа. Мы будем определять значение, меньшее или равное рассматриваемым числам. Другими словами, спросите себя: «Какое наибольшее число делит оба этих числа?» Понимание этой концепции необходимо для деления и факторизации многочленов.

Какой наибольший общий делитель?

Разложение на простые множители также можно использовать для определения наибольшего общего делителя. Однако вместо того, чтобы умножать все простые множители, как мы это делали для наименьшего общего кратного, мы будем умножать только те простые множители, которые являются общими для чисел. Полученное произведение является наибольшим общим множителем.


Обзор

Давайте завершим парой правильных или неверных вопросов:

1. Наименьшее общее кратное 45 и 60 равно 15.

 

2. Наименьшим общим кратным является число, большее или равное рассматриваемым числам.

 

Спасибо за просмотр и приятного изучения!

Часто задаваемые вопросы

Q

Как найти LCM и GCF?

A

Существует множество методов определения LCM и GCF. Две наиболее распространенные стратегии включают составление списка или использование простой факторизации.

Например, НОК 5 и 6 можно найти, просто перечислив кратные \(5\) и \(6\), а затем указав наименьшее кратное, общее для обоих чисел.
\(5, 10, 15, 20, 25, \mathbf{30}, 35…\)
\(6, 12, 18, 24, \mathbf{30}, 36…\)
\(\mathbf{ 30}\) — LCM.

Аналогичным образом, GCF можно найти, перечислив факторы каждого числа, а затем указав наибольший общий фактор. Например, GCF для \(40\) и \(32\) можно найти, перечислив множители каждого числа.

\(40\): \(1, 2, 4, 5, \mathbf8, 10, 20, 40\)
\(32\): \(1, 2, 4, \mathbf8, 16, 32\ )
\(\mathbf8\) — это GCF.

Для больших чисел будет нереально составить список факторов или множителей для определения GCF или LCM. Для больших чисел наиболее эффективно использовать метод простой факторизации.

Например, при нахождении НОК начните с нахождения разложения каждого числа на простые множители (это можно сделать, создав дерево факторов). Простая факторизация \(20\) равна \(2\times2\times5\), а простая факторизация \(32\) — это \(2\times2\times2\times2\times2\). Обведите общие факторы и посчитайте эти только один раз .

Теперь умножьте все множители (помните, что не нужно дважды считать обведенные \(2\)). Это становится \(2\times2\times5\times2\times2\times2\), что равняется \(160\). LCM \(20\) и \(32\) равен \(160\).

Находя GCF, начните с перечисления простой факторизации каждого числа (это можно сделать, создав дерево факторов). Например, простая факторизация \(45\) равна \(5\times3\times3\), а простая факторизация \(120\) равна \(5\times3\times2\times2\times2\). Теперь просто умножьте все факторы, которые являются общими для обоих чисел. В этом случае мы умножим \(5\times3\), что равно \(15\). GCF \(45\) и \(120\) равен \(15\).

Подход с простой факторизацией может показаться довольно длительным процессом, но при работе с большими числами он гарантированно сэкономит время.

Q

Как найти GCF?

A

Существуют две основные стратегии нахождения GCF: перечисление факторов или использование простой факторизации.

Первая стратегия состоит в том, чтобы просто перечислить множители каждого числа, а затем найти наибольший множитель, общий для обоих чисел. Например, если мы ищем GCF для \(36\) и \(45\), мы можем перечислить множители обоих чисел и определить наибольшее общее число.
\(36\): \(1,2,3,4,6,\mathbf9,12,18,36\)
\(45\): \(1,3,5,\mathbf9,15,45 \)
GCF для \(36\) и \(45\) равен \(\mathbf9\).

Составление списка множителей каждого числа с последующим определением наибольшего общего множителя хорошо работает для небольших чисел. Однако при нахождении GCF очень больших чисел более эффективно использовать метод простой факторизации.

Например, при нахождении GCF чисел \(180\) и \(162\) мы начинаем с перечисления простой факторизации каждого числа (это можно сделать, создав дерево факторов). Простая факторизация \(180\) равна \(2\times2\times3\times3\times5\), а простая факторизация \(162\) — это \(2\times3\times3\times3\times3\). Теперь найдите факторы, которые являются общими для обоих чисел. В этом случае оба числа имеют одну общую \(2\) и две \(3\)s, или \(2\times3\times3\). Результатом \(2\times3\times3\) является \(18\), что является GCF! Эта стратегия часто более эффективна при нахождении GCF действительно больших чисел.

Q

Что означает GCF?

A

GCF означает «наибольший общий делитель». GCF определяется как наибольшее число, являющееся множителем двух или более чисел. Например, GCF для \(24\) и \(36\) равен \(12\), потому что наибольший делитель, общий для \(24\) и \(36\), равен \(12\). \(24\) и \(36\) имеют другие общие факторы, но \(12\) является самым большим.

Q

Как найти наименьшее общее кратное?

A

Существует множество методов нахождения наименьшего общего кратного. Двумя распространенными подходами являются перечисление кратных и использование простой факторизации. Список кратных так же, как это звучит, просто перечислите кратные каждого числа, а затем найдите наименьшее кратное, общее для обоих чисел. Например, при нахождении наименьшего общего кратного чисел \(3\) и \(4\) перечислите кратные:
\(3\): \(3,6,9,\mathbf{12},15,18 …\)
\(4\): \(4,8,\mathbf{12},16,20…\)
\(\mathbf{12}\) — это наименьшее кратное, общее для \(3\) и \(4\).

Список кратных — отличная стратегия, когда числа довольно малы. Когда числа большие, такие как \(38\) и \(42\), мы должны использовать метод простой факторизации. Начните с перечисления простой факторизации каждого числа (это можно сделать с помощью факторного дерева).
\(38\): \(2\times19\)
\(42\): \(2\times3\times7\)
Теперь обведите общие множители (считайте эти только один раз ).

Теперь умножьте все множители (не забудьте подсчитать \(2\) только один раз). Это становится \(2\times19\times3\times7\), что равняется \(798\). LCM \(38\) и \(42\) равен \(798\).

Q

Как вынуть LCM?

A

Извлечение НОК полезно при сложении или вычитании дробей. При определении наименьшего общего кратного получается знаменатель, одинаковый для обеих дробей. Например, общий знаменатель для \(\frac{2}{7}+\frac{3}{5}\) будет равен \(35\), потому что \(35\) является НОК для \(7\ ) и \(5\). Новые дроби становятся \(\frac{10}{35}+\frac{21}{35}\), что равняется \(\frac{31}{35}\).

Практические вопросы

Вопрос №1:

 
Каков наибольший общий делитель чисел 16 и 42? Используйте его, чтобы уменьшить дробь \(\frac{16}{42}\).

GCF равно 8, и мы уменьшаем его до \(\frac{2}{5}\).

GCF равен 1, и мы не можем больше уменьшать.

GCF равно 4, и мы уменьшаем его до \(\frac{4}{11}\).

GCF равен 2, и мы уменьшаем его до \(\frac{8}{21}\).

Показать ответ

Ответ:

Правильный ответ D: GCF равен 2, и мы уменьшаем до \(\frac{8}{21}\).

Давайте подойдем к этой проблеме, перечислив простые множители как числителя, так и знаменателя.
\(16=2×2×2×2\)
\(42=2×3×7\)

Здесь мы видим, что 2 — единственный общий делитель чисел 16 и 42 и, следовательно, их наибольший общий делитель. Затем мы можем разделить оба числа на 2, чтобы уменьшить дробь:
\(\frac{16\div2}{42\div2}=\frac{8}{21}\)

Скрыть ответ

Вопрос № 2:

 
Найдите наименьшее общее кратное 2, 6 и 8.

16

18

24

48

Показать ответ

Ответ:

Правильный ответ C: 24,

2 простых чисел.


\(2=2\) (обратите внимание, что мы могли написать \(2\times1\), но 1 понимается или подразумевается, и обычно писать не нужно)
\(6=2\times3\)
\( 8=2\times2\times2\)

Помните, что при вычислении НОК двух или более чисел мы перечисляем каждый простой множитель один раз, который является общим для всех чисел. Поскольку каждое из наших чисел имеет 2 в качестве простого множителя, наш LCM также будет иметь 2 в качестве одного из своих простых множителей.
LCM \(=2\times\) _______

Теперь из 6 у нас осталась 3, а из 8 осталось две двойки. Мы умножаем их, чтобы получить
LCM \(=2\times3\times2\times2=24\)

Обратите внимание, что хотя 2, 6 и 8 являются делителями 48, решение не D, потому что 48 не является наименьшее общее кратное.

Скрыть ответ

Вопрос № 3:

 
Перечислите первые несколько чисел, кратных 3, 5 и 6, чтобы найти наименьшее общее кратное.

LCM IS 15

LCM — 30

LCM — 18

LCM — 75

Показать ответ

Ответ:

Правильный ответ B: LCM — 30.
. : 3, 6, 9, 12, 15, 18, 21, 24, 27, 30, 33, …
Первые несколько кратных 5: 5, 10, 15, 20, 25, 30, 35, 40 , 45, 50, …
Первые несколько кратных 6: 6, 12, 18, 24, 30, 36, 42, 48, 54, 60, …

Как мы видели выше, 30 — это первое (наименьшее) число, которое является общим среди кратных чисел 3, 5 и 6, поэтому наименьшее общее кратное равно 30.

Скрыть ответ


У Кортни 54 конфеты, а у Триш 36. Они хотят приготовить мешочки с конфетами на день рождения своей подруги Ким, но в каждом мешочке должно быть одинаковое количество конфет. Чтобы в каждом пакете было как можно больше конфет, когда Кортни и Триш работают отдельно, сколько пакетов они могут сделать и сколько конфет будет в каждом пакете?

10 пакетов по 9 конфет

9 пакетов по 10 конфет

15 пакетов по 6 конфет

5 пакетов по 18 конфет

Показать Ответ

Ответ:

Правильный ответ D: 5 пакетов по 18 конфет в каждом.

Для начала перечислите простые множители чисел 54 и 36:
\(54=2\times3\times3\times3\)
\(36=2\times2\times3\times3\)

Обратите внимание, что они оба разделяют двойка и две тройки. Произведение этих общих простых множителей равно \(2\times3\times3=18\).

Перевод джипег в пдф онлайн в один файл: Конвертировать JPG в PDF — быстрый, онлайн, бесплатный

конвертер jpg в pdf — конвертер пдф из джипег в пдф

Зачем конвертер пдф из джипег в пдф?

Предположим, вам нужно отправить группу изображений на утверждение, и вы предпочитаете отправлять их в виде одного файла PDF в «формате переносимого документа». Возможно, при преобразовании изображения в PDF необходимо учитывать общий размер файла. Что ж, какой бы ни была причина, наш бесплатный онлайн- фотографий конвертер пдф- лучший и быстрый способ конвертировать JPG в PDF.


конвертер пдф из джипег в пдф (с сохранением исходного качества):

Наш конвертер изображений в PDF имеет удобный интерфейс, который лучше всего подходит для преобразования файлов изображений JPG в формат PDF. Он точно сохраняет макет исходных изображений. Этот онлайн конвертировать джипег в пдф то же качество изображения после процесса преобразования без потери качества. Благодаря этим алгоритмам прецизионного онлайн-преобразования, которые обеспечивают высокое качество изображения!

Как конвертировать JPG в PDF онлайн (шаг за шагом)?

Процесс преобразования изображения в PDF становится удобным с помощью этого бесплатного конвертер JPG в PDF. Вот шаги, которые помогут вам безопасно сохранить JPG как PDF:

Шаг 1:

  • Перетащите или загрузите файл .JPG в этот онлайн-конвертер конвертер пдф. Вы также можете добавить несколько файлов JPG, чтобы преобразовать их в файлы PDF.

Шаг 2:

  • Теперь нажмите кнопку «Конвертировать» и подождите секунду, конвертер(конвертер пдф) превратит из джипег в пдф.

Шаг 3:

  • Просто нажмите кнопку «Загрузить».

Часто задаваемые вопросы:

Как сохранить картинку в формате PDF на вашем компьютере?
  • Сначала откройте фото на компьютере
  • Затем перейдите в меню «Файл»> «Печать» или воспользуйтесь сочетанием клавиш Command + P.
  • Теперь появится диалоговое окно «Печать», просто выберите раскрывающееся меню PDF и выберите «Сохранить как PDF».
  • Наконец, выберите имя нового PDF-файла и нажмите Сохранить.

Как конвертировать ​JPG в PDF на macOS?

Чтобы начать преобразование, откройте предварительный просмотр

  • Зайдите в меню File, там вы увидите опцию Open, выберите ее.
  • Пришло время выбрать изображения JPG, которые нужно преобразовать в PDF.
  • Вы можете увидеть кнопку Открыть внизу окна, нажмите на нее
  • Еще раз нажмите на меню «Файл» и прямо сейчас выберите «Экспорт в PDF».
  • Теперь добавьте желаемое имя для вашего PDF-файла.
  • После этого выберите место для сохранения в раскрывающемся списке «Где».
  • Наконец, нажмите кнопку «Сохранить».

Воспользуйтесь бесплатным онлайн-конвертером JPEG в PDF из нашего лучшего источника, который поможет вам сохранить формат JPG / JPEG (изображение) в PDF на macOS без искажения макета.

Как преобразовать несколько файлов JPG в несколько файлов PDF?

Чтобы преобразовать несколько файлов JPG в файлы PDF:

  • Вам просто нужно вставить изображения JPG в документ текстового процессора, а затем отправить документ в формат PDF в меню печати.

Теперь стало удобно сохранять несколько изображений JPG в документы PDF с помощью конвертер jpg в pdf.

Как преобразовать JPEG в PDF на моем телефоне?

Используйте наш JPEG конвертер пдф, который является наиболее удобным для мобильных устройств, с помощью которого вы можете легко преобразовать неограниченное количество файлов Jконвертировать из джипег в пдф.


Как конвертировать ​JPG в PDF на Android?

Выполните следующие действия, чтобы выполнить такие преобразования на Android:

  • Сначала откройте Google Фото.
  • Нажмите на изображение JPG, которое вы хотите преобразовать.
  • Теперь нажмите на трехточечное меню «⋮»
  • Затем нажмите кнопку «Печать» в меню.
  • Затем выберите «Сохранить как PDF» в меню «Выбрать принтер».
  • Теперь нажмите на значок PDF(конвертер пдф)
  • Назовите преобразованный файл и нажмите «Сохранить».

Совместное использование файла PDF, а не JPG, является идеальным, поскольку получатель с большей вероятностью сможет его открыть. Просто воспользуйтесь этим бесплатным онлайн конвертер JPG в PDF, который обеспечивает точное качество преобразования, он эффективно преобразует один или несколько файлов конвертировать джипег в пдф.

Other Languages: JPG To PDF Converter, Ubah JPG ke PDF, JPG PDF Çevirme, Konwerter JPG na PDF, Převod JPG do PDF, JPG in PDF Umwandeln, Converter JPG em PDF, Convertir JPG en PDF, Convertir JPG a PDF, JPG PDF Konvertáló, Converti da JPG a PDF, JPG PDF 변환, JPG PDF 変換, Μετατροπη JPG σε PDF, แปลงไฟล์ JPG เป็น PDF, Chuyển JPG sang PDF

Комбинатор PDF – Объединение PDF-файлов

ЗАГРУЗИТЬ

Перетащите файлы сюда.

Файл PDF — это тип цифрового документа. В большинстве случаев PDF содержит текст, но он также может содержать активные ссылки и даже изображения. Иногда человек создает PDF на компьютере, но PDF также может быть сканом физических предметов, таких как бумажные документы, удостоверения личности и многое другое.

Вы можете подумать, что PDF — это то же самое, что и DOC. Однако они очень разные. Файлы DOC обычно создаются в текстовом процессоре либо в Интернете, например, в Google Docs, либо в отдельной программе, такой как Microsoft Word. Если вы сохраните этот файл в формате DOC и передадите его кому-то другому, DOC, скорее всего, будет выглядеть совсем по-другому для второго человека. Это потому, что DOC связаны с установленными шрифтами и другими специфическими атрибутами операционной системы, поэтому невозможно сказать, как они будут выглядеть на других системах. У PDF нет такой проблемы: они выглядят одинаково на всех системах!

Можно ли объединять PDF-файлы?

Объединение PDF-файлов вполне возможно. Однако для этого обычно требуется программное обеспечение для редактирования PDF, например, Adobe Acrobat. Эти программы могут быть довольно дорогими.

К счастью, вам не нужно покупать модное программное обеспечение для объединения PDF-файлов. Наш инструмент сделает все это за вас бесплатно, и вам даже не нужно ничего устанавливать.

Наш бесплатный PDF-объединитель: как им пользоваться

Наш бесплатный инструмент может объединить до 20 PDF-файлов за один раз. Он очень прост в использовании.

Сначала перетащите файлы PDF в область с надписью “Перетащите файлы сюда”. Если вы используете мобильное устройство, вы можете просто нажать кнопку “ЗАГРУЗИТЬ”, а затем перейти к своим PDF-файлам.

Теперь, когда файлы загружены, вам нужно убедиться, что они расположены в правильном порядке. В очереди вы увидите все ваши файлы, расположенные в виде сетки. Перемещайте файлы, пока они не будут расположены правильно. Первый PDF в списке будет первым PDF в объединенном файле. Второй PDF будет расположен под первым, и так далее.

Если все выглядит нормально, нажмите кнопку “ОБЪЕДИНИТЬ ФАЙЛЫ”. Наша система объединит все PDF в один новый PDF. Загрузка нового PDF начнется автоматически. Если у вас более 20 PDF, не волнуйтесь. Вы можете объединить их все с помощью нескольких дополнительных шагов. После загрузки объединенного PDF вы можете загрузить его в очередь. Затем добавьте дополнительные PDF и снова объедините их. Вы можете делать это столько раз, сколько захотите.

Обратите внимание, что если вы загрузите несколько файлов, вам нужно будет объединить их вместе и загрузить новый файл в течение часа. Если вы этого не сделаете, наша система отбросит загруженные файлы, и вам придется начинать все сначала.

Безопасен ли PDFJoiner.com?

Наш инструмент абсолютно безопасен в использовании. Вам не нужно беспокоиться о потере оригинальных PDF-файлов при их объединении. Наш инструмент копирует файлы из вашей системы, а затем объединяет эти копии. Ваши оригиналы в безопасности на вашем компьютере.

Кроме того, наша система безопасна. Через 60 минут наш инструмент очищает все конвертации и загрузки. Это гарантирует, что ваша конфиденциальная информация не будет скомпрометирована.

Преобразование JPEG в PDF онлайн

  • Дом
  • org/ListItem»> Индекс функциональности
  • Конвертер PDF: конвертируйте PDF в редактируемый
  • Конвертируйте любой формат в редактируемый PDF онлайн
  • Конвертировать Jpeg в PDF

Формы заполнены

Формы подписаны

Формы отправлены

Начать бесплатно

Загрузите свой документ в редактор PDF

Введите в любом месте или подпишите форму

Печать, электронная почта, факс, или экспорт

900 14 Попробуйте прямо сейчас! Редактировать pdf

Пользователи доверяют управлению документами на платформе pdfFiller

65,5 тыс. +

документов, добавляемых ежедневно

53%

документов, созданных

из шаблонов

4M

90 014 PDF-файлов, отредактированных в месяц

238K

общих документов

в месяц месяц

Универсальное программное обеспечение для работы с PDF

Единая таблетка от всех головных болей, связанных с PDF. Редактируйте, заполняйте, подписывайте и делитесь — на любом устройстве.

Начать бесплатную пробную версию

Инструкции и помощь по преобразованию Jpeg в PDF для

Преобразование JPEG в PDF: редактируйте PDF-файлы из любого места

Формат PDF является одним из наиболее широко используемых форматов документов по разным причинам. С любого устройства можно обмениваться ими между настольными компьютерами и телефонами с разными экранами и настройками. PDF-файлы будут выглядеть одинаково, независимо от того, открываете ли вы их на Mac, Microsoft или на смартфонах.

Безопасность данных — еще одна причина, по которой мы предпочитаем использовать PDF-файлы для хранения и обмена личной информацией и документами. Вот почему важно выбрать безопасный редактор, особенно при работе в Интернете. Используя онлайн-решение, можно отслеживать историю просмотров, чтобы узнать, кто имел к ней доступ раньше.

pdfFiller — это онлайн-инструмент для создания и редактирования документов, который позволяет создавать, изменять, подписывать и отправлять PDF-файлы в одном окне браузера. Преобразуйте файл MS Word или электронную таблицу Google и начните редактировать их внешний вид и добавьте несколько заполняемых полей, чтобы сделать документ доступным для пения. Используйте готовый документ для личных нужд или поделитесь им с другими любым удобным способом — вы получите уведомление, когда человек откроет и заполнит форму.

Используйте мощные функции редактирования для ввода текста, комментирования и выделения. Добавляйте и редактируйте визуальный контент. Изменить порядок страниц формы. Добавьте заполняемые поля и отправьте на подпись. Сотрудничайте с другими, чтобы заполнить поля. Когда документ будет готов, загрузите его на свое устройство или сохраните в облаке.

Чтобы отредактировать документ, выполните следующие действия:

01

Начните с загрузки документа.

02

Чтобы изменить содержимое документа, щелкните вкладку «Инструменты» и следуйте инструкциям.

03

Чтобы вставить заполняемые поля, щелкните вкладку «Добавить заполняемые поля» справа и добавьте в них текст, подписи, изображения и многое другое.

04

По завершении нажмите Готово и перейдите к загрузке, отправке или печати документа.

Как конвертировать JPEG в PDF

Связанные функции

В нынешнем виде на каждый вопрос типа «Нужен ли вам Adobe® Flash® для преобразования PDF?» Каждый веб-сайт должен задать этот вопрос и сделать его доступным для своих пользователей. Это все равно, что сказать, что если вы используете инструмент CDR только для преобразования одного видеоизображения в одно видео MP4, это все равно отличный инструмент для преобразования видео MP4 в видео H.264. В ближайшем будущем у нас есть действительно замечательные вещи. Получите доступ к огромному архиву изображений в формате JPEG. Скачивайте файлы JPEG быстро и легко Экономьте время с удобным интерфейсом приложения Скачивайте изображения в формате JPEG на рабочий стол, компьютер или куда угодно Переходите непосредственно к папкам с изображениями или копируйте папки с изображениями в другие папки. Скачивайте файлы JPEG онлайн, включая фото и видео Копируйте изображения на другие компьютеры или мобильные устройства Экономьте время с удобным интерфейсом приложения Форматы файлов PDF могут быть сложными в управлении по разным причинам, таким как нехватка места для хранения или трудности с поддержанием правильного межстрочного интервала между документами (как отображается на вашем мониторе). JPEG разработан, чтобы помочь вам выбрать и извлечь файлы изображений JPEG, а затем быстро преобразовать их в формат PDF. Кроме того, вы можете управлять и копировать изображения JPEG с любого компьютера, а также на мобильные устройства, не выходя из этого приложения. Получите доступ к обширному архиву изображений JPEG Самая большая коллекция файлов изображений JPEG доступна в формате JPEG бесплатно. Это идеальное хранилище для всех ваших потребностей в изображениях JPEG, включая фотографии, слайд-шоу, логотипы, изображения, видео и изображения, используемые в качестве фона в журналах. Скачивайте изображения в формате JPEG на рабочий стол, компьютер или куда угодно JPEG — интуитивно понятное приложение с удобным интерфейсом. Используйте его для удаления пустых страниц, объединения нескольких PDF-файлов в один или удаления всего документа. Обратите внимание, что на Mac формы PDF создаются и сохраняются в файлы PDF с помощью встроенного приложения для чтения PDF. Для наших последних примеров я объединил примеры 2 и 3 отдельной бумажной тетради — той же тетради, которую мы использовали раньше, только с большим количеством заметок и пустых страниц. Рисунок 1-4a: Использование сенсорной панели для удаления или объединения страниц Страницы этой записной книжки пусты, но с каждой стороны есть по одной странице. Страницы сгруппированы по годам, и мы видим, что они были подписаны «Джоном Доу». Страницы с номерами от 1 до 4 находятся на разных сторонах дневника, поэтому, когда мы объединяем страницы 1 и 2 для создания новой страницы, мы делаем ее первой страницей, подписанной Джоном Доу..

Что говорят наши клиенты о pdfFiller

Убедитесь сами, прочитав отзывы на самых популярных ресурсах:

Елена С

05.01.2017

Администратор юридической практики

28.01.2019 9001 5

Получите мощный редактор PDF для своего Mac или ПК с Windows

Установите настольное приложение, чтобы быстро редактировать PDF-файлы, создавать заполняемые формы и безопасно хранить документы в облаке.

Редактируйте PDF-файлы и управляйте ими из любого места с помощью устройства iOS или Android

Установите наше мобильное приложение и редактируйте PDF-файлы с помощью удостоенного наград набора инструментов, где бы вы ни находились.

Получите редактор PDF в браузере Google Chrome

Установите расширение pdfFiller для Google Chrome, чтобы заполнять и редактировать PDF-файлы прямо из результатов поиска.

Загрузка из Интернет-магазина Chrome

pdfFiller получает высшие оценки в нескольких категориях на G2

Список дополнительных функций

Часто задаваемые вопросы о pdfFiller

Ниже приведен список наиболее частых вопросов клиентов.

Если вы не можете найти ответ на свой вопрос, не стесняйтесь обращаться к нам.

Как преобразовать файл из JPG в PDF?

Перетащите изображения JPG, которые вы хотите объединить в один PDF-файл (или нажмите кнопку «Добавить файл»). При необходимости измените порядок файла. Нажмите кнопку «Преобразовать файл(ы)», чтобы преобразовать изображения JPG в PDF. Сохраните преобразованный файл, нажав кнопку «Загрузить PDF-файл».

Как преобразовать JPG в PDF?

Перетащите изображения JPG, которые вы хотите объединить в один PDF-файл (или нажмите кнопку «Добавить файл»). При необходимости измените порядок файла. Нажмите кнопку «Преобразовать файл(ы)», чтобы преобразовать изображения JPG в PDF. Сохраните преобразованный файл, нажав кнопку «Загрузить PDF-файл».

Как добавить JPEG в PDF?

Нажмите «Редактировать PDF» > «Добавить изображение», а затем выберите изображение JPG, которое вы хотите вставить в документ. Поместите изображение в файл PDF, а затем выберите параметр сохранения в меню «Файл»> «Сохранить», чтобы получить новый файл PDF на свой компьютер.

Как объединить несколько файлов JPEG в один PDF?

Выберите все изображения, которые вы хотите в своем PDF-файле, щелкните правой кнопкой мыши и выберите «Открыть с предварительным просмотром». На боковой панели предварительного просмотра перетащите изображения в том порядке, в котором они должны отображаться в вашем PDF-файле. Выберите/выделите все изображения, которые необходимо включить в документ PDF. В противном случае только одно изображение может оказаться в документе PDF.

Как объединить файлы JPEG в один?

Шаг 1: Запустите программное обеспечение и выберите режим слияния. Шаг 2: Выберите количество изображений и другие настройки. Шаг 3: Добавьте файлы JPG и перетащите их в интерфейс. Шаг 4: Сохраните режим и объедините JPEG. Шаг 5. Добавьте текст в объединенный файл (необязательно)

Как поместить несколько изображений в PDF?

Чтобы объединить изображения в файл PDF, сначала выберите все изображения в проводнике или на рабочем столе. Затем щелкните правой кнопкой мыши одно из выбранных изображений и нажмите «Печать». Появится окно Печать изображений. В раскрывающемся меню «Принтер» в левом верхнем углу выберите Microsoft Print to PDF.

Рабочие процессы электронной подписи стали проще

Подписывайте, отправляйте на подпись и отслеживайте документы в режиме реального времени с помощью signNow.

Начать бесплатную пробную версию

Бесплатный конвертер JPG в PDF онлайн

Главная

>

JPG в PDF

Преобразование изображений в файлы PDF за считанные секунды. Поддерживаются форматы JPG, JPEG, PNG, BMP и другие.

Загрузить для просмотра всех функций

Перетащите сюда изображение

Выбрать файл изображенийВыбрать изображение

Оценить JPG в PDF

4.4 /5

0 голосов

Почему стоит выбрать WPS Office для преобразования изображений в PDF?

  • Бесплатное и высококачественное преобразование

    WPS PDF Converter обеспечивает преобразование высококачественных изображений JPG в PDF и несколько настроек вывода, помогающих преобразовывать изображения в высококачественные файлы PDF.

  • Совместимость с несколькими форматами изображений

    Онлайн-конвертер JPG в PDF — это универсальный инструмент, позволяющий добавлять изображения JPG, GIF, BMP и PNG и преобразовывать их в файлы PDF.

  • Высокая гибкость

    WPS PDF Converter позволяет выводить изображения по одному или в объединенном виде. Вы можете настроить размер страницы, ориентацию страницы и поля по своему усмотрению.

Часто задаваемые вопросы о преобразовании изображений в PDF

Как преобразовать изображения в файлы PDF онлайн?

Как сохранить изображение JPEG в формате PDF без потери качества?

Какой самый быстрый способ конвертировать изображения в файлы PDF?

Узнайте больше о JPG в PDF

Другие статьи >

  • Учебные пособия

    Как конвертировать PDF в JPG на Mac: 3 простых руководства

    PDF — это универсальный стандарт для отправки файлов, и это нередактируемые документы. Мы часто используем PDF для загрузки документа, но иногда веб-сайт или программное обеспечение не имеют формата файла для поддержки формата PDF. Вы можете преобразовать PDF в JPG, если ваш файл PDF не поддерживает его. Mac имеет несколько различных функций для преобразования pdf в jpg. Пользователи внимательно читают статью, потому что я показываю, как мы конвертируем PDF-файл в JPG на Mac. Используйте WPS Office для преобразования PDF в JPG WPS Office широко распространен…

  • Учебные пособия

    Как объединить JPG в PDF

    Иногда, когда у нас есть набор изображений JPG или JPEG и мы можем поделиться с другими, мы выбираем сжатие изображений в файл. Но что, если мы не хотим, чтобы эти фотографии были скопированы или даже украдены для коммерческого использования? Можем ли мы объединить изображения JPG в PDF для кросс-платформенного обмена и представления с защитой интеллектуальной собственности? Здесь мы представляем вам решение для слияния и объединения изображений JPG в файл PDF.

Разложение тангенса в ряд тейлора: Parabola: Разложение тангенса.

Котангенс в нуле в ряд тейлора : Анализ-I

Правила форума

В этом разделе нельзя создавать новые темы.

Если Вы хотите задать новый вопрос, то не дописывайте его в существующую тему, а создайте новую в корневом разделе «Помогите решить/разобраться (М)».

Если Вы зададите новый вопрос в существующей теме, то в случае нарушения оформления или других правил форума Ваше сообщение и все ответы на него могут быть удалены без предупреждения.

Не ищите на этом форуме халяву, правила запрещают участникам публиковать готовые решения стандартных учебных задач. Автор вопроса обязан привести свои попытки решения и указать конкретные затруднения.

Обязательно просмотрите тему Правила данного раздела, иначе Ваша тема может быть удалена или перемещена в Карантин, а Вы так и не узнаете, почему.

 
never-sleep 

 Котангенс в нуле в ряд тейлора

10. 01.2012, 14:52 

27/11/11
153

Нужно разложить в ряд Тейлора в окрестности с точностью до

В лоб по формуле Тейлора — не получается, ибо не определено

Тут дело в том, что приравнивая коэффициенты при одинаковых степенях, мы получим при
равенство

Мне кажется, что это все связано с неопред. котангенса в нуле…

Появилась идея разложить тангенс в нуле, а потом воспользоваться тем, что
и раскладывать в Тейлора так, но мне не нравится эта затея…

Может подскажете — как быть?)


   

                  

Nemiroff 

 Re: Котангенс в нуле в ряд тейлора

10. 01.2012, 14:58 

Заслуженный участник

20/07/09
4026
МФТИ ФУПМ

Ну вообще, котангенс в нуле издалека напоминает бесконечность, так что странно ждать нормального значения у него при нулевой степени.
Добавьте, что ли, еще минус первую степень икса в неопределенное разложение котангенса.


   

                  

never-sleep 

 Re: Котангенс в нуле в ряд тейлора

10. 01.2012, 15:00 

27/11/11
153

Nemiroff в сообщении #525264 писал(а):

Ну вообще, котангенс в нуле издалека напоминает бесконечность, так что странно ждать нормального значения у него при нулевой степени.
Добавьте, что ли, еще минус первую степень икса в неопределенное разложение котангенса.

Ок, спасибо! А как вы до этого догадались и как это можно обосновать, что мы должны добавлять отрицательные степени? А почему только минус первую? А может еще нужно минус вторую?
Как в аналогичных примерах действовать)

Есть ли еще подобные функции, где с такими подвохами нужно бороться?!


   

                  

PAV 

 Re: Котангенс в нуле в ряд тейлора

10. 01.2012, 15:02 

Супермодератор

29/07/05
8248
Москва

Рассмотрите произведение — особенность в нуле исчезнет.


   

                  

Nemiroff 

 Re: Котангенс в нуле в ряд тейлора

10. 01.2012, 15:08 

Заслуженный участник

20/07/09
4026
МФТИ ФУПМ

Разделил косинус на синус, вынес из синус одну степень икса и разложил знаменатель.


   

                  

svv 

 Re: Котангенс в нуле в ряд тейлора

10. 01.2012, 15:16 

Заслуженный участник

23/07/08
9987
Crna Gora

never-sleep писал(а):

Есть ли еще подобные функции, где с такими подвохами нужно бороться?!

Нужно всё-таки иметь в виду, что исходное задание — разложить котангенс в ряд Тейлора в нуле — невыполнимо. Ни сама функция, ни её производные в этой точке не определены. Если Вам дано такое задание, Вы вправе просто объяснить его некорректность и больше ничего.

Вообще, полезно помнить, что не любую функцию и не в любой точке можно разложить в ряд Тейлора.

Все дальнейшие советы проистекают из стремления представить функцию в нуле хоть как-то, раз нельзя с помощью ряда Тейлора. Или — Тейлором, но несколько другую функцию (). Но это уже другие задачи.


   

                  

never-sleep 

 Re: Котангенс в нуле в ряд тейлора

10.01.2012, 18:21 

27/11/11
153

PAV в сообщении #525269 писал(а):

Рассмотрите произведение — особенность в нуле исчезнет.

Ок, спасибо, понятно)

— 10.01.2012, 18:25 —

Nemiroff в сообщении #525273 писал(а):

Разделил косинус на синус, вынес из синус одну степень икса и разложил знаменатель.

Спасибо) Я понял так)

Правильно?

— 10.01.2012, 18:30 —

svv в сообщении #525275 писал(а):

Нужно всё-таки иметь в виду, что исходное задание — разложить котангенс в ряд Тейлора в нуле — невыполнимо. Ни сама функция, ни её производные в этой точке не определены. Если Вам дано такое задание, Вы вправе просто объяснить его некорректность и больше ничего.

Вообще, полезно помнить, что не любую функцию и не в любой точке можно разложить в ряд Тейлора.

Все дальнейшие советы проистекают из стремления представить функцию в нуле хоть как-то, раз нельзя с помощью ряда Тейлора. Или — Тейлором, но несколько другую функцию (). Но это уже другие задачи.

Ок, спасибо, теперь понятнее. В ряде тейлора вроде как нет отрицательных степеней…видимо поэтому это разложение и нельзя назвать рядом тейлора.


   

                  

SpBTimes 

 Re: Котангенс в нуле в ряд тейлора

10.01.2012, 19:10 

Заслуженный участник

18/12/10
1600
spb

Лоран, если хотите


   

                  

Показать сообщения за: Все сообщения1 день7 дней2 недели1 месяц3 месяца6 месяцев1 год Поле сортировки АвторВремя размещенияЗаголовокпо возрастаниюпо убыванию 
  Страница 1 из 1
 [ Сообщений: 8 ] 

Модераторы: Модераторы Математики, Супермодераторы


Помогите решить / разобраться (М)

 
denmanorwat 

 Разложить по Тейлору 1/cos(x)

01. 10.2021, 17:56 

31/07/20
16

Здравствуйте. Разбираюсь в разложении функции в ряд Тейлора в окрестности нуля. Автор предлагает следующий трюк: По Тейлору раскладываем косинус: . Представляем кусок разложения заменой переменной: . Далее, раскладываем по Тейлору не , а уже функцию с заменённой переменной: , после чего подставляем . Возникает вопрос: а почему мы можем пользоваться трюком с заменой переменной чтобы добиться разложения тангенса по Тейлору?


   

                  

nnosipov 

 Re: Разложить по Тейлору 1/cos(x)

01. 10.2021, 18:56 

Заслуженный участник

20/12/10
8862

denmanorwat в сообщении #1533521 писал(а):

а почему мы можем пользоваться трюком с заменой переменной

А почему нет? Что нам мешает заменить сложное выражение от одной буквой ? Мы же не собираемся (надеюсь) игнорировать о-малые. Да и при , так что все оценки валидны.

— Пт окт 01, 2021 22:56:45 —

denmanorwat в сообщении #1533521 писал(а):

разложения тангенса

Наверное, секанса.


   

                  

novichok2018 

 Re: Разложить по Тейлору 1/cos(x)

01. 10.2021, 21:48 

Заблокирован

16/04/18

1129

Это же какие то Бернулли?


   

                  

denmanorwat 

 Re: Разложить по Тейлору 1/cos(x)

01.10.2021, 22:45 

31/07/20
16

nnosipov в сообщении #1533535 писал(а):

denmanorwat в сообщении #1533521 писал(а):

а почему мы можем пользоваться трюком с заменой переменной

А почему нет? Что нам мешает заменить сложное выражение от одной буквой ? Мы же не собираемся (надеюсь) игнорировать о-малые. Да и при , так что все оценки валидны.

— Пт окт 01, 2021 22:56:45 —

denmanorwat в сообщении #1533521 писал(а):

разложения тангенса

Наверное, секанса.

Спасибо. Прочитал ваше сообщение, подумал, и да, действительно, ничего не мешает. И поэтому о-малые игнорировать таки не стоит. Спасибо за ответ.
P.S. Да, секанса, опечатка.


   

                  

Nemiroff 

 Re: Разложить по Тейлору 1/cos(x)

01.10.2021, 22:52 

Заслуженный участник

20/07/09
4026
МФТИ ФУПМ

denmanorwat в сообщении #1533604 писал(а):

И поэтому о-малые игнорировать таки не стоит.


.
Заменим всюду сперва на , затем на .
Получим и
Первое ещё куда ни шло: при подстановке получаем .
Второе не столь позитивно — .


   

                  

nnosipov 

 Re: Разложить по Тейлору 1/cos(x)

02.10.2021, 04:24 

Заслуженный участник

20/12/10
8862

novichok2018 в сообщении #1533585 писал(а):

Это же какие то Бернулли?

Числа Эйлера, см.

https://en.wikipedia.org/wiki/Trigonome … _expansion


   

                  

thething 

 Re: Разложить по Тейлору 1/cos(x)

03.10.2021, 05:16 

Заслуженный участник

27/12/17
1381
Антарктика

denmanorwat в сообщении #1533521 писал(а):

Возникает вопрос: а почему мы можем пользоваться трюком с заменой переменной чтобы добиться разложения тангенса по Тейлору?

У нас есть разложение для . Далее, замечаем, что выражение в какой-то окрестности нуля меньше единицы (ибо вообще стремится к нулю), так что подстановка оправдана. Ну а возможность перегруппировки слагаемых вытекает из абсолютной сходимости и теорем о повторных рядах. Не понятно только, причём тут о-малые, когда речь о рядах

Тейлора? Строгое доказательство можете посмотреть во втором томе Фихтенгольца. Пункт 446 так и называется: подстановка ряда в ряд.


   

                  

Показать сообщения за: Все сообщения1 день7 дней2 недели1 месяц3 месяца6 месяцев1 год Поле сортировки АвторВремя размещенияЗаголовокпо возрастаниюпо убыванию 
  Страница 1 из 1
 [ Сообщений: 7 ] 

Модераторы: Модераторы Математики, Супермодераторы


{n-1} a_k\ a_{n-1-k}&n>0\\ \end{случаи} $$ то есть последовательность $$(a_n)_{n\in\mathbb{N}}=\left(\frac 11,\frac 13, \frac 2{15}, \frac {17}{315}, \frac { 62}{2835}, \frac{1382}{155925},\cdots\right)$$

$\endgroup$

1

$\begingroup$

Этот ответ основан на ответе coffeemath. 7) \end{выравнивание} Отсюда можно вывести линейную систему \начать{выравнивать} \begin{pматрица} 1 и 0 и 0 и 0 и 0 и 0&0\\ 0 и 1 и 0 и 0 и 0 и 0 &0\\ -1/2 &0 &1 & 0& 0& 0&0\\ 0 и -1/2 и 0 и 1 и 0 и 0&0 \\ 1/24 и 0 и -1/2 и 0 и 1 и 0&0\\ 0 и 1/24 и 0 и -1/2 и 0 и 1 и 0\\ 1/720 и 0 и 1/24 и 0 и -1/2 и 0 и 1 \end{pматрица} \begin{pматрица} а\\б\\в\\г\\е\\е\\г \end{pматрица} «=» \begin{pmatrix} 0\1\0\-1/6\0\1/120\0 \end{pматрица} \end{выравнивание} Что имеет решение \начать{выравнивать} \begin{pматрица} а\\б\\в\\г\\е\\е\\г \end{pматрица} = \begin{pmatrix} 0\1\0\1/3\0\2/15 \end{pматрица} \end{выравнивание} [Обратите внимание, что вы можете извлечь две линейные системы из приведенной выше. Затем вам нужно найти только нечетные степени $x$, поскольку $\tan(x)$ нечетно.] 97) \end{выравнивание} Что (на удивление) правильно, поскольку мы можем сравнить наше решение с WolframAlpha.

$\endgroup$

3

$\begingroup$

Если вы можете использовать известные ряды для $\sin(x)$ и $\cos(x)$, составьте уравнение $(\cos x)*(\tan x) = \sin x$, где вы подставите неизвестные константы как коэффициенты ряда $\tan x$.

18 в 7 степени: § Возведение в степень онлайн. Калькулятор «Возведение в степень»

Human Papillomavirus высокого канцерогенного риска (16, 18, 31, 33, 35, 39, 45, 51, 52, 56, 58, 59, 66, 68 типы), ДНК количественно, скрининг с определением возможности интеграции вируса в геном [реал-тайм ПЦР]

Исследование используется для диагностики папиллома-вирусной инфекции, с одновременным определением количества вирусной ДНК в исследуемом материале. Анализ позволяет сделать вывод о степени поражения слизистой, оценить эффективность терапии в процессе лечения. Определяется суммарное количество ДНК 14 типов ВПЧ высокого канцерогенного риска (ВКР) с отдельным определением ДНК ВПЧ 16-го, 18-го и 45-го генотипов. Тест также позволяет оценить вероятность интеграции ВПЧ 16-го, 18-го и 45-го типа в геном человека (что является опасным и прогностически неблагоприятным фактором).

Синонимы русские

Папиллома-вирусы высокого канцерогенного риска, вирусы папилломы высокого канцерогенного риска, ВПЧ высокого канцерогенного риска.

Синонимы английские

Human Papillomavirus, DNA; HPV, Viral Load.

Метод исследования

Полимеразная цепная реакция в режиме реального времени.

Какой биоматериал можно использовать для исследования?

Соскоб урогенитальный.

Как правильно подготовиться к исследованию?

  • Женщинам исследование рекомендуется производить до менструации или через 2-3 дня после её окончания.
  • Мужчинам не мочиться в течение 3 часов до взятия урогенитального мазка.

Общая информация об исследовании

Вирусы папилломы человека (ВПЧ) представляют собой широко распространённую и вариабельную группу вирусов, поражающих эпителий кожи и слизистых оболочек и обладающих онкогенным потенциалом. ВПЧ передаётся при тесном контакте с инфицированным эпителием, поэтому основные пути заражения – половой и контактно-бытовой. Возможна передача ВПЧ от инфицированной матери к плоду.

К факторам, провоцирующим развитие ВПЧ-инфекции, относятся ранее начало половой жизни, большое количество половых партнеров, сниженный иммунитет, применение оральных контрацептивов, авитаминозы, инфекции, передаваемые половым путём, курение и проживание в крупных городах.

Инкубационный период может длиться от 2 месяцев до 2-10 лет. Характерно скрытое течение заболевания, при котором отсутствуют клинические проявления, а при кольпоскопическом, цитологическом и гистологическом обследовании выявляется норма. В 30 % случаев в течение 6-12 месяцев может произойти избавление от вируса. Диагностика скрытой ВПЧ-инфекции осуществляется только методом ПЦР.

ВПЧ может по-разному воздействовать на эпителий: способствовать возникновению доброкачественных образований (папиллом, кондилом) или объединению ДНК папиллома-вируса с геном клетки, что приводит к дисплазии (неоплазии) и в дальнейшем к раку (чаще всего в переходной зоне шейки матки).

К папиллома-вирусам высокого онкогенного риска относятся ВПЧ типов 16, 18, 31, 33, 35, 39, 45, 51, 52, 56, 58, 59, 66, 68. 

Для выявления возбудителя используется метод полимеразной цепной реакции, которая позволяет обнаружить ДНК вируса. Принцип метода основан на многократном увеличении числа копий специфичного для данного возбудителя участка ДНК.

Полимеразная цепная реакция (ПЦР) – это метод молекулярной диагностики, позволяющий выявлять генетический материал возбудителя на ранних этапах заболевания. Он характеризуется высокими показателями диагностической чувствительности и специфичности, быстротой получения конечного результата и доступностью. Особенностью метода является возможность выявлять ДНК вируса даже при малом содержании её в исследуемом биологическом материале. Метод основан на многократном увеличении числа копий специфичного для данного возбудителя участка ДНК. Для диагностики может использоваться разнообразный биологический материал, взятый у детей, взрослых лиц, людей с патологией иммунной системы, беременных женщин. Метод позволяет определять количество молекул ДНК исследуемого вируса, что является важным в диагностике острой формы инфекции, реактивации персистентной инфекции, носительстве вируса, а также при контроле эффективности проводимой противовирусной терапии.

По данным исследований, количественное содержание вируса в материале коррелирует со степенью неоплазии: чем выше вирусная нагрузка, тем выраженнее цитологические изменения в эпителии. При обследовании необходимо учитывать генотип вируса, наличие и степень цитологических и гистологических изменений в тканях, увеличение или снижение вирусной нагрузки через несколько месяцев (6 месяцев и более) после предыдущего обследования.

В исследовании проводится определение суммарного количества ДНК 14 типов ВПЧ высокого канцерогенного риска (ВКР) с отдельным определением ДНК ВПЧ 16, 18 и 45 генотипов. ВПЧ ВКР являются главной этиологической причиной развития РШМ и предшествующей ему тяжелой дисплазии. ВПЧ обнаруживается в 95 % случаев РШМ, из них 16, 18, 45 генотипы в совокупности являются причиной 75 % плоскоклеточных раков и 94 % аденокарцином шейки матки.

Известно, что развитие РШМ часто ассоциировано с интеграцией ДНК вируса в геном клетки-хозяина. Наиболее часто интегрируют ВПЧ 16 и 18 генотипов, при этом происходит разрыв участка Е1/Е2 при сохранении онкогена Е6/Е7. В тест-системе, на которой выполняется данное исследование, по разным каналам прибора детектируются разные участки генома ВПЧ 16, 18 и 45 типов. Выявление области Е6 при отсутствии области Е1/Е2 позволяет косвенно судить о возможности интеграции вируса в геном человека. В случае обнаружения возможности интеграции вируса в геном в бланк ответа выводится соответствующая информация.

Для чего используется исследование?

  • Чтобы оценить риск развития новообразований, ассоциированных с ВПЧ (рак шейки матки, рак прямой кишки, рак аногенитальной области и других локализаций).
  • Для контроля за эффективностью лечения предраковых заболеваний, ассоциированных с ВПЧ.
  • Для мониторинга папиллома-вирусной инфекции и прогнозирования ее течения.

Когда назначается исследование?

  • При выявлении цитологических изменений в мазке на атипию, в мазке по Папаниколау, в гистологическом препарате.
  • При кондиломах и других морфологических изменениях половых путей.
  • При скрининге рака шейки матки у женщин старше 25-30 лет (в качестве дополнительного исследования).
  • При наблюдении за инфицированными ВПЧ.
  • При лечении рака и предраковых состояний, ассоциированных с ВПЧ.

Что означают результаты?

Уровень вирусной нагрузки интерпретируется с учетом результатов цитологического исследования мазков, гистологических изменений в биоптате и генотипа вируса, изменении его количества с течением времени.

  • Количество ДНК ВПЧ не определяется при отсутствии вируса в исследуемом образце или его минимальном количестве (ниже детектируемого уровня) – риск развития патологического процесса, связанного с ВПЧ, минимальный.
  • Клинически малозначимая концентрация вируса (менее 103 копий ДНК ВПЧ на 105 клеток) – минимальный риск развития дисплазии, транзиторное течение вирусного процесса.
  • Клинически значимая концентрация вируса (более 103 копий ДНК ВПЧ на 105 клеток) – хроническая инфекция с высоким риском развития дисплазии и РШМ.
  • Более 105 копий ДНК ВПЧ на 105 клеток при установленном факте персистентного течения инфекции (ВПЧ выявляется более 1 года) – усиленная вирусная нагрузка, ассоциированная с повышенным риском тяжелой дисплазии, часто встречается при РШМ.
  • Снижение вирусной нагрузки в 10 раз за 6 месяцев – транзиторная инфекция.
  • Рост вирусной нагрузки через 6 и более месяцев после лечения указывает на возможность рецидива.

Важные замечания

  • Инфицирование ВПЧ не всегда приводит к раку шейки матки.
  • Возможно одновременное заражение несколькими генотипами ВПЧ.
  • Результат анализа должен интерпретироваться с учетом заключений цитологического и гистологического исследований.

Также рекомендуется

  • Human Papillomavirus 16/18 (HPV 16/18), ДНК [реал-тайм ПЦР]
  • Human Papillomavirus 6/11 (HPV 6/11), ДНК [реал-тайм ПЦР]
  • Human Papillomavirus высокого канцерогенного риска (16, 18, 31, 33, 35, 39, 45, 51, 52, 56, 58, 59 типы), ДНК генотипирование [реал-тайм ПЦР]
  • Цитологическое исследование мазков (соскобов) с поверхности шейки матки (наружного маточного зева) и цервикального канала – окрашивание по Папаниколау (Рар-тест)
  • Цитологическое исследование мазков (соскобов) с поверхности шейки матки (наружного маточного зева) и цервикального канала на атипию
  • Антиген плоскоклеточной карциномы (SCCA)

Кто назначает исследование?

Акушер-гинеколог, дерматовенеролог, уролог, инфекционист, проктолог, педиатр, ЛОР.

Литература

  • Arbyn M. et al. (2010). «European Guidelines for Quality Assurance in Cervical Cancer Screening. Second Edition – Summary Document». Annals of Oncology 21 (3): 448 – 458.
  • Hsiu-Ting Tsai, Ching-Hu Wu, Hsiao-Ling Lai, et al. Association between Quantitative High-Risk Human Papillomavirus DNA Load and Cervical Intraepithelial Neoplasm Risk Cancer Epidemiol Biomarkers Prev 2005;14:2544-2549.
  • Saslow D, Solomon D, Lawson HW, et al. American Cancer Society, American Society for Colposcopy and Cervical Pathology, and American Society for Clinical Pathology Screening Guidelines for the Prevention and Early Detection of Cervical Cancer. Am J Clin Pathol. 2012;137:516-542.
  • «Genital HPV Infection – CDC Fact Sheet». Centers for Disease Control and Prevention (CDC). April 10, 2008. Retrieved 13 November 2009.
  • Материалы и рекомендации Противоракового Общества России. Режим доступа: http://www.pror.ru/

Вирус папилломы человека (ВПЧ)

Вирус папилломы человека (ВПЧ) — одна из самых распространённых инфекций, передающихся половым путём и при контакте «кожа-к-коже». Нет точной статистики, сколько людей в мире заражены ВПЧ, потому что у большинства заражённых вирус никак себя не проявляет.

Анализы

НК вируса папилломы (HPV) 16/18 типов

3-4 дня

от 215 ₽

В корзину

НК вируса папилломы человека — скрининг (16, 18, 31, 33, 35, 39, 45, 51, 52, 56, 58, 59 типы)

3-4 дня

от 450 ₽

В корзину

НК вируса папилломы человека — генотипирование (16, 18, 31, 33, 35, 39, 45, 51, 52, 56, 58, 59 типы)

3-4 дня

от 1 000 ₽

В корзину

Биологи считают, что около 80 % населения земли — носители ВПЧ.

Опасность вируса в том, что некоторые его типы могут привести к онкологии. 98 % рака шейки матки (по данным учёных США — 100 %) связаны с ВПЧ. Вирус провоцирует рак влагалища, вульвы, анального канала, мужских половых органов, горла, рта. Причём смертельно опасное заболевание может развиться спустя годы, десятилетия после заражения, если возникнут благоприятные для этого условия.

Всего в группу ВПЧ входит более 170 вирусов и штаммов, из них примерно 40 передаются половым путём, а 13 способны вызвать рак.

Заражение отдельными типами ВПЧ — при наличии высокого иммунитета — проходит незаметно, бесследно для организма. Другие штаммы заставляют клетки кожи усиленно делиться, в итоге появляются бородавки, папилломы, кондиломы («венерические бородавки»). Третьи типы, интегрируя в ДНК человека, действуют как онкогены и способствуют злокачественной трансформации клеток, росту опухолей.

Это важно! ВПЧ не является достаточным фактором для развития онкологического заболевания. Но он — один из важнейших онкофаторов, «провокатор» озлокачествления клеток.

Чтобы началось перерождение здоровой ткани в раковую, необходимо сочетание нескольких условий, в их числе — сбои в работе иммунной системы. Именно иммунитет — главный защитник в том числе и от увеличения вирусной нагрузки ВПЧ. Очень важно периодически сдавать анализа на ВПЧ, чтобы предотвратить опасность, вызванную вирусом. Так, можно вылечить предраковое состояние шейки матки. Другие онкологии, связанные с ВПЧ, лучше поддаются лечению, если оно начинается на ранней стадии заболевания и контролируется периодическими исследованиями на снижение/повышение вирусной нагрузки.

ВПЧ: способы заражения, симптомы, онкогенные типы вируса


Основные способы заражения:
  • вагинальный, оральный, анальный секс с человеком, заражённым ВПЧ;
  • при контактах «кожа-к-коже» с человеком, заражённым папилломавирусом, или контакте с поверхностями, которых касался заражённый человек — бытовое заражение возможно, если на коже имеются порезы, ссадины, другие повреждения;
  • при родах — от матери к ребёнку.

Последние исследования американских и европейских учёных показали, что велика вероятность заражения ВПЧ в медицинских учреждениях — при переливании крови, использовании медицинского оборудования, вдыхании вирусных частиц, например — при лазерной абляции или электрокоагуляции кондилом.

Сложность борьбы с неполовыми формами заражениями в том, что вирус чрезвычайно устойчив к большинству дезинфицирующих средств. ВПЧ — первый вирус, оказавшийся нечувствительным к инактивации (обработке) глутаровым альдегидом (средством для стерилизации хирургических инструментов, требующих абсолютной чистоты). Перед врачами и технологами встала проблема обеззараживания приборов, которые нельзя автоклавировать и подвергать воздействию агрессивных химических соединений.


Симптомы и типы вируса папилломы человека 

Симптомы заражения различаются в зависимости от типа ВПЧ. Некоторые типы, например — ВПЧ5, сохраняются в организме человека без клинических симптомов и могут быть обнаружены только специальными исследованиями. Штаммы ВПЧ1, 2, 4, 7, 22, 63 вызывают образование бородавок на руках, ногах, подошвах.

Типы 6, 11, 42, 44 способны вызвать развитие генитальных бородавок, папилломатоза гортани; 6, 16, 18, 31 и другие — анальной дисплазии; 60 — вирусной кисты.

К раку половых органов способны привести штаммы 26, 53, 66. Штаммы с высоким онкориском — 33, 35, 39, 51, 52, 56, 58, 59. Самым высоким риском трансформации клеток в злокачественные обладают типы 16, 18, 31, 45.

Анализы на ВПЧ у женщин и мужчин. Когда назначаются?

Анализы на ВПЧ нужно пройти каждому взрослому человеку самостоятельно, без назначения, т. к. вероятность, что вы заражены — 8 из 10. Направление на анализ обычно выдаётся дерматологом, урологом, гинекологом при наличии характерных внешних признаков или заболеваний, причиной которых может стать вирус.

Анализы на ВПЧ сдают при планировании беременности, при выявлении причин и лечении бесплодия, патологий беременности и вынашивания. В этом случае анализы сдают оба партнёра.

Факторами риска и поводом для сдачи анализа у женщин также являются:
  • ранняя половая жизнь;
  • отношения с разными, иногда сразу несколькими половыми партнёрами;
  • общие хронические, гинекологические заболевания, патологии;
  • слабый иммунитет.
Факторами риска и поводом для сдачи анализа у мужчин также являются:
  • множественные половые контакты;
  • половые контакты с женщинами, заражёнными ВПЧ;
  • плохая гигиена;
  • сужение крайней плоти;
  • слабый иммунитет.

Методы диагностики ВПЧ

Кольпоскопическое исследование

Кольпоскопия — осмотр с помощью кольпоскопа влагалищной части шейки матки, входа и стенок влагалища. Это простой, недорогой, но высокоинформативный метод диагностирования заболеваний шейки матки. Клиническое значение имеет расширенная кольпоскопия с применением нескольких тестов — с 3-процентной уксусной кислотой, йодным раствором Люголя. Тесты выявляют различные типы эпителия, позволяют оценить размеры и качество патологических образований (при их наличии), сосудистый рисунок, качество шеечных желёз.

Во время кольпоскопии проводят прицельную биопсию с наиболее атипично изменённых участков.

Цитологическое исследование

Задача цитологического исследования шеечных мазков (тест Папаниколау, пап-тест) — выявление специфических для ВПЧ-инфекции клеток — койлоцитов и дискератоцитов.

Подтверждением папилломовирусной инфекции считается обнаружение койлоцитов, трансэпителиальной лимфоцитарной инфильтрации, базально-клеточной гиперплазии в биоптате (биоматериале, взятом на исследование).

Пап-тест обязателен для:

  • женщин после 30 лет;
  • женщин, у кого был ранее диагностирован ВПЧ;
  • женщин, у кого во время кольпоскопии обнаружили зоны с изменённым эпителием.

По результатам пап-теста определяют класс опасности для здоровья женщины: 1—2 класс — без подозрения на рак, 3 класс — подозрение на онкологию, 4—5 класс — наличие раковых клеток в малом или большом количестве.

К недостаткам цитологического исследования относят сложность исполнения, высокие квалификационные требования к врачу-цитологу. Потому проходить исследование нужно в диагностических центрах и лабораториях, персонал которых постоянно подтверждает свой профессионализм.

Гистологическое исследование

Гистологический метод обнаружения ВПЧ можно было бы считать золотым стандартом диагностики вируса, однако мешает его высокая стоимость, невозможность частого проведения и не всегда точный прицельный забор биоптата из шейки матки. Для проведения гистологической диагностики также требуются специалисты очень высокой квалификации.

Поэтому гистологическое исследование биоптата часто служит дополнением к пап-анализу. Оно позволяет оценить состояние клеток, степень поражения, определить, чем является новообразование — опухолью или кондиломой.

ПЦР диагностика папилломавируса 

Полимеразная цепная реакция (ПЦР) относится к высокоразрешающим технологиям детекции нуклеиновых кислот. Современные ПЦР тест-системы обладают высокой чувствительностью, используются не только для выявления ВПЧ, но и вирусной нагрузки на организм (количественный показатель заражённости) главных клинически значимых генотипов (16, 18, 31, 33, 35, 39, 45, 51, 52, 56, 58, 59), которые ответственны за почти 94 % случаев тяжёлых цервикальных дисплазий и рака шейки матки. ПЦР тест-системы используют также для обнаружения штаммов ВПЧ 6 и 11.

Такие возможности тест-системы позволяют прогнозировать течение ВПЧ-инфекции, оценивать эффективность терапии. Установлено, что папилломавирусная инфекция имеет дозозависимый эффект: чем выше концентрация ДНК вируса в исследуемом материале, тем выше риск развития неоплазии и раковой опухоли.

В результатах теста указывают концентрацию ВПЧ:

  • Lg < 3 — папилломавирус обнаружен в клинически незначимом количестве;
  • Lg 3–5 — папилломавирус обнаружен в клинически значимом количестве;
  • Lg > 5 — папилломавирус обнаружен в высокой концентрации, вирусная нагрузка на организм высокая.

Виды исследуемого материала, правила забора:

  • для пап-теста — шеечный мазок;
  • для цитологического исследования — биоптат, взятый прицельно с атипично изменённых участков слизистой.
  • для ПЦР-теста — соскоб клеток слизистых оболочек генитального тракта.

Мазок у женщин берут из цервикального канала, у мужчин — из уретры. Для мазка используют мягкую щётку или ватный тампон. Их бережно вводят в канал, затем осторожно вынимают, вращая. На поверхности щётки/тампона остаются эпителиальные клетки, нужные для исследования.

Как подготовиться к анализу на ВПЧ?


Взятие биоматериала не проводится во время менструации, за 5 дней до её начала и в течение 5 дней по окончании. Нельзя проходить исследование, если есть воспалительные процессы.

За 2 суток до забора биоматериала женщинам и мужчинам нужно воздержаться от сексуальных контактов. Женщинам нельзя в течение 48 часов использовать вагинальные кремы, лекарства, суппозитории, спринцевания, тампоны, вместо ванны нужно принимать душ. Мазок берут до проведения любых гинекологических манипуляций или через 2 суток после них.

Если биоматериал берут из уретры, то от последнего мочеиспускания до забора биоматериала должно пройти не менее 90 минут.

При заборе биоптата для гистологического исследования соблюдаются те же правила, что и при подготовке к забору мазка. 

После биопсии в течение 2—3 недель нельзя:

вступать в половые контакты,

  • испытывать значительные физические нагрузки,
  • перегреваться (баня, сауна, жаркая погода),
  • купаться в открытом водоёме или бассейне,
  • принимать препараты, разжижающие кровь;
  • использовать вагинальные средства.

Если возникнет кровотечение, следует пользоваться только прокладками (не тампонами!)

На точность результата могут влиять антибиотики, пробиотики, местные антисептики, которые вы принимали/применяли даже 2 месяца назад. Перед сбором материала нужно рассказать врачу обо всех лекарственных средствах, которыми вы пользовались или пользуетесь.

Стоимость исследования на наличие папилломавирусной инфекции в АО «СЗДЦМ»

Стоимость исследования на заражение ВПЧ зависит от вида исследования, охвата штаммов вируса, определения типа/без определения, расчёта вирусной нагрузки.

Цены на виды исследований в медицинских подразделениях АО «СЗДЦМ» представляют собой разнообразные комбинации по охватности, подробности, прогностической ценности.

Какой способ исследования выбрать, вам подскажет врач — гинеколог, дерматолог, венеролог. Если вы сдаёте анализы по собственной инициативе, выберите исследование на наличие онкогенных штаммов ВПЧ.

Где сдать анализы на ВПЧ?

Анализы на ВПЧ вы можете сдать в медицинских подразделениях АО «СЗДЦМ», расположенных в Санкт-Петербурге, Ленинградской области, в Великом Новгороде, Старой Руссе и других городах.

Чтобы найти ближайший к вам пункт, воспользуйтесь интерактивной картой или перечнем медицинских учреждений АО «СЗДЦМ».

Во всех наших отделениях — терминалах и центрах — вас встретят внимательные, опытные специалисты с высокой квалификацией. Медицинские учреждения АО «СЗДЦМ» оснащены современным оборудованием, лабораторными материалами, одноразовыми инструментами и расходными материалами.

Мы гарантируем вам точность исследований, заботливое отношение, полную конфиденциальность ваших личных данных и результатов обследования.

Будьте здоровы! А для этого вовремя и регулярно проходите важные обследования в АО «СЗДЦМ».
Мы поможем вам сохранить и вернуть здоровье! 

Сколько будет 5 в 7-й степени?

Итак, вы хотите знать, сколько будет 5 в 7-й степени, не так ли? В этой статье мы объясним, как именно выполнить математическую операцию под названием «возведение числа 5 в степень 7». Это может показаться фантастическим, но мы объясним это без жаргона! Давай сделаем это.

Что такое возведение в степень?

Давайте сначала зафиксируем наши термины, а затем посмотрим, как вычислить число 5 в 7-й степени.

Когда мы говорим об возведении в степень, все, что мы на самом деле имеем в виду, это то, что мы умножаем число, которое мы называем 9) для обозначения показателя степени. Знак вставки полезен в ситуациях, когда вы не хотите или не нуждаетесь в использовании надстрочного индекса.

Итак, мы упомянули, что возведение в степень означает умножение базового числа само на себя для получения показателя степени число раз. Давайте посмотрим на это более наглядно:

5 в 7-й степени = 5 x … x 5 (7 раз)

Итак, каков ответ?

Теперь, когда мы объяснили теорию, лежащую в основе этого, давайте поработаем над числами и выясним, чему равно 5 в 7-й степени:

5 в степени 7 = 5 7 = 78 125

Почему мы вообще используем возведение в степень 5 7 ? Что ж, нам намного проще писать умножения и выполнять математические операции как с большими, так и с маленькими числами, когда вы работаете с числами с большим количеством конечных нулей или большим количеством десятичных знаков.

Надеюсь, эта статья помогла вам понять, как и почему мы используем возведение в степень, и дала вам ответ, который вы изначально искали. Теперь, когда вы знаете, что такое 5 в 7-й степени, вы можете продолжить свой веселый путь.

Не стесняйтесь поделиться этой статьей с другом, если вы считаете, что она поможет ему, или перейдите вниз, чтобы найти еще несколько примеров.

Процитируйте, дайте ссылку или ссылку на эту страницу

Если вы нашли этот контент полезным в своем исследовании, пожалуйста, сделайте нам большую услугу и используйте приведенный ниже инструмент, чтобы убедиться, что вы правильно ссылаетесь на нас, где бы вы его ни использовали. Мы очень ценим вашу поддержку!

  • Сколько будет 5 в 7-й степени?

  • «Сколько будет 5 в 7-й степени?». VisualFractions.com . По состоянию на 27 апреля 2023 г. http://visualfractions.com/calculator/exponent/what-is-5-to-the-7th-power/.

  • «Сколько будет 5 в 7-й степени?». VisualFractions.com , http://visualfractions. com/calculator/exponent/what-is-5-to-the-7th-power/. По состоянию на 27 апреля 2023 г.

  • Сколько будет 5 в 7-й степени?. VisualFractions.com. Получено с http://visualfractions.com/calculator/exponent/what-is-5-to-the-7th-power/.

Калькулятор возведения в степень

Хотите найти решение еще одной задачи? Введите число и мощность ниже и нажмите «Рассчитать».

Вычисление возведения в степень

Случайный список примеров возведения в степень

Если вы добрались до этого места, вам должно быть ДЕЙСТВИТЕЛЬНО нравится возведение в степень! Вот несколько случайных вычислений:

Сколько будет 48 в 52-й степени?

Сколько будет 24 в 63-й степени?

Сколько будет 14 в 45-й степени?

Сколько будет 38 в 33-й степени?

Сколько будет 60 в 33-й степени?

Сколько будет 12 в 13-й степени?

Сколько будет 74 в 37-й степени?

Сколько будет 26 в 8-й степени?

Сколько будет 63 в 51-й степени?

Сколько будет 88 в 30-й степени?

Сколько будет 42 в 97-й степени?

Сколько будет 57 в 9-й степени?

Сколько будет 34 в 49-й степени?

Сколько будет 89 в 59-й степени?

Сколько будет 31 в 96-я сила?

Сколько будет 88 в 89-й степени?

Сколько будет 26 в 94-й степени?

Сколько будет 54 в 17-й степени?

Сколько будет 60 в 32-й степени?

Сколько будет 95 в 60-й степени?

Сколько будет 48 в 73-й степени?

Сколько будет 26 в 60-й степени?

Сколько будет 22 в 35-й степени?

Сколько будет 66 в 84-й степени?

Сколько будет 45 в 37-й степени?

Сколько будет 48 в 18-й степени?

Сколько будет 43 в 57-й степени?

Сколько будет 42 в 63-й степени?

Сколько будет 57 в 80-й степени?

Сколько будет 45 в 14-й степени?

Сколько будет 19 в 36-й степени?

Сколько будет 34 в 77-й степени?

Сколько будет 4 в 54-й степени?

Сколько будет 44 в 31-й степени?

Сколько будет 41 в 38-й степени?

Сколько будет 5 в 74-й степени?

Сколько будет 50 в 66-й степени?

Сколько будет 87 в 49-й степени?

Сколько 50 к 79сила?

Сколько будет 25 в 84-й степени?

Сколько будет 51 в 64-й степени?

Сколько будет 57 в 63-й степени?

Сколько будет 99 в 29-й степени?

Сколько будет 395 в 3-й степени?

Сколько будет 100 в 77-й степени?

Сколько будет 70 в 47-й степени?

Сколько будет 37 в 48-й степени?

Сколько будет 60 в 47-й степени?

Сколько будет 69 в 87-й степени?

Сколько будет 17 в 41 степени?

Сколько будет 47 в 21-й степени?

Сколько будет 59 в 81 степени?

Сколько будет 68 в 30-й степени?

Сколько будет 21 в 62-й степени?

Сколько будет 71 в 73-й степени?

Сколько будет 34 в 79-й степени?

Сколько будет 57 в 30-й степени?

Сколько будет 26 в 36-й степени?

Сколько будет 85 в 24-й степени?

Сколько будет 3 в 39-й степени?

Сколько будет 17 в 14-й степени?

Сколько будет 99 в 98-й степени?

Сколько 32 к 29сила?

Сколько будет 99 в 45-й степени?

Сколько будет 20 в 25-й степени?

Что такое 570392 во 2-й степени?

Сколько будет 11 в 26-й степени?

Сколько будет 81 в 39-й степени?

Сколько будет 41 в 68-й степени?

Сколько будет 78 в 68-й степени?

Сколько будет 2 в 4-й степени?

Сколько будет 26 в 8-й степени?

Сколько будет 13 в 80-й степени?

Сколько будет 84 в 81-й степени?

Что такое 9к 69-й державе?

Сколько будет 95 в 24-й степени?

Сколько будет 67 в 44-й степени?

Сколько будет 93 в 34-й степени?

Сколько будет 81 в 14-й степени?

Сколько будет 14 в 95-й степени?

Сколько будет 31 в 10-й степени?

Сколько будет 48 в 83-й степени?

Сколько будет 94 в 63-й степени?

Сколько будет 100 в 91-й степени?

Сколько будет 58 в 75-й степени?

Сколько будет 49 в 47-й степени?

Сколько будет 35 в 46-й степени?

Сколько будет 56 в 26-й степени?

Сколько будет 62 в 34-й степени?

Сколько будет 85 в 45-й степени?

Сколько будет 44 в 48-й степени?

Сколько будет 90 в 7-й степени?

Сколько будет 33 в 20-й степени?

Сколько будет 68 в 44-й степени?

Сколько будет 26 в 85-й степени?

Сколько будет 36 в 17-й степени?

Сколько будет 17 в 23-й степени?

Сколько будет 97 в 45-й степени?

Сколько будет 44 в 42 степени?

Сколько будет 100 в 78-й степени?

Сколько будет 64 в 12-й степени?

Калькулятор и конвертер научных представлений

9029 1
Научное представление Десятичное число
Миллион в экспоненциальном представлении 1 × 10 6 1 000 000
10 Миллион в научном выражении обозначение 1 × 10 7 10 000 000
100 миллионов в экспоненциальном представлении 1 × 10 8 100 000 ,000
Миллиард в экспоненциальном представлении 1 × 10 9 1 000 000 000
10 Миллиарды в экспоненциальном представлении 1 × 10 10 10 000 000 000
100 Миллиарды в экспоненциальном представлении 1 × 10 900 20 11 100 000 000 000
Триллион в экспоненциальном представлении 1 × 10 12 1 000 000 000 000
10 Триллион в экспоненциальном представлении 1 × 10 13 10 000 000 000 000
1 00 триллионов в научных нотация 1 × 10 14 100 000 000 000 000

Экспоненциальная запись: сложение и вычитание

Научное обозначение: умножение и деление

В каждой области науки приходится иметь дело с числами, числа могут быть в любом формате, например, вес земли в килограммах — это очень большое число или масса электрон — очень маленькое число. Представление этих типов чисел непросто.

Предположим, что имеется число с 22 нулями, записанное таким образом:

602200000000000000000000000

Это число нелегко прочитать или понять. Итак, существует способ, известный как научная запись, которую мы можем использовать для записи очень больших или малых чисел более простым и понятным способом.

  • Десятичная в научной записи
  • Десятичная в научной записи

 

Важность научной записи

В 1998 году НАСА запустило орбитальный аппарат для поиска данных об изменении климата на Марсе, но через три года после выхода на орбиту э заблудился и расследование на этот вопрос показывает, что оценка данных была неправильной из-за того, что две команды передают свои данные в разных единицах измерения. Представление числа в стандартной форме очень важно, потому что точность чисел имеет значение, читая слишком большое или слишком маленькое число, мы можем ошибиться при подсчете нулей.

 

Как представить числа в экспоненциальном представлении?

Числа записываются в произведение так, как если бы первое число было мантисса, а второе число было бы степенью 10:

Любое число = мантисса x 10 большое количество их научное обозначение будет таким:

7 000 000 000 000 000 000 000 = 7 × 10 21

Здесь 7 — мантисса, а 21 — показатель степени. Теперь мы видим, что при чтении и записи этих типов чисел вероятность ошибки уменьшается, и мы можем легко использовать эти числа в любой письменной работе.

Прежде чем использовать научные обозначения, давайте найдем лежащую в их основе теорию. Как мы можем найти различные степени для 10?

  • 10 0 = 1
  • 10¹ = 10
  • 10³ = 1000

Здесь 10 степень помогает найти нули, например, когда степень равна 0, нуля нет следуйте за 1, когда мощность равна 3, есть 3 нули следуют за 1. Аналогично 10 100 = 1000….0 представляют 100 нулей. Это очень большое число, и его нелегко прочитать, поэтому с помощью научных обозначений представление очень большого числа становится легким.

Чтобы выразить числа в экспоненциальном представлении, слева от десятичной точки должна оставаться только одна цифра, для этого десятичная точка перемещается влево или вправо в зависимости от числа, больше или меньше нуля . Умножение на другой показатель степени позволяет нам перемещать десятичные разряды.

 

Например, в

9,2867 × 10 4 = 92876,00

 

показатель степени 10 положителен, он позволяет десятичной дроби двигаться вправо. В то время как, с другой стороны, отрицательная экспонента сдвигает десятичную дробь влево что мы должны оставить всего одна цифра слева от запятой.

Еще одно преимущество научного представления заключается в том, что мы можем легко складывать, вычитать, умножать и делить большие числа. Давайте посмотрим, как мы можем использовать этот метод для сложения чисел.

 

Для сложения или вычитания чисел в экспоненциальном представлении используются следующие шаги:

  • показатели степени 10 должны оставаться одинаковыми в обоих терминах.
  • Уравняв степени 10, мы можем складывать или вычитать числа.
  • Наконец, мы должны скорректировать ответ в соответствии с экспоненциальной записью.

 

Добавить экспоненциальное представление: (5,7 × 10 4 ) + (2,25 × 10 5 )
  1. ponent в зависимости от малой мощности.

5,7 × 10 4 ) + (2,25 × 10 1 × 10 4 )

  1. Теперь сгруппируйте числа
9 0002 (5,7 + 2,25 × 10 1 ) × 10 4
(5,7 + 22,5) × 10 4
28,2 × 10 4

  1. Теперь скорректируйте число в соответствии со стандартной формой научного представления 003

     

    Аналогичным образом мы можем вычесть две величины .

    Вычесть экспоненциальное представление: (6,67 × 10 8 ) – (8,4 × 10 6 )
    1. Отрегулируйте степень большой экспоненты в соответствии с малой степенью.

    (6,67 × 10 2 × 10 6 ) – (8,4 × 10 6 )

    1. Теперь сгруппируйте числа

    (667 – 8,4) × 10 6 905 44 658,6 × 10 6

    1. Теперь отрегулируйте число в соответствии с к стандартной форме экспоненциального представления

    (6,58 × 10 2 ) × 10 6 = 6,58 × 10 8

      900 03

    Все числа в экспоненциальном представлении имеют основание 10, поэтому мы можем легко умножать и разделить их:

    1. Умножаем и делим два числа, умножаем или делим их мантиссы, складываем и вычитаем степени показателей соответственно.
    2. В обоих случаях мы должны преобразовать число в стандартную форму научного представления.

Sin 2 x 2 cos x 0: Mathway | Популярные задачи

Mathway | Популярные задачи

1Найти точное значениеsin(30)
2Найти точное значениеsin(45)
3Найти точное значениеsin(30 град. )
4Найти точное значениеsin(60 град. )
5Найти точное значениеtan(30 град. )
6Найти точное значениеarcsin(-1)
7Найти точное значениеsin(pi/6)
8Найти точное значениеcos(pi/4)
9Найти точное значениеsin(45 град. )
10Найти точное значениеsin(pi/3)
11Найти точное значениеarctan(-1)
12Найти точное значениеcos(45 град. )
13Найти точное значениеcos(30 град. )
14Найти точное значениеtan(60)
15Найти точное значениеcsc(45 град. )
16Найти точное значениеtan(60 град. )
17Найти точное значениеsec(30 град. )
18Найти точное значениеcos(60 град. )
19Найти точное значениеcos(150)
20Найти точное значениеsin(60)
21Найти точное значениеcos(pi/2)
22Найти точное значениеtan(45 град. )
23Найти точное значениеarctan(- квадратный корень из 3)
24Найти точное значениеcsc(60 град. )
25Найти точное значениеsec(45 град. )
26Найти точное значениеcsc(30 град. )
27Найти точное значениеsin(0)
28Найти точное значениеsin(120)
29Найти точное значениеcos(90)
30Преобразовать из радианов в градусыpi/3
31Найти точное значениеtan(30)
32Преобразовать из градусов в радианы45
33Найти точное значениеcos(45)
34Упроститьsin(theta)^2+cos(theta)^2
35Преобразовать из радианов в градусыpi/6
36Найти точное значениеcot(30 град. )
37Найти точное значениеarccos(-1)
38Найти точное значениеarctan(0)
39Найти точное значениеcot(60 град. )
40Преобразовать из градусов в радианы30
41Преобразовать из радианов в градусы(2pi)/3
42Найти точное значениеsin((5pi)/3)
43Найти точное значениеsin((3pi)/4)
44Найти точное значениеtan(pi/2)
45Найти точное значениеsin(300)
46Найти точное значениеcos(30)
47Найти точное значениеcos(60)
48Найти точное значениеcos(0)
49Найти точное значениеcos(135)
50Найти точное значениеcos((5pi)/3)
51Найти точное значениеcos(210)
52Найти точное значениеsec(60 град. )
53Найти точное значениеsin(300 град. )
54Преобразовать из градусов в радианы135
55Преобразовать из градусов в радианы150
56Преобразовать из радианов в градусы(5pi)/6
57Преобразовать из радианов в градусы(5pi)/3
58Преобразовать из градусов в радианы89 град.
59Преобразовать из градусов в радианы60
60Найти точное значениеsin(135 град. )
61Найти точное значениеsin(150)
62Найти точное значениеsin(240 град. )
63Найти точное значениеcot(45 град. )
64Преобразовать из радианов в градусы(5pi)/4
65Найти точное значениеsin(225)
66Найти точное значениеsin(240)
67Найти точное значениеcos(150 град. )
68Найти точное значениеtan(45)
69Вычислитьsin(30 град. )
70Найти точное значениеsec(0)
71Найти точное значениеcos((5pi)/6)
72Найти точное значениеcsc(30)
73Найти точное значениеarcsin(( квадратный корень из 2)/2)
74Найти точное значениеtan((5pi)/3)
75Найти точное значениеtan(0)
76Вычислитьsin(60 град. )
77Найти точное значениеarctan(-( квадратный корень из 3)/3)
78Преобразовать из радианов в градусы(3pi)/4
79Найти точное значениеsin((7pi)/4)
80Найти точное значениеarcsin(-1/2)
81Найти точное значениеsin((4pi)/3)
82Найти точное значениеcsc(45)
83Упроститьarctan( квадратный корень из 3)
84Найти точное значениеsin(135)
85Найти точное значениеsin(105)
86Найти точное значениеsin(150 град. )
87Найти точное значениеsin((2pi)/3)
88Найти точное значениеtan((2pi)/3)
89Преобразовать из радианов в градусыpi/4
90Найти точное значениеsin(pi/2)
91Найти точное значениеsec(45)
92Найти точное значениеcos((5pi)/4)
93Найти точное значениеcos((7pi)/6)
94Найти точное значениеarcsin(0)
95Найти точное значениеsin(120 град. )
96Найти точное значениеtan((7pi)/6)
97Найти точное значениеcos(270)
98Найти точное значениеsin((7pi)/6)
99Найти точное значениеarcsin(-( квадратный корень из 2)/2)
100Преобразовать из градусов в радианы88 град.

Mathway | Популярные задачи

1Найти точное значениеsin(30)
2Найти точное значениеsin(45)
3Найти точное значениеsin(30 град. )
4Найти точное значениеsin(60 град. )
5Найти точное значениеtan(30 град. )
6Найти точное значениеarcsin(-1)
7Найти точное значениеsin(pi/6)
8Найти точное значениеcos(pi/4)
9Найти точное значениеsin(45 град. )
10Найти точное значениеsin(pi/3)
11Найти точное значениеarctan(-1)
12Найти точное значениеcos(45 град. )
13Найти точное значениеcos(30 град. )
14Найти точное значениеtan(60)
15Найти точное значениеcsc(45 град. )
16Найти точное значениеtan(60 град. )
17Найти точное значениеsec(30 град. )
18Найти точное значениеcos(60 град. )
19Найти точное значениеcos(150)
20Найти точное значениеsin(60)
21Найти точное значениеcos(pi/2)
22Найти точное значениеtan(45 град. )
23Найти точное значениеarctan(- квадратный корень из 3)
24Найти точное значениеcsc(60 град. )
25Найти точное значениеsec(45 град. )
26Найти точное значениеcsc(30 град. )
27Найти точное значениеsin(0)
28Найти точное значениеsin(120)
29Найти точное значениеcos(90)
30Преобразовать из радианов в градусыpi/3
31Найти точное значениеtan(30)
32Преобразовать из градусов в радианы45
33Найти точное значениеcos(45)
34Упроститьsin(theta)^2+cos(theta)^2
35Преобразовать из радианов в градусыpi/6
36Найти точное значениеcot(30 град. )
37Найти точное значениеarccos(-1)
38Найти точное значениеarctan(0)
39Найти точное значениеcot(60 град. )
40Преобразовать из градусов в радианы30
41Преобразовать из радианов в градусы(2pi)/3
42Найти точное значениеsin((5pi)/3)
43Найти точное значениеsin((3pi)/4)
44Найти точное значениеtan(pi/2)
45Найти точное значениеsin(300)
46Найти точное значениеcos(30)
47Найти точное значениеcos(60)
48Найти точное значениеcos(0)
49Найти точное значениеcos(135)
50Найти точное значениеcos((5pi)/3)
51Найти точное значениеcos(210)
52Найти точное значениеsec(60 град. )
53Найти точное значениеsin(300 град. )
54Преобразовать из градусов в радианы135
55Преобразовать из градусов в радианы150
56Преобразовать из радианов в градусы(5pi)/6
57Преобразовать из радианов в градусы(5pi)/3
58Преобразовать из градусов в радианы89 град.
59Преобразовать из градусов в радианы60
60Найти точное значениеsin(135 град. )
61Найти точное значениеsin(150)
62Найти точное значениеsin(240 град. )
63Найти точное значениеcot(45 град. )
64Преобразовать из радианов в градусы(5pi)/4
65Найти точное значениеsin(225)
66Найти точное значениеsin(240)
67Найти точное значениеcos(150 град. )
68Найти точное значениеtan(45)
69Вычислитьsin(30 град. )
70Найти точное значениеsec(0)
71Найти точное значениеcos((5pi)/6)
72Найти точное значениеcsc(30)
73Найти точное значениеarcsin(( квадратный корень из 2)/2)
74Найти точное значениеtan((5pi)/3)
75Найти точное значениеtan(0)
76Вычислитьsin(60 град. )
77Найти точное значениеarctan(-( квадратный корень из 3)/3)
78Преобразовать из радианов в градусы(3pi)/4
79Найти точное значениеsin((7pi)/4)
80Найти точное значениеarcsin(-1/2)
81Найти точное значениеsin((4pi)/3)
82Найти точное значениеcsc(45)
83Упроститьarctan( квадратный корень из 3)
84Найти точное значениеsin(135)
85Найти точное значениеsin(105)
86Найти точное значениеsin(150 град. )
87Найти точное значениеsin((2pi)/3)
88Найти точное значениеtan((2pi)/3)
89Преобразовать из радианов в градусыpi/4
90Найти точное значениеsin(pi/2)
91Найти точное значениеsec(45)
92Найти точное значениеcos((5pi)/4)
93Найти точное значениеcos((7pi)/6)
94Найти точное значениеarcsin(0)
95Найти точное значениеsin(120 град. )
96Найти точное значениеtan((7pi)/6)
97Найти точное значениеcos(270)
98Найти точное значениеsin((7pi)/6)
99Найти точное значениеarcsin(-( квадратный корень из 2)/2)
100Преобразовать из градусов в радианы88 град.
3 6 Решить для ? cos(x)=1/2 7 Найти x sin(x)=-1/2 8 Преобразование градусов в радианы 225 9 Решить для ? cos(x)=(квадратный корень из 2)/2 10 Найти x cos(x)=(квадратный корень из 3)/2 11 Найти x sin(x)=(квадратный корень из 3)/2 92=9 14 Преобразование градусов в радианы 120 градусов 15 Преобразование градусов в радианы 180 16 Найти точное значение желтовато-коричневый(195) 92-4 38 Найти точное значение грех(255) 39 Оценить лог база 27 из 36 40 Преобразовать из радианов в градусы 2 шт. 92-3sin(x)+1=0 43 Найти x tan(x)+ квадратный корень из 3=0 44 Найти x sin(2x)+cos(x)=0 45 Упростить (1-cos(x))(1+cos(x)) 92=25 59 График f(x)=- натуральный логарифм x-1+3 60 Найдите значение с помощью единичного круга угловой синус(-1/2) 61 Найти домен квадратный корень из 36-4x^2 92=0 66 Найти x cos(2x)=(квадратный корень из 2)/2 67 График у=3 68 График f(x)=- логарифмическая база 3 x-1+3 92 71 Найти x квадратный корень из x+4+ квадратный корень из x-1=5 72 Решить для ? cos(2x)=-1/2 73 Найти x логарифмическая база x из 16=4 9х 75 Упростить (cos(x))/(1-sin(x))+(1-sin(x))/(cos(x)) 76 Упростить сек(х)sin(х) 77 Упростить кубический корень из 24 кубический корень из 18 92=0 96 Найти x 3x+2=(5x-11)/(8г) 97 Решить для ? sin(2x)=-1/2 98 Найти x (2x-1)/(x+2)=4/5 92=0

.

 

Подписаться І 3

Подробнее

Отчет

2 ответа от опытных наставников

Лучший Новейшие Самый старый

Автор: ЛучшиеНовыеСамыеСтарые

Парвиз Ф. ответил 02.09.13

Репетитор

4,8 (4) 92 +2CosX -3 =0

   

      (CosX +3)( CosX -1 ) =0

  

       CosX =1 — единственный ответ.

             X = 0  , X= 2n(360)

   

 

Голосовать за 0 Понизить

Подробнее

Отчет

Дебби Д. ответил 22.04.13

92 -2y +1 = (y — 1)(y — 1), поэтому замените y на cos (x), и вы получите

(cos (x) — 1 )(cox (x) — 1 ) = 0  на ноль свойство продукта, cos (x) — 1 = 0,

, поэтому cos (x) = 1, а угол, косинус которого равен 1, равен 0,

, поэтому x = 0 (плюс любое число, кратное 360 градусам или 2 пи радианы)

Голосовать за 0 Понизить

Подробнее

Отчет

Все еще ищете помощи? Получите правильный ответ, быстро.

Юзгу тестирование: Электронная информационно-образовательная среда ЮЗГУ (версия 2.0) Информационный портал ЮЗГУ.

Электронная информационно-образовательная среда ЮЗГУ. Учебные курсы ЮЗГУ

Перейти к основному содержанию

Пропустить новости сайта


Пропустить Навигация Пропустить Вход Пропустить Основное меню Пропустить Ссылки

Расписание занятий студентов

Научная библиотека

Электронный каталог Научной библиотеки ЮЗГУ

Пропустить Календарь

мая 2023

1 2 3 4 5 6 7
8 9 10 11 12 13 14
15 16 17 18 19 20 21
22 23 24 25 26 27 28
29 30 31    
Пропустить Последние действия

Действия с Четверг, 18 мая 2023, 14:01

Полный отчет о последних действиях

Со времени Вашего последнего входа ничего не произошло

Пропустить Счетчики Версия сайта для слабовидящих

Заказать онлайн-тест ЮЗГУ (Юго-Западный государственный университет)

Как мы выполняем онлайн-тест ЮЗГУ

Оформить заявку

Гарантии заказа онлайн-теста ЮЗГУ

  • На протяжении выбранного вами гарантийного срока вы можете вернуть потраченные деньги, если качество написанной под заказ студенческой работы вас не устроит или не будет соответствовать требованиям учебного заведения.

  • Перед сдачей клиенту, каждая работа проходит тщательную проверку на плагиат по выбранному вами сервису.

  • Написанием работ занимаются только опытные авторы, которые хорошо знают тонкости учебной дисциплины и имеют практический опыт в отрасли.

  • Мы строго храним тайну сотрудничества между агентством по написанию студенческих работ и клиентом.

Отзывы наших клиентов

Узнайте почему выбирают студенты:

  • Хочу выразить благодарность за поддержку сотрудникам компании ФастФайн. Когда у меня возникли проблемы с написанием бакалаврской работы, мне сразу нашли автора, который был со мной на связи все время, даже после сдачи работы. Считаю, что профессионалам FastFine можно доверять написание любых работ, здесь «своих» не бросают.

  • Хотя бы раз в семестр обращался в компанию ФастФайн. Не обошелся без помощи профессионалов и при написании дипломной работы. Тема специфическая, касается литературы 18 века. Не хватало времени и сил поработать с информационными источниками. Автор оперативно и недорого подготовил теоретический раздел. Получил «отлично» на защите, за что искренне благодарен этой конторе. Всем будущим выпускника рекомендую!

  • Мне нужно было срочно заполнить сайт уникальными статьями. Нашла у конкурентов подходящий контент и решила заказать рерайт. Компанию fastfine нашла через поисковик, почитала отзывы. Цены и сроки меня устроили, решила попробовать. Если честно, даже не ожидала, что можно переписать уже готовый текст своими словами и получить такой высокий процент уникальности. Видно, что это авторская работа, даже стиль написания немного отличается от исходника. На мой взгляд, статьи получились намного лучше оригинальных, такие не стыдно выкладывать. Автору ставлю отличную оценку! Тем, кого интересует рерайт высокого качества, рекомендую сайт fastfine.ru.

Как сдать тест онлайн

Теперь каждый студент ЮЗГУ и школьник знает, что такое дистанционное обучение. Проверку знаний преподаватели тоже часто проводят удаленно. Самый распространенный вариант такой проверки — это тестирование.  

Проблема в том, что, если раньше ответы можно было подсмотреть у товарища, то теперь списать фактически невозможно. Даже если вы знаете ответы, вас может подвести слабая связь с интернетом или проблемы с компьютером. В общем, теперь тестирование проходит сложнее. Чтобы не рисковать успеваемостью, лучше купить тест онлайн.

Выполнение тестов онлайн: главные проблемы

Многие студенты ЮЗГУ наивно полагают, что смогут списать, вбив вопрос в поисковике. Но не все так просто. Их ждут трудности:

  • Они не находят нужный ответ. В интернете легко наткнуться на ложную информацию. Многие данные быстро устаревают.
  • Они не успевают найти ответ. Времени не много, а ответ на вопрос чаще всего спрятан где-то в длинных статьях.
  • Они сталкиваются с мошенниками, которые просят отправить СМС за правильный ответ.
  • Преподаватели замечают, что они списывают.

Любой вариант грозит двойкой за тест. А от него может зависеть общая оценка по дисциплине, зачет, а может это и вовсе часть экзамена. Поэтому решение тестов онлайн на заказ — это отличный способ перестраховаться. Обеспечьте себя хорошей оценкой и не рискуйте успеваемостью.

Как это работает

Помощь с тестами онлайн в ЮЗГУ проходит в несколько этапов:

  • Вы пишите нам и предоставляете доступ к личному кабинету, где проходит тестирование.
  • В согласованное время специалист нашей компании подключается к вам и решает задания.
  • Вы получаете положительную оценку.

И не нужно просиживать долгие часы за подготовкой, зубрить ответы и переживать. С нашей поддержкой сессия пройдет весело и без проблем.

Заказ тестов онлайн

Стоимость онлайн-тестов зависит от исполнителя. Многие компании завышают цену, делают ее неподъемной для студентов. Мы же устанавливаем адекватные тарифы, потому ценим своих клиентов. Кроме этого, мы гарантируем безопасность:

  • Мы никуда не пропадем в день теста. Если мы обещали, мы сделаем.
  • Мы предоставим в помощники специалистов, которые ориентируются в теме и без труда решат тест.
  • Мы справимся с любой дисциплиной и темой.

Сотрудничество с FastFine поможет вам без проблем решить проблемы с учебой. С нами экзамены и тесты покажутся вам несложными.

По вузам

  • КРСУ
  • ЛГУ
  • ЛГПУ
  • ЛГТУ
  • ЛЭГИ
  • Литературный институт имени А. М. Горького

Бесконтактный георадар: результаты, области применения и ограничения испытаний в США

NASA/ADS

Бесконтактный георадар: результаты, области применения и ограничения испытаний в США

  • Доусон, К.
  • ;
  • Лейн, Дж. В., младший
Аннотация

Хотя возможности и грузоподъемность систем малых незанятых летательных аппаратов (БАС) и их интеграция в науку о Земле развиваются, адаптация традиционных геофизических инструментов для развертывания БАС идет медленно. Геологическая служба США (USGS) собирала данные с помощью sUAS для выполнения нашей научной миссии в широком спектре геолого-геофизических исследований, в том числе уделяя приоритетное внимание разработке набора инструментов, обеспечивающих полностью бесконтактное измерение потока с помощью sUAS. Здесь мы представляем результаты продолжающихся USGS испытаний бесконтактного георадара (GPR) для гидрологических и гидрогеофизических исследований.

В 2017 году Геологическая служба США начала сотрудничество по разработке и тестированию бесконтактных систем георадара с воздушной связью, которые будут развернуты над целевой областью исследования. Полевые испытания на нескольких участках в Соединенных Штатах были сосредоточены на использовании бесконтактного георадара для оценки площади поперечного сечения пресноводных водотоков; затем эти измерения можно использовать в сочетании с данными о скорости потока для оценки расхода потока. Прототипы георадара, испытанные для батиметрического и приповерхностного картографирования, развертывались по канатной дороге, мосту и (или) БПЛА, как правило, на расстоянии от 1 до 20 метров над поверхностью земли или воды.

Тестирование показало, что бесконтактный георадар, развернутый с помощью sUAS и других методов, может успешно измерять батиметрию потока и отображать неглубокие слои при определенных геофизических условиях. Тестирование также показало, что необходимо решить определенные проблемы сбора и обработки данных, а также системные ограничения, прежде чем Геологическая служба США сможет внедрить широкое оперативное использование бесконтактного георадара. Геологическая служба США также разрабатывает инструменты поддержки принятия решений, чтобы информировать негеофизиков о понимании гидрологических и геологических условий, подходящих для бесконтактного развертывания георадара, и связанных с этим ограничений метода, основанных на опыте батиметрического картирования, приповерхностного изображения и глубины снежного покрова. опросы.


Публикация:

Тезисы осенней встречи AGU

Дата публикации:
Декабрь 2020
Биб-код:
2020AGUFMNH0280006D «/>
Ключевые слова:
  • 3394 Приборы и техника;
  • АТМОСФЕРНЫЕ ПРОЦЕССЫ;
  • 1920 Новые информационные технологии;
  • ИНФОРМАТИКА;
  • 4314 Математическое и компьютерное моделирование;
  • ПРИРОДНЫЕ ОПАСНОСТИ;
  • 4262 Системы наблюдения за океаном;
  • ОКЕАНОГРАФИЯ: ОБЩИЕ СВЕДЕНИЯ

Исследование коренной породы мышьяка и урана: часто задаваемые вопросы по мышьяку и урану

Часто задаваемые вопросы о мышьяке и уране в воде.

В 2009 году Геологическая служба США (USGS) исследовала присутствие мышьяка и урана в колодцах с питьевой водой в восточном Массачусетсе. Район исследования был выбран потому, что записи свидетельствуют о высоких концентрациях мышьяка в этом районе. Эти вопросы и ответы касаются некоторых распространенных проблем, связанных с естественным присутствием этих элементов в колодцах коренных пород.

Пропустить оглавление

Содержание

Вы пропустили раздел оглавления.

Для чего проводились испытания частных скважин и кто проводил это исследование?

Благодаря лучшему пониманию воздействия мышьяка и урана на здоровье человека Агентство по охране окружающей среды США (EPA) установило стандарты питьевой воды на содержание мышьяка и урана. Однако без тестирования может быть трудно определить, в каких частных колодцах может присутствовать мышьяк или уран в количествах, превышающих стандарты. Исследование Геологической службы США (USGS) было проведено для определения геологических характеристик, которые могут помочь определить области с более высокой вероятностью загрязнения мышьяком или ураном. Обратите внимание, что в исследовании изучались только уровни мышьяка и урана природного происхождения, а не уровни мышьяка и урана, возникающие в результате выбросов этих химических веществ в результате промышленных или других искусственных процессов.

В 2003 году Агентство по охране окружающей среды США установило стандарт питьевой воды 30 частей на миллиард (млрд) для урана в системе питьевого водоснабжения. В 2001 году EPA изменило стандарт содержания мышьяка в питьевой воде с 50 частей на миллиард до 10 частей на миллиард; этот стандарт вступил в силу в 2006 году.

Цель исследования Геологической службы США, начатого в 2008 году, состояла в том, чтобы определить, существует ли корреляция между присутствием мышьяка и урана в скважинах, пробуренных в коренных породах, и типом коренных пород. Эта информация будет использоваться для руководства будущей деятельностью по разведке общественного водоснабжения. Дополнительным преимуществом исследования является то, что эта информация может быть использована для определения того, находится ли частная скважина для водоснабжения в районе с более высокой или низкой вероятностью превышения любого из этих стандартов.

Исследование включало лабораторный анализ образцов частных колодцев, собранных 478 владельцами частных колодцев, расположенных в 116 населенных пунктах в центральном и северо-восточном штате Массачусетс. Район исследования был выбран на основе обзора существующих общедоступных записей о скважинах коренных пород, которые указывают на более высокие концентрации мышьяка в этом районе. Меньше известно о распределении урана в колодезной воде. Исследование включало анализ урана, чтобы лучше понять его возможную корреляцию с типами коренных пород в Массачусетсе. Геологическая служба США и Департамент охраны окружающей среды штата Массачусетс (MassDEP) совместно финансировали исследование частной скважины.

Чтобы устранить любые потенциальные опасения по поводу воздействия на здоровье человека, связанного с мышьяком и/или ураном, обнаруженным в пробах воды из частных колодцев, взятых в ходе этого проекта, Департамент общественного здравоохранения штата Массачусетс (MDPH) предложил некоторым участникам анализ мочи в качестве государственной услуги. исследования Геологической службы США. За дополнительной информацией обращайтесь в MDPH по телефону 617-624-5757 или 800-240-4266.

Должен ли я беспокоиться о мышьяке и уране в моем личном колодце с питьевой водой?

Для того, чтобы определить, следует ли вам беспокоиться о мышьяке и уране в вашей воде из частного колодца, вам сначала нужно знать, какие уровни присутствуют в воде из вашего колодца. MassDEP и MDPH рекомендуют всем домовладельцам проверять свои колодцы на наличие мышьяка и радионуклидов, таких как уран. Если результаты первоначального и последующего тестирования превышают общедоступные стандарты питьевой воды, вам следует рассмотреть возможность установки системы очистки воды для достижения этих стандартов.

Пока я жду результатов анализов, можно ли пить воду? Или купаться в нем?

Да. Основываясь на информации, доступной по результатам тестирования воды и сопутствующих исследований MDPH на мышьяк и уран в моче, вероятность того, что чей-либо колодец представляет опасность для здоровья, очень мала. Тем не менее, мы рекомендуем вам проверить вашу воду. Проживание в районе, где испытания USGS показывают, что грунтовые воды имеют больше шансов превысить стандарты питьевой воды по мышьяку и урану, не обязательно означает, что у вашего колодца будут проблемы. На самом деле мы ожидаем, что большинство частных скважин на этих участках не превышают нормы. Таким образом, вполне вероятно, но не обязательно, что ваша вода будет ниже стандартов. Единственный способ узнать наверняка, это хорошо проверить. Тем временем, если вы обеспокоены, вы можете использовать воду в бутылках, пока не получите результаты теста.

Государственные нормы содержания мышьяка и урана в питьевой воде составляют 10 микрограммов и 30 микрограммов на литр воды соответственно (или 10 частей на миллиард и 30 частей на миллиард соответственно). Стандарты питьевой воды устанавливаются Агентством по охране окружающей среды США для защиты здоровья населения. Они основаны на потреблении человеком 2 литров (приблизительно 2 кварты или 8 чашек воды в день) воды каждый день в течение всей жизни. Фактические риски для здоровья зависят от трех ключевых факторов: уровня мышьяка и урана в питьевой воде; количество воды, фактически потребленной из вашей частной скважины; и сколько лет расходуется вода. Риски снижаются, если вы выпиваете менее 2 литров в день или потребляете воду меньше, чем всю жизнь.

Почему мышьяк и уран могут присутствовать в колодезной воде?

Некоторые районы Новой Англии могут содержать более высокие уровни мышьяка и урана в коренных породах, чем другие районы региона. Мышьяк можно найти в двух различных формах: органический мышьяк и неорганический мышьяк. Неорганические формы мышьяка обычно обнаруживаются в подземных водах и представляют большую опасность для здоровья, чем органические формы мышьяка (которые присутствуют в некоторых пищевых продуктах, например в рыбе), которые считаются менее токсичными. В результате естественных процессов мышьяк и уран могут покинуть горную породу и попасть в грунтовые воды в коренной породе. Если скальный колодец установлен в месте, которое перехватывает подземные воды, содержащие относительно высокие концентрации мышьяка и/или урана, то содержание мышьяка и/или урана, извлекаемого из колодца, может превышать общедоступные стандарты питьевой воды.

Каковы нормы содержания мышьяка и урана в питьевой воде?

Государственный и федеральный стандарт питьевой воды (PWS) для мышьяка составляет 0,010 миллиграмма на литр (мг/л) (или 10 частей на миллиард частей на миллиард). Стандарт питьевой воды для урана составляет 0,030 мг/л (или 30 частей на миллиард). Стандарты частных колодцев определяются местным советом здравоохранения (BOH) и обычно совпадают со стандартами штата и федеральными стандартами PWS.

Показало ли исследование корреляцию между геологией и концентрацией мышьяка в колодцах коренных пород?

Да. 478 проб воды из колодцев, отобранных для исследования, были распределены среди многих различных типов геологических единиц коренных пород, нанесенных на карту в районе исследования. Результаты показали корреляцию между вероятностью превышения общественного стандарта питьевой воды по мышьяку и типом коренных пород. Исследование также показало сильную корреляцию между концентрациями мышьяка и близостью скважины к зоне разлома Клинтон-Ньюбери. Зона разлома Клинтон-Ньюбери простирается от восточной оконечности границы Массачусетса и Коннектикута на юге до долины реки Мерримак на северо-востоке.

Геологическая служба США спрогнозировала расчетное количество скважин в районе исследования, которое, как ожидается, превысит стандарт мышьяка, путем умножения вероятностей, полученных на основе результатов выборки, на расчетное количество скважин в этом районе.

В результате этого метода был сделан прогноз о том, что примерно в 5700 жилых скальных колодцах из 90 000 в исследуемой области концентрация мышьяка может превышать общедоступный стандарт питьевой воды. По оценкам Геологической службы США, примерно 3800 из этих 5700 скважин не очищаются от мышьяка.

Показало ли исследование корреляцию между геологией и концентрацией урана в скважинах коренных пород?

Да. Результаты 478 проб, отобранных для исследования, показали корреляцию между вероятностью превышения общедоступного стандарта питьевой воды по урану и типом коренных пород. В целом скважины, пройденные в гранитных (гранитного типа) коренных породах, имели наибольшую вероятность превышения норматива.

Геологическая служба США спрогнозировала оценочное количество скважин в районе исследования, которое, как ожидается, превысит урановый стандарт, умножив вероятности, полученные на основе результатов выборки для каждой отдельной единицы коренных пород, на оценочное количество скважин в каждой единице. Этот метод привел к прогнозу, что примерно 3300 жилых скальных колодцев из

человек в районе исследования имеют концентрацию урана, превышающую общедоступный стандарт питьевой воды. По оценкам Геологической службы США, примерно 3000 из этих 3300 скважин не очищаются от урана.

Какой процент образцов превысил общедоступные стандарты питьевой воды по содержанию мышьяка и урана?

Результаты 478 проб из частных колодцев из 116 городов, которые были собраны и проанализированы для исследования Геологической службы США, следующие:

  • примерно 13% проб, взятых из случайно выбранных колодцев, превышали стандарт содержания мышьяка в питьевой воде в 10 частей на миллиард
  • менее чем в 3% всех собранных проб содержание урана в питьевой воде превышало 30 частей на миллиард.

Каковы потенциальные последствия для здоровья, связанные с высоким содержанием мышьяка в питьевой воде?

Кратковременное воздействие питьевой воды с более высоким содержанием неорганического мышьяка может привести к тошноте, рвоте, диарее, сердечно-сосудистым заболеваниям и последствиям для головного мозга (например, энцефалопатии), а также к снижению выработки красных и белых кровяных телец и нарушению функции нервной системы. Длительное употребление питьевой воды, содержащей неорганический мышьяк, может привести к ряду изменений кожи (например, пятнам потемневшей кожи, небольшим «мозолям» или «бородавкам» на ладонях, подошвах ног и туловище) и повреждению нервной системы ( то есть периферическая невропатия). Несколько исследований также показали, что длительное воздействие неорганического мышьяка может увеличить риск развития некоторых видов рака, включая рак легких, кожи, мочевого пузыря, печени, почек и простаты.

Дополнительные ресурсы

Каковы потенциальные последствия для здоровья, связанные с высоким содержанием урана в питьевой воде?

Основным последствием воздействия повышенных уровней урана на здоровье является повреждение почек. Тем не менее, большинство эффектов, наблюдаемых у людей, были вызваны сильными кратковременными воздействиями, а некоторые профессиональные исследования показали обратное воздействие на почки после прекращения воздействия. Хроническое воздействие также может привести к почечным последствиям. Воздействие на здоровье природного урана в питьевой воде связано с химической токсичностью урана. Агентство по охране окружающей среды США установило свой стандарт питьевой воды для урана в первую очередь из-за его химической токсичности, но в качестве меры предосторожности также рассмотрело возможность урана вызывать рак после продолжительного воздействия питьевой воды, исходя из предположения, что любой компонент, обладающий радиоактивными свойствами, может вызвать рак.

Дополнительные ресурсы

Существуют ли другие радионуклиды, помимо урана, о которых мне следует беспокоиться?

В исследовании Геологической службы США изучалось присутствие мышьяка и урана только в колодезной воде. Другие встречающиеся в природе радионуклиды, такие как газ радон и радий, обычно встречаются в тех же типах коренных пород, что и уран. Таким образом, несмотря на отсутствие данных о вероятности для газообразного радона и радия, районы с повышенной вероятностью повышенных концентраций урана, как правило, будут иметь повышенную вероятность повышенных концентраций радона и радия. Дополнительную информацию о стандартах общественной питьевой воды по радону и радию, воздействии на здоровье и вариантах лечения см. в разделе «Вопросы и ответы по радионуклидам».

Подвергается ли общественное водоснабжение риску содержания мышьяка или урана в воде?

Нет. Колодцы общественного водоснабжения, обслуживающие жилые дома, проверяются на мышьяк и уран. Любая общественная система водоснабжения, которая не может в обычном порядке соответствовать общественным стандартам питьевой воды, должна обеспечивать очистку. Они также обязаны сообщать о любых результатах качества воды, которые превышают общедоступные стандарты питьевой воды, в своем ежегодном отчете о доверии потребителей, который предоставляется их клиентам.

Где можно проверить воду из колодца на мышьяк, уран и другие радионуклиды?

Доступный для поиска список лабораторий, сертифицированных MassDEP, см. в электронной таблице лабораторий, сертифицированных MassDEP, которые тестируют соединения мышьяка, ниже.

Дополнительные ресурсы

Где я могу проверить пробу воды из колодца на концентрацию мышьяка 3 и мышьяка 5?

Неорганический мышьяк встречается в природе в виде мышьяка 3 и мышьяка 5. Мышьяк 3 значительно труднее удалить из воды, чем мышьяк 5. Поэтому, если общая концентрация мышьяка в вашей колодезной воде превышает государственный стандарт питьевой воды, вам следует подумать о том, чтобы взять образец проверены на содержание мышьяка, чтобы выбрать вариант очистки, который удалит достаточное количество мышьяка для получения готовой воды, соответствующей стандарту. См. список лабораторий, сертифицированных MassDEP, которые проверяют мышьяк и уран (ссылка на предыдущий раздел).

Включен ли мой город в изучаемую территорию?

См. список карт города, ссылка на который приведена ниже, чтобы определить, доступна ли информация о вероятности мышьяка или урана для вашего города. Если ваш город указан в списке, вы также найдете ссылки на PDF-карты вашего города для зон вероятности мышьяка и урана.

Дополнительные ресурсы

Где я могу найти конкретную информацию о вероятности наличия мышьяка или урана в колодезной воде?

Вы можете использовать Инструмент поиска скважин для получения оценок вероятности содержания мышьяка и урана, чтобы ввести адрес улицы и название города, чтобы получить информацию о вероятности того, что вода из вашего колодца может не соответствовать общедоступным стандартам питьевой воды по содержанию мышьяка и урана. Кроме того, доступны карты городов, на которых показаны зоны вероятности мышьяка и урана.

Как инструмент для определения местоположения скважин, так и соответствующие карты городов для зон вероятности залегания урана определяют участки коренных пород, обозначенные как гранит или пегматит, которые расположены за пределами области, охваченной исследованием Геологической службы США. Хотя статистические данные о вероятности появления урана для этих конкретных гранитных или пегматитовых единиц отсутствуют, колодцы коренных пород в этих типах горных пород обычно имеют повышенную вероятность содержания природного радона, радия и урана в концентрациях, превышающих пределы питьевой воды.

Дополнительные ресурсы

Какую обработку нужно установить для удаления мышьяка или урана и сколько это будет стоить?

Мышьяк:
Обычно в воде присутствуют два варианта или разновидности неорганического мышьяка: «мышьяк 3» и «мышьяк 5». Это важно, потому что «мышьяк 3» очень трудно удалить из воды, и его необходимо заменить или окислить до «мышьяка 5», прежде чем его можно будет удалить. Для удаления мышьяка чаще всего используются анионообменные фильтры, фильтры обратного осмоса, активированный оксид алюминия и другие типы фильтров с адсорбционными средами. В зависимости от того, обрабатываете ли вы только воду, поступающую в кухонную раковину, или всю воду, поступающую в ваш дом, средняя стоимость установки систем удаления мышьяка составляет от 1200 до 3000 долларов (не включая стоимость предварительной обработки для преобразования). мышьяк 3» до «мышьяк 5», что может потребоваться, а может и не потребоваться).

Уран:
Обратный осмос и анионообмен чаще всего используются для удаления урана. В зависимости от того, обрабатываете ли вы только воду, поступающую в кухонную раковину, или всю воду, поступающую в ваш дом, средняя стоимость установки систем удаления урана составляет от 1600 до 2500 долларов.

Нужно ли очищать всю воду или только воду, поступающую в кран кухонной раковины?

Для тех колодцев, которые превышают стандарты питьевой воды по мышьяку и урану в Массачусетсе, концентрации, как правило, опасны только для приема внутрь. Таким образом, обработка только той воды, которая используется для питья и приготовления пищи, — это, как правило, все, что требуется для защиты здоровья вашей семьи. Если концентрация мышьяка в вашем организме превышает 0,5 миллиграмма на литр (мг/л) (или 500 частей на миллиард), что крайне редко встречается в Массачусетсе, вам следует подумать о лечении всего дома.

Могу ли я сбрасывать отработанную воду из моей системы очистки обратного осмоса в мою септическую систему согласно Разделу 5?

Ответ зависит от типа системы очистки обратного осмоса, концентрации мышьяка и урана в сырой воде и использования имущества. Сбросы воды обратного осмоса на месте из точек использования (POU) (т. Е. Очистка только воды, поступающей в кухонную раковину). Устройства для очистки питьевой воды в жилых домах с одной-четырьмя квартирами. 5 септическая система, но только при соблюдении следующих предельных значений концентрации мышьяка и урана в сырой (неочищенной) воде:

  • общая концентрация мышьяка составляет менее 1 миллиграмма на литр (мг/л) или 1 часть на миллион (ppm) в сырой (неочищенной) воде; и
  • свяжитесь с Джо Черрутти или Стивом Халлемом (контактная информация указана в конце этой страницы) для получения информации о допустимых концентрациях урана в неочищенной воде.

Устройства POU, которые используются на объектах, состоящих из более чем четырех жилых единиц, или используются для целей, отличных от жилых, не могут сбрасываться в септическую систему Раздела 5. Системы обработки питьевой воды в точке входа (POE) (т. Е. Обработка всей воды, поступающей в дом) не могут сбрасываться в септическую систему Раздела 5. Сточные воды из системы очистки питьевой воды, которые нельзя сбрасывать в септическую систему Раздела 5, должны сбрасываться либо в муниципальную систему очистки сточных вод, либо в сухой колодец. Для сброса в сухой колодец требуется подача заявки на регистрацию MassDEP Underground Injection Control (UIC), за исключением объектов, которые используются только для проживания одной семьи.

Могу ли я сбрасывать сточные воды, содержащие мышьяк или уран, из моей системы очистки питьевой воды в сухой колодец?

Для всех сбросов из систем очистки воды требуется регистрация UIC, одобренная MassDEP, за исключением свойств, которые используются только для проживания одной семьи. См. обсуждение предыдущего вопроса об исключении для некоторых сбросов обратного осмоса (RO) для систем POU. Чтобы утвердить заявку на регистрацию UIC для сбросов системы очистки воды, MassDEP оценивает, может ли сброс привести к нарушению стандарта питьевой воды в принимающих грунтовых водах. Из-за эффектов разбавления MassDEP может одобрить сбросы в сухой колодец, которые превышают общедоступные стандарты питьевой воды, но не существует установленного предела, и каждое заявление рассматривается в каждом конкретном случае. MassDEP не может одобрить любые сбросы воды, в которых содержание мышьяка превышает федеральный порог содержания опасных отходов (характеристика токсичности) в 5 миллиграммов на литр (мг/л) или 5 частей на миллион (ppm), или в которых содержание урана превышает 0,600 мг/л или 600 частей на миллиард (ppb). Свяжитесь с Джо Черутти или Стивом Халлемом (контактная информация указана в конце этой страницы) для получения информации о допустимых концентрациях урана в сбросе в скважину UIC.

Есть ли средства для домовладельцев с низким доходом для установки системы очистки?

Финансирование Программы ссуд и грантов на благоустройство и ремонт домов Министерства сельского хозяйства США (USDA) (раздел 504) может быть доступно домовладельцам с очень низким доходом в соответствующих сообществах. Эти ссуды или гранты могут быть использованы домовладельцами для проведения общего ремонта с целью улучшения или модернизации, защиты от погодных условий и/или устранения нарушений кодекса. безопасное и санитарное состояние жилого помещения. Эта программа доступна в сельских населенных пунктах и ​​малых городах с населением до 10 000 человек. Некоторые сообщества с населением от 10 000 до 20 000 человек могут иметь право на участие. Домовладельцам следует посетить веб-сайт USDA Rural Development, чтобы узнать о требованиях и о том, как подать заявку, на веб-сайте USDA Rural Development: https://www.rd.usda.gov/ma.

Кто регулирует частные колодцы?

В соответствии с Общим законом штата Массачусетс (MGL Ch.111 s.122) местные советы по здравоохранению (BOH) имеют основную юрисдикцию в отношении регулирования частных колодцев. Местный BOH уполномочен принять Положение о частных колодцах, которое устанавливает критерии для размещения частных колодцев, строительства, качества и количества воды.

Что делает государство с этой проблемой?

MassDEP предоставил большую часть финансирования исследования Геологической службы США, в ходе которого были определены зоны высокой и низкой вероятности, а MDPH предоставил финансирование для картографирования частных скважин, которое использовалось в исследовании Геологической службы США, а MDPH финансировал и проводил анализ мочи, который проводился одновременно. с исследованием Геологической службы США.

MassDEP разработал Инструмент определения местоположения скважин для получения оценок вероятности содержания мышьяка и урана, чтобы предоставить владельцам частных скважин простой способ получить дополнительную информацию о вероятности того, что вода из их коренных скважин превышает общедоступные стандарты питьевой воды для мышьяка и урана.

MassDEP, MDPH и Геологическая служба США предоставили и будут продолжать поддерживать связь с местными советами здравоохранения, информируя их о результатах исследования и о том, куда они могут направить владельцев частных скважин для получения дополнительной информации.

MassDEP разработал карты в формате PDF зон вероятности содержания мышьяка и урана в масштабе города и сделал их доступными для широкой общественности и местных советов здравоохранения для городов, для которых существует информация Геологической службы США о вероятности того, что мышьяк или уран превышают общедоступные стандарты питьевой воды. , или для которых существуют типы коренных пород, которые, как известно, имеют повышенную вероятность содержания урана и других радионуклидов в концентрациях, превышающих общедоступные стандарты питьевой воды.

MDPH предоставляет информацию о влиянии мышьяка и урана на здоровье любому владельцу частной скважины, который ищет информацию.

MassDEP предоставляет информацию о доступных технологиях очистки любому частному владельцу скважины, который ищет информацию.

Обратите внимание, что MassDEP не регулирует частные колодцы с питьевой водой.

Может ли мой питомец пить воду, содержание мышьяка/урана в которой превышает стандарт питьевой воды?

Различные виды домашних животных могут иметь различную чувствительность к мышьяку и урану в питьевой воде. Тем не менее, рекомендуется давать домашним животным питьевую воду, соответствующую тем же стандартам/рекомендациям, которые установлены для людей.

К кому я могу обратиться за дополнительной информацией или вопросами?

Если у вас есть дополнительные вопросы, на которые нет ответов в этом документе и связанных ссылках, вы можете получить дополнительную информацию по следующему адресу:

По вопросам, касающимся отчета Геологической службы США, обращайтесь в Центр водных исследований Геологической службы США: https://www.usgs. gov/centers/new-england-water/connect

По вопросам, касающимся воздействия мышьяка и урана на здоровье, обращайтесь в MDPH по телефону 617-624-5757 или 800-240-4266.

По вопросам, касающимся лечения мышьяком, обращайтесь к Joe Cerutti, MassDEP, по телефону 617-292-5859, электронная почта: [email protected].

По вопросам лучевой терапии обращайтесь к Steve Hallem, MassDEP, по телефону 617-292-5681, электронная почта: stephen. [email protected].

Помогите нам улучшить Mass.gov своими отзывами

Вы нашли то, что искали на этой веб-странице? Если у вас есть предложения по сайту, сообщите нам. Как мы можем улучшить страницу? *

Пожалуйста, не указывайте личную или контактную информацию.

Отзывы будут использованы только для улучшения сайта. Если вам нужна помощь, посетите страницу «Контакты и сервисный центр MassDEP». Пожалуйста, ограничьте ввод до 500 символов.

Пожалуйста, удалите любую контактную информацию или личные данные из вашего отзыва.

Если вам нужна помощь, посетите страницу контактов и сервисного центра MassDEP.

Пожалуйста, сообщите нам, как мы можем улучшить эту страницу.

Пожалуйста, удалите любую контактную информацию или личные данные из вашего отзыва.

Если вам нужна помощь, посетите страницу контактов и сервисного центра MassDEP.

Формула суммы пятой степени: Сумма пятой степени | Формулы с примерами

Элементарная алгебра

Элементарная алгебра
  

С.Т. Завало. Элементарная алгебра. Изд-во «Просвещение», М., 1964 г.

В основу этой книги положен курс лекций по элементарной алгебре, читавшийся мною на протяжении ряда лет в Черкасском государственном педагогическом институте.

Первая глава книги — вступительная. В ней сжато изложены сведения о некоторых математических понятиях, с которыми читателю придется встретиться в последующих главах. В главах II—X изложен учебный материал по элементарной алгебре, предусмотренный программой специального курса элементарной математики для студентов-математиков педагогических институтов.

Книга рассчитана на студентов-математиков педагогических институтов. Она может быть также пособием для учителей математики средней школы.



Оглавление

Глава I. ПРЕДВАРИТЕЛЬНЫЕ ЗАМЕЧАНИЯ
§ 2. Понятия кольца и поля
§ 3. Упорядоченные поля
§ 4. Понятие функции и аналитического выражения
§ 5. Элементарные функции и их классификация
§ 6. Метод математической индукции
Глава II. ОБЩИЕ СВЕДЕНИЯ ОБ УРАВНЕНИЯХ
§ 1. Понятие уравнения. Решения уравнения
§ 2. Классификация уравнений, изучаемых в элементарной математике
§ 3. Равносильность уравнений
§ 4. Преобразование уравнений при их решении
Глава III. ЭЛЕМЕНТАРНЫЕ МЕТОДЫ РЕШЕНИЯ АЛГЕБРАИЧЕСКИХ И ДРОБНО-РАЦИОНАЛЬНЫХ УРАВНЕНИЙ С ОДНИМ НЕИЗВЕСТНЫМ
§ 1. Алгебраические уравнения n-й степени с одним неизвестным
§ 2. Корни квадратного трехчлена
§ 3. Исследование квадратного трехчлена над полем действительных чисел
§ 4. Двучленные уравнения
§ 5. Трехчленные уравнения, приводящиеся к квадратным
§ 6. Симметрические уравнения
§ 7. Алгебраическое уравнение n-й степени с рациональными коэффициентами
§ 8. Частные приемы решения уравнений высших степеней
§ 9. Дробно-рациональные уравнения
Глава IV. ТЕОРИЯ СОЕДИНЕНИЙ
§ 2. Перестановки
§ 3. Сочетания
§ 4. Размещения
§ 5. Перестановки с повторениями
§ 6. Сочетания с повторениями
§ 7. Размещения с повторениями
Глава V. БИНОМ НЬЮТОНА И ПОЛИНОМИАЛЬНАЯ ТЕОРЕМА
§ 1. Бином Ньютона
§ 2. Биномиальные коэффициенты и их основные свойства
§ 3. Треугольник Паскаля
§ 4. Полиномиальная теорема
§ 5. Вычисление сумм степеней первых n чисел натурального ряда
Глава VI. МНОГОЧЛЕНЫ ОТ НЕСКОЛЬКИХ ПЕРЕМЕННЫХ
§ 1. Многочлен от нескольких переменных и его каноническая форма
§ 2. Однородный многочлен от n переменных и число его членов
§ 3. Число членов в каноническом представлении многочлена от n переменных
§ 4. Тождественность двух многочленов
§ 5. Тождественные преобразования многочленов. Тождество Лагранжа
§ 6. Применение метода неопределенных коэффициентов при выполнении алгебраических действий над многочленами
Глава VII. СИСТЕМЫ УРАВНЕНИЙ С НЕСКОЛЬКИМИ НЕИЗВЕСТНЫМИ
§ 1. Понятие системы уравнений
§ 2. Равносильность систем уравнений
§ 3. Уравнения и системы уравнений, являющиеся следствием данной системы уравнений
§ 4. Основные элементарные методы решения систем уравнений
§ 5. Решение нелинейных систем алгебраических уравнений элементарными методами
1. Решение системы двух уравнений с двумя неизвестными, из которых одно—второй степени, а другое — первой.
2. Решение системы двух уравнений второй степени с двумя неизвестными, которые не имеют членов первой степени.
3. Решение системы двух уравнений второй степени с двумя неизвестными в общем виде.
4. Решение системы двух однородных уравнений с двумя неизвестными.
5. Решение системы двух уравнений с двумя неизвестными, одно из которых однородное, а второе не однородное.
7. Решение нелинейной системы алгебраических уравнений, в состав которой входят линейные уравнения.
8. Решение нелинейной системы алгебраических уравнений, левая часть одного из которых представляется в виде произведения.
§ 6. Графическое решение нелинейных систем алгебраических уравнений с двумя неизвестными
Глава VIII. НЕРАВЕНСТВА
§ 1. Основные свойства неравенств
§ 2. Тождественные неравенства
§ 3. Применение неравенств для определения наибольших и наименьших значений
§ 4. Решение неравенств
§ 5. Решение алгебраических неравенств с одним неизвестным первой и второй степени
§ 6. Решение систем алгебраических неравенств первой степени с двумя неизвестными
§ 7. Применение неравенств для задания числовых и точечных множеств
Глава IX. ИРРАЦИОНАЛЬНЫЕ УРАВНЕНИЯ НАД ПОЛЕМ ДЕЙСТВИТЕЛЬНЫХ ЧИСЕЛ
§ 1. Корни с натуральными показателями в поле действительных чисел
§ 2. Тождественные преобразования иррациональных выражений в поле действительных чисел
§ 3. Решение иррациональных уравнений и систем, в состав которых входят иррациональные уравнения, в поле действительных чисел
Глава X. ПОКАЗАТЕЛЬНЫЕ И ЛОГАРИФМИЧЕСКИЕ УРАВНЕНИЯ В ПОЛЕ ДЕЙСТВИТЕЛЬНЫХ ЧИСЕЛ
§ 1. Теоретические основы решения показательных и логарифмических уравнений
§ 2. Решение показательных уравнений с одним неизвестным
§ 3. Решение логарифмических уравнений с одним неизвестным
§ 4. Решение трансцендентных уравнений, приводящихся к показательным и логарифмическим уравнениям
§ 5. Решение некоторых трансцендентных систем уравнений
§ 6. Графические способы решения трансцендентных уравнений и систем
ЛИТЕРАТУРА

Формулы сокращенного умножения

Для упрощения выражений, разложения многочленов на множители, приведения многочленов к стандартному виду используются формулы сокращенного умножения. Формулы сокращенного умножения нужно знать наизусть.

Пусть а, b   R. Тогда:

1. Квадрат суммы двух выражений равен квадрату первого выражения плюс удвоенное произведение первого выражения на второе плюс квадрат второго выражения.

(a + b)2 = a2 + 2ab + b2

2. Квадрат разности двух выражений равен квадрату первого выражения минус удвоенное произведение первого выражения на второе плюс квадрат второго выражения.

(a — b)2 = a2 — 2ab + b2

3. Разность квадратов двух выражений равна произведению разности этих выражений и их суммы.

a2 — b2 = (a -b) (a+b)

4. Куб суммы двух выражений равен кубу первого выражения плюс утроенное произведение квадрата первого выражения на второе плюс утроенное произведение первого выражения на квадрат второго плюс куб второго выражения.

(a + b)3 = a3 + 3a2b + 3ab2 + b3

5. Куб разности двух выражений равен кубу первого выражения минус утроенное произведение квадрата первого выражения на второе плюс утроенное произведение первого выражения на квадрат второго минус куб второго выражения.

(a — b)3 = a3 — 3a2b + 3ab2 — b3

6. Сумма кубов двух выражений равна произведению суммы первого и второго выражения на неполный квадрат разности этих выражений.

a3 + b3 = (a + b) (a2 — ab + b2)

7. Разность кубов двух выражений равна произведению разности первого и второго выражения на неполный квадрат суммы этих выражений.

a3 — b3 = (a — b) (a2 + ab + b2)

8. Разность чисел в четвертой степени

(a — b)4 = a4 — 4a3b + 6a2b2 — 4ab3 + b4

9. Сумма чисел в четвертой степени

(a + b)4 = a4 + 4a3b + 6a2b2 + 4ab3 + b4

10. Разность чисел в пятой степени

(a — b)5 = a5 — 5a4b + 10a3b2 — 10a2b3 + 5ab4 — b5

11. Сумма чисел в пятой степени

(a + b)5 = a5 + 5a4b + 10a3b2 + 10a2b3 + 5ab4 + b5

12. Квадрат трехчлена

(a + b + c)2 = a2 + b2 + c2 + 2ab + 2ac + 2bc

13. Квадрат линейной формы

(a + b + c + … + u + v)2 = a2 + b2 + c2 + … + u2 + v2 + 2(ab + ac + … + au + av + bc + … + bu + bv + … + uv)

14. Куб трехчлена

(a + b + c)3 = a3 + b3 + c3 + 3a2b + 3ab2 + 3a2c + 3ac2 + 3b2c + 3bc2 + 6abc

Степени и степени

Обновлено 26 июня 2019 г. | Инфопожалуйста Персонал

в степени — это произведение , умножающее число само на себя.


Обычно степень представлена ​​ основанием, и показателем степени. Базовое число сообщает , какое число умножается. Показатель степени , небольшое число, написанное выше и справа от основного числа, говорит о сколько раз умножается основное число.

Например, «6 в 5-й степени» можно записать как «6 5 ». Здесь базовое число равно 6, а показатель степени равен 5. Это означает, что 6 умножается само на себя 5 раз: 6 х 6 х 6 х 6 х 6

6 х 6 х 6 х 6 х 6 = 7 776 или 6 5 = 7,776

базовый номер 2-я степень 3-я степень 4-я степень 5-я степень 90 014
1 1 1 1 1
2 4 8 16 32
3 9 27 81 243
4 16 64 256 1024
5 25 125 625 3 125
6 36 216 1 296 7 776
7 49 343 2 ,401 16 807
8 64 512 4 096 32 768
9 81 729 6 561 59 049
10 100 1 000 9001 4 10 000 100 000
11 121 1 331 14 641 161 051
12 144 1 728 20 736 248 832


9001 3 Таблица умножения
Факториалы Числа и формулы
Факториалы Числа и формулы Таблица умножения

Источники +

Наши общие источники

Видео с вопросами: Формулы множественных углов из формулы Эйлера

Стенограмма видео

Используйте формулу Эйлера, чтобы вывести формулу для cos 5 𝜃 и sin 5 𝜃 через sin 𝜃 и cos 𝜃.

Напомним, формула Эйлера говорит, что 𝑒 в степени 𝑖𝜃 равно cos 𝜃 плюс 𝑖 sin 𝜃. Итак, как мы применим это, чтобы вывести формулу для cos, равного пяти 𝜃, и sin, равного пяти 𝜃? Что ж, мы собираемся начать с возведения обеих частей этой формулы в пятую степень. Теперь мы можем сказать, что 𝑒 в 𝑖𝜃 все в пятой степени равно 𝑒 в пяти 𝑖𝜃. Но тогда, конечно, мы могли бы использовать формулу Эйлера, чтобы переписать это как cos пять 𝜃 плюс 𝑖 sin of Five 𝜃. Итак, у нас есть уравнение потому что пять 𝜃 плюс 𝑖 грех пяти 𝜃 равно косинусу 𝜃 плюс 𝑖 грех 𝜃 все в пятой степени. И теперь мы можем использовать биномиальную теорему, чтобы распределить эти скобки.

Это говорит о том, что 𝑎 плюс 𝑏 в 𝑛-й степени является суммой от 𝑘 равной нулю до 𝑛 из 𝑛 выберите 𝑘, умноженное на 𝑎 в степени 𝑛 минус 𝑘, умноженное на 𝑏 в 𝑘-й степени. Когда 𝑛 равно пяти, мы имеем 𝑎 плюс 𝑏 в пятой степени равно 𝑎 в пятой степени плюс пять выбрать один 𝑎 в четвертой степени 𝑏 плюс пять выбрать два раза 𝑎 в кубе умножить на 𝑏 в квадрате и так далее. На самом деле, пять выбирают один и пять выбирают четыре равно пяти, а пять выбирают два и пять выбирают три равны 10. Итак, у нас есть следующая формула, которая поможет нам распределить скобки cos 𝜃 плюс 𝑖 sin 𝜃 в пятой степени. Первый член — это просто cos 𝜃 в пятой степени, а второй — пять cos 𝜃 в четвертой степени, умноженные на 𝑖 sin 𝜃.

Но на самом деле, давайте переместим 𝑖 вперед и запишем это как пять 𝑖 cos 𝜃 в четвертой степени sin 𝜃. Тогда третий член равен 10 кос в кубе 𝜃 умножить на 𝑖 грех 𝜃 в квадрате, что можно записать как 10 кос в кубе 𝜃 умножить на 𝑖 в квадрате умножить на квадрат греха 𝜃. Но мы знаем, что 𝑖 в квадрате равен минус единице. Таким образом, мы можем переписать это далее как отрицательные 10 cos в кубе 𝜃 sin в квадрате 𝜃. Тогда наш четвертый член равен 10 кос в квадрате 𝜃 умножить на 𝑖 грех 𝜃 в кубе. И если мы считаем 𝑖 в кубе равным 𝑖 умножить на 𝑖 в квадрате, мы увидим, что все это выражение можно переписать как минус 10 𝑖 умножить на кос в квадрате 𝜃 умножить на грех в кубе 𝜃. Тогда у нас есть пять cos 𝜃 умноженных на 𝑖 sin 𝜃 в четвертой степени. А так как 𝑖 в четвертой степени равно 𝑖 в квадрате, это отрицательная единица в квадрате, то есть просто единица. И этот термин становится пятью cos 𝜃 sin 𝜃 в четвертой степени.

Наш последний термин равен 𝑖 sin 𝜃 в пятой степени. 𝑖 в пятой степени равно 𝑖 в четвертой степени, умноженное на 𝑖. Итак, мы имеем просто 𝑖 sin 𝜃 в пятой степени. Итак, наше уравнение теперь представляет собой кос пять 𝜃 плюс 𝑖 грех пять 𝜃 равно кос 𝜃 в пятой степени плюс пять 𝑖 cos 𝜃 в четвертой степени умножить на грех 𝜃 минус 10 кос в кубе 𝜃 грех в квадрате 𝜃 и так далее. И теперь мы готовы вывести формулу для пяти 𝜃. Мы делаем это, приравнивая или сравнивая действительные части с каждой стороны нашего уравнения. в левой части это просто кос пять 𝜃, тогда как в правой части у нас есть кос 𝜃 в пятой степени минус 10 куб куб 𝜃 квадрат греха 𝜃 плюс пять кос 𝜃 грех 𝜃 в четвертой степени.

Поскольку мы знаем, что действительные компоненты в каждой части нашего уравнения должны быть равны, мы создаем следующее уравнение. И мы могли бы оставить это так. Но мы могли бы также вспомнить, что квадрат греха 𝜃 плюс квадрат квадрата 𝜃 равен единице. А затем, написав, что грех в квадрате 𝜃 равен единице минус косинус в квадрате 𝜃, мы находим косинус пять 𝜃 равно косинусу 𝜃 в пятой степени минус 10 косинус в кубе 𝜃 умноженный на один минус косинус в квадрате 𝜃 плюс пять косинусов 𝜃 умноженный на один минус косинус в квадрате 𝜃 в квадрате .

Наконец, мы распределяем скобки. И мы находим, что правая часть этого уравнения становится 16 cos 𝜃 в пятой степени минус 20 cos в кубе 𝜃 плюс пять cos 𝜃. Итак, мы получили нашу формулу для пяти 𝜃. Фактически, мы повторяем этот процесс для пяти 𝜃. На этот раз, однако, мы собираемся сравнить воображаемые части. В левой части у нас есть грех пять 𝜃, тогда как в правой части у нас есть пять кос 𝜃 в четвертой степени грех 𝜃 минус 10 кос в квадрате 𝜃 грех в кубе 𝜃 плюс грех 𝜃 в пятой степени. Таким образом, наше уравнение для греха пять 𝜃 становится грехом пять 𝜃 равно пяти, потому что 𝜃 в четвертой степени, грех 𝜃 минус 10, потому что в квадрате 𝜃 грех в кубе 𝜃 плюс грех 𝜃 в пятой степени.

Самоучка математика 2 класс таблица умножения: Сайт Никитиной Натальи Борисовны — Тренажёр

Сайт Никитиной Натальи Борисовны — Тренажёр

ТРЕНАЖЁР 

Учение может быть интересным! Предлагаю Вам и вашему ребёнку ссылки на тесты, игры, программы….
  1.  1001 ВИКТОРИНА 
  2. УЧИМСЯ ИГРАЯ (сайт) 
  3. УЧИ.РУ (сайт) 
  4. ТЕСТЫ(русский) 
  5. БИБЛИОЗНАЙКА 
  6. ВИДЕОРЕПЕТИТОР 
  7. УРОКИ школьной программы 
  8. ДОМАШНЯЯ ШКОЛА 
  9. ИГРАЕМ САМИ 
  10. ОБРАЗОВАТЕЛЬНЫЕ ТЕСТЫ 
  11. ОТЛИЧНИК 
  12. РАСТУ.РУ 
  13. САМОУЧКА 
  14. ТРЕНАЖЁР (игры) 
  15. ТАБЛИЦА СЛОЖЕНИЯ и УМНОЖЕНИЯ 
  16. МУЗЕЙ русского слова
  17. Интерактивная игра «В стране ребусов»
  18. Интеллектуальный марафон (2 класс)
  19. Интерактивная викторина «Космическая»
  20. Презентация«Занимательная математика»
  21. Викторина«Фразеологизмы. Доскажи словечко»;
  22. Викторина «Знаешь ли ты фразеологизмы?»; 3-4 классы

ИНТЕРАКТИВНЫЕ ПЛАКАТЫ:

  1. Пишем буквы правильно
  2. Пишем цифры правильно
  3. Живое — неживое
  4. Зимующие — перелётные

МАТЕРИАЛ К УРОКАМ:

  1. Простая наука
  2. Наука для детей
  3. ЛАБУКАП
  4. СЕРВИС ДЛЯ СОЗДАНИЯ КНИЖНЫХ ОБЛОЖЕК
  5. Золотое кольцо России (Иваново) 
  6. Интеллектуальный марафон (2 класс)

ТЕСТЫ  

 

ТАБЛИЦА СЛОЖЕНИЯ:

  1. Состав числа 10, 9, 8 / отработка вычислительных навыков в пределах 10/.   
  2. Интерактивная игра-тренажёр «Решаем с Леопольдом.  /Закрепление приёмов вычитания чисел с переходом через десяток в пределах 20/
  3. Игра-тренажёр»Орехи для белочки. /закрепление знания табличных случаев — 1 класс/
  4. Игра-тренажёр «Собери бананы», 1 класс /»Сложение и вычитание чисел в пределах 20/
  5. Интерактивная игра-тренажёр«Помоги Русалке» / таблица сложения чисел в пределах 20/.  
  6. Интерактивная игра-тренажёр «Игра в футбол», /закрепление сложения чисел в пределах 20/. 
  7. Дидактическая игра«Найди домик» /сложение и вычитание в пределах 100, без перехода через разряд/.  
  8. Интерактивный тренажер «Накорми собаку». Математика,  /таблица сложения в пределах 20/.
  9. УЧИМСЯ СЧИТАТЬ
  10. Тренажер по математике «Новогодний сюрприз»,1 класс
  11. Презентация-тренажёр по математике «Состав чисел 2-10»; 1 класс
  12. Состав числа (1 класс)
  13. Навык счёта
  14. Интерактивный тест «Сложение и вычитание в пределах 100» /2 класс/
  15. Итоговый тест (2 класс)
  16. Интерактивный тренажёр «Сложение и вычитание в пределах 20»
  17. Интерактивный тренажёр «Вычитание в пределах 100»
  18. Интерактивный тренажёр«Сложение в пределах 100»
  19. Дидактическая игра  «Ромашки для кошки»; 2 класс
  20. Дидактическая игра «Давай поиграем»; 2 класс

    ТАБЛИЦА УМНОЖЕНИЯ:

    1. Отличник
    2. Игра Таблица умножения
    3. Медовый марафон (игра)
    4. Игра Таблица умножения 
    5. Математический тетрис / для тренировки и закрепления таблицы умножения в начальной школе/
    6. Таблица умножения v4. 0  /Программа для обучения и тестирования детей/
    7. Интерактивный тренажёр / отработка табличных случаев деления/
    8. Таблица умножения в мультиках
    9. Повторяем таблицу /умножения/
    10. УЧИМСЯ СЧИТАТЬ (отличный тренажёр)
    11. БИ2О2Т (учись считать играючи)
    12. Тренажёр навыка умножения
    13. ИграемСами(таблица умножения)
    14. Тест Знаешь таблицу умножения?
    15. САМОУЧКА Таблица умножения 6-9
    16. САМОУЧКА Открой таблицу умножения
    17. Интерактивный тренажёр«Таблица умножения»
    18. Презентация-тренажер «Таблица умножения»
    19. Игра-тренажер «Нарядим елочку!»
    20. Тренажёр «Таблица умножения»
    21. Интерактивный тренажер «Играем в снежки»
    22. Интерактивный тренажёр» В гости к Винни-Пуху. Табличное умножение и деление»; 3 кл.
    23. Пазл«Снежная Королева. Табличное деление«; 2-4 классы
    24. Тренажер-игра«Незнайкина мозаика. Табличное умножение»; 2-4 классы

    ЗАДАЧИ:

    1. Задачи

        РУССКИЙ ЯЗЫК

        1. САМОУЧКА. Русский языкI�span>
        2. ОНЛАЙН — ТЕСТЫ по русскому языку;
        3. ПИШИ ПРАВИЛЬНО!
        4. Повторение изученного за 1 полугодие
        5. Итоговый тест (2 класс)
        6. Время глагола 
        7. Игра-тренажёр по русскому языку «Цветы для Красной Шапочки».  /«Буква парного согласного на конце слов»/.  
        8. Интерактивный тренажёр «Правописание парных согласных»
        9. Игра-тренажёр «Помоги Бабе-Яге. / «Буква безударного гласного в корне слова»/  
        10. Тренажёр «Рыбалка» / «Правописание безударных гласных в корне слова»/.
        11.  Интерактивный тренажер «Безударные гласные. Маша и медведь»
        12.  Интерактивный тест «Безударные гласные»; 2 класс
        13.  Интерактивный тест «Фонематический анализ и синтез» /2-3 класс/
        14. Игра-тренажёр «Алёша Попович ищет Любаву. Определение частей речи» /2-3 классы/
        15. Интерактивный тренажёр «Части речи»
        16. Интерактивный тренажёр «Части речи»
        17. Тест «Части речи» / 2 класс/
        18. Интерактивный тренажёр «Родственные слова»
        19. Презентация-тренажёр «Родственные слова» ( 2-3 классы)
        20. Интерактивный тренажёр«В мире родственных слов»
        21. Презентация-тренажер «Род имён существительных»; 1-2 классы
        22. Тренажер«Состав слова (приставка)»; 3 класс
        23. Интерактивный кроссворд«Многозначные слова»

        СЛОВАРЬ

        1. Слова из словаря
        2. Словарные слова «Растения»
        3. Словарные слова «Животные»
        4. Интерактивная игра «Словарные слова»
        5. Викторина по русскому языку«Во саду ли, в огороде»
        6. Презентация-пазл» Животные»;
        7. «Иллюстрированный словарь» для 3 класса
        8. «УРОЖАЙ.  Словарная работа»; 2-3 классы

        МАТЕМАТИКА

        1. Действия с именованными числами
        2. Дроби
        3. Определивремя по часам / умение определять и быстро рассчитывать время/.
        4. Интерактивная презентация «Своя игра»  

        ОКРУЖАЮЩИЙ МИР

        1. Угадай детёныша 
        2. Животные природных зоны
        3. О молниях, змеях и прочих...
        4. Игра»Слово» /При запуске презентации, выберите «Включить содержимое для этого сеанса/ — 4 класс.
        5. Тренажер по окружающему миру «Мир вокруг нас» (1-2 класс)
        6. Олимпиадные вопросы по окружающему миру
        7. Интерактивный кроссворд «Вода» /3 класс/
        8. Тест «Группы животных» /2 класс/
        9. Интерактивная интеллектуальная игра по экологии«Юные экологи»
        10. Интерактивный тестс автоматизированной проверкой ответа на тему «Космос»
        11. Итоговый тест  (2 класс)
        12. Презентация«Детский месяцеслов»; 1-2 класс
        13. Тест«Водоемы»; 2 класс
        14. Тест  /4 класс/

        ЧТЕНИЕ

        1. Интерактивная игра «Сказки А. С.Пушкина», 3 — 5 класс

        ШЕСТИЛЕТКИ

        1. Игра — презентация «Колобок. Найди 10 отличий». / найти 10 отличий в картинках, нажав на них. В конце ребенку предлагается прослушать сказку./
        2. Игра — сказка «Курочка Ряба»  /Ребёнку предлагается найти 6 слов, изображенных в картинках, нажав на них. В конце игры предлагается прослушать сказку/.
        3.  

        Классные игры для обучения таблице умножения

        Изучите приведенные ниже инструкции по забавным играм, которые подходят для классных комнат, небольших групп и отдельных учащихся.

          = Любимая игра multiplication.com!


         Онлайн-игры

        Подходит для: Физические лица
        900 05 Продолжительность: Варьируется
        Сложность: Легко
        Уровень шума: Шумно

        • Ознакомьтесь с нашим огромным разнообразием онлайн-игр, отлично подходящих для самостоятельного обучения 90 051
        • Недавно мы представили несколько многопользовательских игр с новыми вызовы!

        Посетите нашу страницу с играми, чтобы начать.


        Вокруг света

        Подходит для: Классы
        Продолжительность: Длинная
        Сложность: Легкая
        900 05 Уровень шума: Тихо

        • выбрано.
        • Первый игрок стоит позади ученика рядом с ним.
        • Учитель держит карточку.
        • Ученик, ответивший первым, переходит к следующему ученику.
        • Если сидящий ученик первым говорит ответ, ученики меняются местами.

        Этот процесс продолжается до тех пор, пока хотя бы один ученик не сделает полный круг.


        Buzz

        Подходит для: Небольшие группы
        Продолжительность: 900 06 Гибкий
        Уровень сложности: Простой
        Уровень шума: Тихий

        Эта игра используется для повторения определенного семейства фактов, и в нее можно играть в небольшой группе или со всем классом.

        • Выберите число от 2 до 9. Первый учащийся говорит 1, следующий учащийся говорит 2 и так далее.
        • Вместо того, чтобы произносить число, кратное выбранному числу, учащийся произносит «жужжание».
        • Если игрок забывает произнести жужжание или произносит его в неподходящее время, он или она выбывает
        • Продолжайте, пока группа не наберет последнее число, кратное 9.
        • Например, если выбрано «2». Первый учащийся говорит «1», следующий учащийся говорит «жужжание», следующие учащиеся говорят «3», следующий учащийся говорит «жужжание» и так далее, пока не будет достигнуто число 18 (2 x 9).

        Материалы не требуются.


        Карточки у дверей

        Подходит для: Классы
        9 0005 Продолжительность: Короткая
        Сложность: Легко
        Уровень шума: Шумно

        • Выберите карточки, соответствующие фактам студенты учатся.
        • Когда учащиеся выстраиваются в очередь, чтобы войти или выйти из класса, поднимите флеш-карту, когда каждый учащийся проходит через дверь.
        • Ответ на флеш-карту — «пропуск» в класс.
        • Если учащийся не решает задачу, он или она должны отойти в сторону и сообразить ответ, прежде чем он или она войдет в класс.

        Вы можете выбрать карточки в соответствии со способностями каждого учащегося. Это упражнение может занять немного времени в первые пару раз, когда вы попробуете его, но оно будет проходить быстрее, когда учащиеся запомнят факты.


        Flip Up

        Подходит для: Пары
        Длительность: 9 0006 Гибкий
        Сложность: Простой
        Уровень шума: Шумно

        Учащиеся соревнуются, кто сможет правильно ответить на все задачи за наименьшее время.

        • Два ученика сидят лицом друг к другу.
        • Один учащийся держит карточки, чтобы задача была обращена к другому игроку.
        • Другие ученики произносят ответ вслух, и ученик, держащий карточку, подтверждает ответ.
        • Если учащийся отвечает правильно, флэш-карта сбрасывается.
        • Если учащийся ошибся, карточка возвращается в колоду.
        • Учащийся продолжает до тех пор, пока не даст все правильные ответы и не поменяется местами.

         Бинго умножения

        Подходит для: Классы
        Продолжительность: Варьируется
        Сложность: Легко
        Уровень шума: Шумно

        • Распечатайте шаблон карты бинго и сделайте копию для каждого ученика.
        • Каждый учащийся выбирает любое из 25 чисел внизу карточки бинго и записывает по одному в каждом квадрате.
        • Удалите все нулевые флэш-карты, кроме одной.
        • Произвольно возьмите флешку и прочитайте задачу вслух.
        • Каждый учащийся с ответом на своей карточке бинго отмечает квадрат.
        • Продолжайте, пока кто-нибудь не выиграет бинго.

        Двойники

        Подходит для: Пары, небольшие группы
        9 0005 Длительность: Гибкая
        Сложность: Легкая
        Уровень шума: Шумно

        Изучение фактов умножения с 2 в качестве множителя может быть веселым и легким с домино.

        • Используйте двойные домино, чтобы продемонстрировать, что умножение на два равносильно сложению двойных чисел (например, 5 + 5 = 2 x 5, 6 + 6 = 2 x 6).
        • Попросите своих учеников составить утверждения о фактах сложения и умножения для двойных домино.
        Материалы:
        Домино

        Slap Happy

        9 0002
        Подходит для: Пары, небольшие группы
        Продолжительность: Гибкий
        Сложность: Легко
        Уровень шума: Шумный

        • Каждый игрок берет по 7 карт. В свою очередь каждый игрок ищет совпадающую проблему и продукт.
        • Если он или она совпали, он или она берет ложку, а все остальные игроки пытаются схватить оставшиеся ложки.
        • Игрок раскрывает карты. Если игрок прав, игрок без ложки пишет М (первая буква в УМНОЖИТЬ).
        • Если игрок ошибается, он пишет М. Карты помещаются в стопку сброса.
        • Если во время хода у игрока нет спичек, он берёт карты из стопки.

        Каждый раз, когда игрок произносит заклинание УМНОЖЕНИЕ, он выбывает из игры. Игра продолжается до тех пор, пока не останется один игрок.

        Материалы:
        Ложки (на одну ложку меньше, чем общее количество игроков, например, пять ложек на шестерых игроков)
        Два набора карточек (PDF; первый набор с ответами, второй с проблемами)
        Совет учителя: Используйте Slap-o-Matic (из настольной игры HandsDown) вместо ложек.

         Тэг команды

        Подходит для: Небольшие группы, классы
        Продолжительность: Варьируется
        Сложность: Легко
        Уровень шума: Шумный

        • Положите две одинаковые стопки флеш-карт на стол в передней части комнаты.
        • Разделите учащихся на две группы.
        • Попросите учеников выстроиться в две одиночные линии лицом к парте. Первый учащийся в каждой шеренге должен находиться примерно в 10 футах от парты.
        • Когда игра начинается, первый человек в очереди бежит к столу, берет первую карту из своей стопки, показывает карту команде, объявляет ответ, кладет карту в стопку сброса и затем спешит отметить следующий человек в очереди.
        • Если учащийся не знает ответа или дает неверный ответ, он кладет карточку в низ стопки и выбирает следующую карточку. Этот учащийся продолжает выбирать карточки до тех пор, пока не узнает ответ на одну из них или пока не будут выбраны пять карточек.
        • Две команды играют одновременно, и первая команда, которая правильно ответит на все факты умножения в своей стопке, побеждает.
        Материалы:
        Два набора карточек (PDF)

        Футбол с таблицей умножения

        Подходит для: Небольшие группы, классы
        Продолжительность: Варьируется
        Сложность: Высокая
        Уровень шума: Тихий

        • Создайте игровое поле с помощью проектора или меловой/белой доски.
        • На стороне карточки с ответом напишите результаты футбольного матча (например, 25-ярдовый пас в аут, 3-ярдовый бег крайним защитником, неполный пас, потеря 5 ярдов из-за нащупывания).
        • Создайте около 100 игр на флеш-картах. Большинство розыгрышей должны приносить хорошие результаты, но некоторые розыгрыши являются ошибками, чтобы добавить элемент неожиданности.
        • Разделите учащихся на две команды. Каждый игрок по очереди отвечает на факт.
        • Если он или она отвечает правильно, происходит розыгрыш карты. Если игрок пропускает факт умножения, противоположной команде дается шанс правильно ответить на этот факт.
        • Если другая команда отвечает на факт правильно, команда восстанавливает нащупывание и начинает с первой попытки.
        • Старт на 20-ярдовой линии в начале игры, в начале второго тайма и после тачдаунов. При 4-м дауне команда может выбрать 1-й даун, пант (40 ярдов) или попытку броска с игры (должен быть как минимум на 40-ярдовой линии).

        Вы можете быть изобретательны со штрафами (например, штрафы могут быть наложены за невнимательность, когда не ваша очередь, чрезмерную болтовню или помощь кому-то другому).


         Война

        Подходит для: Пары
        Длительность: 9 0006 Варьируется
        Сложность: Легко
        Уровень шума: Шумно

        • Напишите на доске: Туз = 1, В = 0, Д = 11, К = 12.
        • Разбейте учащихся на команды по 2 человека и попросите их перетасовать свои карточки.
        • Попросите учеников разложить карты на две стопки и сложить стопкой лицевой стороной вниз перед каждым игроком.
        • Оба ученика одновременно переворачивают свои верхние карты. Как можно быстрее они перемножают 2 карты вместе и выкрикивают ответ.
        • Учащийся, который первым дает правильный ответ, кладет карты в свою выигрышную стопку. В случае ничьей учащиеся должны продолжать переворачивать свои карты до тех пор, пока кто-нибудь не выиграет стопку.

        Когда весь исходный стек разыгран, игроки подсчитывают свои выигрыши. Побеждает игрок с наибольшей суммой.

        Материалы:
        Одна колода игральных карт на каждую пару учащихся
         

        I Mean Number

        90 014
        Подходит для: Классы
        Продолжительность: Гибкая
        Сложность: Легкая
        Уровень шума: Шумно

        • Создайте именную бирку для каждого учащегося и напишите факт умножения (например, 7 x 5) на именной бирке, а не его или ее имя.
        • Каждый ученик в течение дня носит бирку с именем. Когда учащийся хочет поговорить с кем-то, он должен назвать его или ее ответом на свой факт умножения. (например, 35).
        Материалы:
        Именные бирки
         

        Как помочь ученикам с таблицей умножения?

        В этой статье я расскажу о некоторых общих принципах, помогающих учащимся с таблицей умножения:

        1. Зачем изучать таблицы?
        2. Структурированное изучение таблиц
        3. Пример: запоминание таблицы 3 в пять шагов
        4. Нужны ли упражнения на время?
        5. Игры на умножение
        6. Музыка
        7. А мнемонические подсказки?
        8. Умножение мамонта по математике 1 книга


        Зачем изучать таблицу умножения?

        Я чувствую, что изучение таблицы умножения даже важнее , чем освоение фактов сложения и вычитания. Почему? Потому что хорошее знание таблиц облегчает изучение основных фактов деления , многозначного умножения, деления в длинное, математики большинства дробей и факторизации . Даже в алгебре вам все равно нужно будет уметь упрощать рациональные выражения и многочлены множителей, возможно, даже перемножать матрицы.

        Изучайте и осваивайте таблицу умножения! — НОВЫЙ курс INTERACTIVE с видео и интерактивными упражнениями.

        • Ваш ребенок сможет освоить столы раз и навсегда!
        • Обучающие видео
        • Структурированное сверло
        • Интерактивные упражнения с оцениванием
        • Проблемы со словами
        • Шаблоны и задачи
        • Соединения с концепциями
        → Проверьте это!

        Или, можно сказать, так: если ваш ребенок не знает таблиц, ему будет очень трудно освоить все эти темы. Я не говорю, что дети не будут изучать эти темы концептуально — я имею в виду, что им будет трудно быстро решать задачи и упражнения, и вместо этого они могут «увязнуть» только в умножениях.

        Вот почему я считаю, что каждый учитель/родитель должен приложить все усилия, чтобы их ученики выучили таблицу умножения. Потратьте на это 1-2 месяца. Это может окупиться!

        Тем не менее, многие студенты в современном мире борются со столами. В этой статье подробно объясняется, как учить таблицу умножения в структурированной манере , а также даются другие полезные советы для практики.


        Структурированное изучение таблиц

        Этот метод направлен на запоминание определенной таблицы умножения с помощью так называемых КОНСТРУКЦИОННОЕ СВЕРЛО . Она существенно отличается от случайной детализации тем, что мы фактически используем структуру каждой таблицы в детализации. Вы должны начинать упражнения только после того, как ребенок поймет саму концепцию умножения.

        Пожалуйста, посмотрите этот список бесплатных ВИДЕО (по одному для каждой таблицы), которые обучают таблицам от 2 до 12, используя это структурированное упражнение.

        При использовании этого метода (или любого упражнения) обязательно объясните ученику, что цель состоит в том, чтобы запомнить факты — вспомнить их по памяти, а не получать ответы счетом или каким-либо другим методом. Так же, как ваш ребенок, вероятно, уже запомнил ваш адрес и телефон число, теперь она или он собирается запомнить некоторые математические факты. Вы должны ожидать, что ребенок ответит немедленно, когда вы тренируете. Если он или она не знает ответ наизусть (по памяти), то скажите ему или ей правильный ответ.

        Короткие тренировки обычно лучше всего . Например, вы можете тренироваться пять или десять минут за раз, в зависимости от объем внимания ребенка.

        Тем не менее, постарайтесь проводить по крайней мере два сеанса в течение дня в соответствии с вашим графиком. разрешает. Исследования того, как мозг обучается, показали, что новые воспоминания быстро забывается, и что новую информацию лучше всего сохранять, когда она просматривается в течение 4-6 часов 90 734 с момента первоначального изучения. (Кстати, это принцип применим ко чему-либо новому, чему человек учится.)

        Работа с карандашом и бумагой, которую ученик выполняет в одиночку, на самом деле не работает хорошо для запоминания фактов, потому что ребенок может получить ответы, считая и не по памяти. Правильная дрель требует затрат времени от инструктор. Если можете, привлекайте старших братьев и сестер к выполнению задания по сверлению.

        Вот пятишаговый метод запоминания, взятый из моей книги Math Mammoth Multiplication 1. Обычно только несколько шагов будут включены в любой сеанс, в зависимости от концентрации и способностей ребенка.


        Пример: запоминание таблицы 3 за пять шагов

        Вы можете просмотреть короткое видео, объясняющее основные пункты структурированного сверла здесь:

        Имейте таблицу, которую нужно выучить, уже написанную на бумаге. Здесь мы будем использовать Таблица из трех в качестве примера.

        1 х 3 = 3
        2 х 3 = 6
        3 х 3 = 9
        4 х 3 = 12
        5 х 3 = 15
        6 х 3 = 18
        7 х 3 = 21
        8 х 3 = 24 90 395 9 × 3 = 27
        10 × 3 = 30

        11 × 3 = 33
        12 × 3 = 36

        1. Первое задание — запомнить список ответов (список пропусков). Пусть ваш ребенок изучит первую половину списка пропусков счета (3, 6, 9, 12, 15, 18), произнося цифры вслух, указывая на ответы один за другим пальцем или ручкой. Эта техника использует чувства зрения, слышать и осязать одновременно, чтобы зафиксировать информацию в мозгу. После он просмотрел список несколько раз, попросите его повторить его по памяти.

          Ожидайте, что ваш ребенок ответит, и не давайте ей ответы слишком легко, потому что, ТОЛЬКО приложив усилия, она запоминайте факты. Как и мышцам, разум нуждается в упражнениях, чтобы стать сильнее.

          Требуйте, чтобы она запомнила список пропусков как вперед, так и назад. Продолжайте практиковаться, пока она не сможет «отбарабанить» первый список из 3, 6, 9, 12, 15, 18. С некоторыми таблицами, такими как таблицы 2, 5 и 10, это помогает указать на закономерность в них. Шаблон в таблице из 9более тонкий, но все же полезный.

        2. Тогда заняться последней половиной списка: 21, 24, 27, 30, 33, 36. Сделайте то же самое, что и с первой половиной списка.
        3. Далее работа с весь список ответов (список пропусков). Практикуйте список, двигаясь вверх по и вниз по , пока не проходит гладко и легко. Эти шаги может хватить на один день. Но обязательно просмотрите позже в день.

        4. Далее, случайным образом отрабатывайте отдельные задачи , оставляя весь список задач видимым для студента (конечно, без ответов). Закройте ответы. Можно устно спросить («Сколько будет 5 умножить на 3?»), указывая на задачу на графике. Я рекомендую читать вопрос вслух, одновременно указывая на проблему, потому что, опять же, использование нескольких органов чувств помогает зафиксировать информацию в тексте. ум лучше. Посмотрите видео, чтобы увидеть, как это делается.

          Цель на этом этапе – связать каждый ответ 3, 6, 9, 12, 15, 18, 21, 24, 27, 30, 33, 36, с определенным факт умножения (например, 7 × 3).

          Вы также можете смешивать факты из предыдущих таблиц, которые она уже знает эти новые задачи и тренирует их вместе с карточками.

        5. Последним шагом будет сверление в обратном направлении вокруг . Теперь вы говорите ответ («21»), и студент должен произвести задача («3 × 7»). Держите таблицу видимой, скрывайте проблемы (но не ответы) и указывайте на ответы в произвольном порядке. Эта техника может работать и по-другому. вокруг, где студент говорит ответы, а вы создаете проблемы. Иногда давайте неправильные ответы, чтобы проверить их.

          В качестве расширения вы можете произнести ответы от несколько таблиц, которые вы изучили, и попросите студента дать соответствующие проблема. Иногда ответов несколько. Например, 36, 30, 24 и т. д. 20 находятся в нескольких разных таблицах умножения. Это особенно хорошо упражняйтесь, готовясь к понятиям деления и факторинга.

        Запоминание, вероятно, не произойдет за одну ночь. В последующие дни вы можете смешайте шаги 1-5 (обычно вам не нужно слишком концентрироваться на шагах 1 и 2 после первой практики). Этот вид сверления требует немного времени и усилий от учителя, но может быть очень эффективный. Домашние школьники, очевидно, могут делать некоторые из них во время занятий. другие задачи, такие как путешествие в машине и так далее.

        Пока вы делаете эту таблицу за таблицей, вы также можете попробовать научить процесс ребенку, чтобы он научился запоминать сам. Она может скрыть ответы и попытаться воспроизвести список в уме.

        Посмотрите также мою серию бесплатных видеороликов по таблицам умножения, по одному видео для каждой таблицы, используя это структурированное упражнение.


        Онлайн-практика

        Вы можете попрактиковаться в таблице умножения онлайн прямо здесь, на сайте mathmammoth.com. Вы можете выбрать точные таблицы, которые вы хотели бы практиковать, время или время тренировки, а также количество вопросов.

        Другой вариант — использовать инструмент Дэвида Чендлера из «Математики без границ» под названием «Изучай их вперед и назад». Это поможет вам попрактиковаться и освоить таблицу умножения, используя ТАКОЙ же порядок изучения таблиц, как и в книге Math Mammoth Multiplication 1.

        Этот инструмент также тренирует таблицы как вперед, так и назад одновременно — это именно тот подход. Я взял в своих книгах и видео таблицы умножения. Практиковать их в обоих направлениях так здорово, потому что это позволяет учащимся осваивать «замаскированные» факты деления одновременно.

        Что это значит? Например, учащиеся не только изучают ответы на вопросы 2 × 6, 3 × 6, 4 × 6, 5 × 6 и т. д., но и практикуются в решении задач с недостающими факторами: ___ × 6 = 12, ___ × 6 = 30, ___ × 6 = 72, ___ × 6 = 42 и т. д.


        Другие полезные идеи

        Изучайте и осваивайте таблицы умножения! — НОВЫЙ курс INTERACTIVE с видео и интерактивными упражнениями.

        • Ваш ребенок сможет освоить столы раз и навсегда!
        • Обучающие видео
        • Структурированное сверло
        • Интерактивные упражнения с оцениванием
        • Проблемы со словами
        • Шаблоны и задачи
        • Соединения с концепциями
        → Проверьте это!

        • Повесьте на стену 12 на 12 сетку (или 10 на 10) с написанными на ней шаблонами для пропуска счета. Напоминайте ребенку смотреть на нее несколько раз в день. Это может творить чудеса для визуалов!
        • Рядом повесьте еще один плакат с пустым сетка, в которую ребенок заносит те факты, которые он усвоил. Посмотрите пример того, как я использую сетку 12×12 в конце урока для таблицы 3 (взято из моей книги Math Mammoth Multiplication 1).
        • Повторите списки пропусков или факты умножения вслух перед сном. Это может превратить их в усвоил факты к следующему утру.
        • Если вы хотите использовать карточки, обратите внимание на эти: Умножение наизусть из книги «Математика для любви». Они укрепляют концепцию умножения в то же время, практикуя беглость речи.

        Нужны ли тренировки на время?

        Мне кажется, что упражнения на время — лишь один из многих инструментов, когда дело доходит до изучения математики. факты. Некоторые дети будут процветать на них; другими словами, они будут учиться быстро, когда они используются. Возможно, им нравится соревноваться в гонках с часы. Существует ряд компьютерных игр на время, которые очень хорошо подходят для факты бурения. Вот некоторые онлайн:

        • Онлайн-таблицы умножения Практика здесь, на MathMammoth.com — вы можете выбрать практику по времени или без, количество вопросов и таблицы, которые вы хотите попрактиковать.
        • Игра «Сопоставление таблиц умножения со скрытыми картинками» — выберите, какие именно таблицы вы хотите попрактиковать.
        • Игры Math Magician имеют простой 1-минутный обратный отсчет, и если вы ответите на 20 вопросы в это время, вы получите награду.

        • Программное обеспечение Sheppard содержит несколько типов игр, предназначенных только для практики математических фактов, включая тренировки на время. Некоторые из игр не учитывают время, но дают вам больше очков, чем быстрее вы идете.

        Вы также можете использовать обычные распечатанные рабочие листы по умножению для таблиц умножения

        Тем не менее, для других учеников упражнения на время могут быть контрпродуктивными, и они могут закончиться слезами и разочарованием. Доказательство в пудинге: просто попробуйте и посмотрите, что из этого выйдет.


        Игры на умножение

        Игры очень полезны, когда вы переходите к этапу «случайного повторения» (после выполнения структурированных упражнений). Здесь я составил ДЛИННЫЙ список онлайн-игр и программного обеспечения для умножения.

        Компьютерные упражнения могут быть очень полезными для детей, когда они начинают замечать, что действительно узнают факты и могут успешно выполнять упражнения. Они действительно могут прийти, чтобы насладиться процессом.

        Заслуживает внимания и эта простая карточная игра (версия «Война товаров»). По сути, каждому игроку раздаются две карты лицом вверх, игроки умножают их, и человек с наибольшим произведением получает все карты в этом раунде.

        Или попробуйте игру в кости под названием Damult Dice.


        Музыка

        Музыка (песни) может очень эффективно помочь детям выучить таблицу умножения. Она работает с мозгом способами, о которых мы еще не до конца осознаем, но несомненно, что музыка улучшает обучение и работу мозга.

        Вы можете просто использовать знакомую мелодию («Гребите, гребите, гребите в лодке», «С днем ​​рождения», «Б-и-н-го» или любую другую мелодию, которую ребенок уже знает) и просто добавляйте цифры вместо слов. Некоторым детям больше нравится рэп, и эти мелодии тоже работают. Или, если вы хотите, чтобы песни были специально написаны для таблицы умножения, посмотрите мой обзор Гора умножения CD Хэпа Палмера.

        Еще одна полезная идея состоит в том, чтобы использовать небольшие «танцевальные» упражнения (прыжки, вращение, касание пола, марширование и т. д.) вместе с музыкой для кинестетического ученика. Девочки могут воспринимать это как «танец», а мальчики — как «движение». Большинству детей понравится вставать со своих мест. Игра «ручной джайв» может быть средством обучения пропускному счету, например, 4, 8, 12, 16, 20…

        Использование музыки и движения может быть особенно полезным для учащихся, которые боятся математики, потому что им это не кажется «математикой». Обходя свой «ментальный блок», они могут без труда выучить таблицы — они воспринимают это как забаву, а «математические триггеры», которые мозг использует для «блокировки» мозга и создания «реакции страха», никогда не стимулируются. Кроме того, младшие братья и сестры часто разучивают песни (и их следует поощрять к разучиванию) в одно и то же время.


        Как насчет мнемоники для таблицы умножения?

        Мнемонические устройства сами по себе неплохи. Мы используем их постоянно, в повседневных жизненных ситуациях. Может быть, у вас есть телефонный номер, который вы делите на двузначные числа, и помните, что в нем есть последовательные числа или двойные числа и т. д. Однажды я запомнил определенный четырехзначный пин-код, представив его как два двузначных числа, и вспомнив, что последний был на 9 меньше, чем первый… но через некоторое время я вспомнил его без этого.

        Например, Times Tales — мнемоническая программа для таблицы умножения. С каждым «сложным» фактом таблицы умножения связывается глупая история и картинка. Например, если ребенок выучит 8 × 7 с помощью Times Tales, он запомнит дурацкую картинку с изображением «девушки восемь» и символа «семь» за рулем автомобиля, а также запомнит фразу «Сейчас 56». Это не слишком отличается от использования такой рифмы, как «5, 6, 7, 8, пятьдесят шесть семь раз по 8». Такая программа также может повысить уверенность в себе.

        На некоторых детей эти глупые истории не действуют, потому что они плохо запоминают истории, и тогда это может вызвать разочарование. Кроме того, детям постарше могут больше не нравиться глупые истории. Все такие «помощи» хороши на своем месте, но вы должны убедиться, что мнемоническая «помощь» сама по себе не станет дополнительным бременем.

        Имейте в виду, что мнемонические подсказки, конечно, являются только дополнительными (дополнительными) ресурсами и не заменяют изучение самой концепции умножения или изучение структур в таблицах.


        Баланс

        БАЛАНС, как и во всем. Если вы уже приложили значительные усилия, а ребенок не запоминает таблицы, пожалуйста, не делайте из таблицы умножения причину, по которой ваш ученик ненавидит математику. Вы можете отступить и повторить попытку позже. Некоторые дают своим ученикам «костыль» — выписанные таблицы — и в конце концов ребенок замечает, как медленно ему/ей приходится проверять ответы в таблице вместо того, чтобы знать их, и решает запомнить их.

        Мария Миллер


        См. также

        Онлайн-практика с таблицами умножения
        Простая и бесплатная онлайн-практика здесь, на MathMammoth.com — вы можете выбрать практику по времени или без по времени, количество вопросов и таблицы, которые вы хотите упражняться.

        Структурированные обучающие видеоролики для таблицы умножения
        Набор видеороликов, в которых используется метод структурированного повторения для таблицы умножения.

        Игра «Сопоставление таблиц умножения со скрытыми картинками» — выберите, какие именно таблицы вы хотите попрактиковать. Эта игра прямо здесь, на MathMammoth.com!


        Комментарии

        Я просто хотел поблагодарить вас за этот учебный план! Мы перешли на домашнее обучение в этом году, 4-й класс. Моя дочь начала медленно, и я не был уверен, что это правильный продукт для нее. Потом я понял, что она недостаточно хорошо знает математические факты, чтобы двигаться дальше. Так что я просто вернулся к главам с фактами о математике в младших классах, и она поработает над ними. Ваши листы умножения потрясающие! Ваш способ преподавания настолько великолепен, что она действительно быстро схватывает их, и мы скоро сможем перейти к учебной программе 4-го класса. Вы действительно учите концепциям, а не только решению проблем. В отличие от многих домашних школьников, мне никогда не придется тратить больше денег на математическую программу за математической программой, пытаясь найти ту, которая работает. Спасибо!!
        Мишель

        Между прочим, я просто хочу, чтобы вы знали, какой находкой были ваши материалы. Как любитель математики, я был в восторге, когда наткнулся на учебный план, который преподавал математику именно так, как я «думаю». Ваши объяснения замечательны, а количество практики идеально. После пяти лет борьбы с математикой мой шестиклассник вырос как на дрожжах. Сейчас она действительно любит математику. Прежде чем получить математику, она провела три года, работая над таблицей умножения, используя карточки, упражнения, компьютерные игры, мнемонические устройства и программы с рифмованными историями в сочетании со своими учебниками. Все безрезультатно. После 5 месяцев использования ваших продуктов и методов она запомнила все свои факты умножения и перешла к делению в длинное число. Мой 5-летний ребенок слушает и действительно умеет считать двойки, тройки, четверки и пятерки без пропусков. Видимо ваши методы работают. БОЛЬШОЕ ВАМ СПАСИБО за всю вашу работу.

        Многие благословения,

        Кристин Суонсон

        Просто хочу поблагодарить вас за Math Mammoth! Очень доступный. Я выбросил все остальное, и это все, что мы используем. Моя ненавидящая математику дочь, которая только что закончила третий класс, выполняет умножение из синей серии, чтобы подготовиться к четвертому классу в следующем году. Она делает это сама летом, потому что ей это нравится! Никогда не думал, что увижу это. Она действительно боролась с умножением, и она говорит, что это облегчает ей понимание. Мне нравится возможность использовать синюю серию, чтобы заполнить пробелы в том, что они не «получают» с первого раза. Это так настраивается для разных детей. Я тоже люблю формат pdf. Я могу собрать книгу в любом порядке. Мне нравится, когда нет учебника для учителя. С тремя детьми трудно тасовать 3 книги. Это намного лучше, потому что, когда у каждого ребенка есть вопросы, у них уже все есть с собой, когда они приносят их мне. Я был так взволнован, чтобы найти это. Желание Я нашел его пару лет назад! Спасибо за этот учебный план и спасибо за то, что сделали его доступным. Я купил все 6 уровней сразу. Я никогда не мог позволить себе сделать это с любой другой учебной программой. На этом борьба (по большей части) с моей дочерью закончилась. Спасибо, что сделали обучение моих детей математике доступным, легким и даже приятным!

        Сара

        Мария, я просто хотел отправить вам короткое сообщение, чтобы сказать сердечное спасибо за то, что вы предоставили мне инструменты, чтобы так хорошо обучать математике моих детей. Для меня это первые дни с материалами Math Mammoth, так как я только что заказал светло-голубые тексты 1A и 3A/B в Rainbow Resource (нашел их по рекомендации одного из их специалистов по вопросам и ответам).

        До того, как остаться дома с детьми, я преподавал в 1-м и 2-м классах в государственных школах, в основном как учитель чтения/письма. Поскольку я хочу вернуться к преподаванию, когда мой младший подрастет, мои дети учатся в государственной школе, но я продолжаю их занятия, чтобы заполнить пробелы или обогатить их. Даже со всем моим «обучением» я был действительно озадачен тем, как лучше всего учить математике моих старших мальчиков. Учебная программа в их школе просто не кажется систематической или достаточно всеобъемлющей. Я думаю, что учителя делают все возможное с материалами, которые им дают.

        Тем не менее, мой 3-классник столкнулся с трудностями в освоении его фактов умножения, и ваше сегодняшнее электронное письмо, наряду с видео, так поучительно! Я бы никогда не подумал учить таблицы таким образом. Большое спасибо! Я бы хотел, чтобы все учителя государственных школ имели доступ к таким замечательным материалам, которые вы предоставляете (хорошо, если у них есть подключение к Интернету, но вы знаете, что школьная система предлагает им так много всего другого). Я надеюсь, что вы получите много положительных отзывов от многих людей за ваши усилия, потому что я уверен, что ваши усилия приносят пользу многим. Спасибо, что поделились своими знаниями!!!

        Дженнифер Новицки

        Математический мамонтовый тур


        Запутались в различных вариантах? Совершите виртуальный электронный тур по Math Mammoth! Вы получите:

        Первоначальное электронное письмо для загрузки вашего ПОДАРКА из более чем 400 бесплатных рабочих листов и образцов страниц из моих книг. Шесть других электронных писем «TOURSTOP» , которые объясняют важные вещи и часто задаваемые вопросы, касающиеся учебного плана Math Mammoth. (Узнай отличия между всеми этими разноцветными сериями!)

        Таким образом, у вас будет время переварить информацию в течение одной-двух недель, а также возможность лично спросить меня об учебной программе. Ежемесячный сборник советов по обучению математике и обновлений Math Mammoth (отказаться от подписки в любое время)

        Мы уважаем конфиденциальность вашей электронной почты.

        Примечание : СНАЧАЛА вы получите электронное письмо с просьбой подтвердить свой адрес электронной почты. Если вы не можете найти это электронное письмо с подтверждением, проверьте папку СПАМ/НЕПЛАХ.

        «Мини» курс обучения математике


        Это небольшой «виртуальный» двухнедельный курс, где вы будете получать электронные письма по важным темам преподавания математики, в том числе:

        — Как помочь учащемуся, отстающему
        — Проблемы со словами
        — Обучение таблице умножения
        — Почему дроби такие сложные
        — Значение ошибок
        — Стоит ли использовать временные тесты
        — И многое другое!

        Вы также получите:

        ПОДАРОК ​​ из более чем 400 бесплатных рабочих листов и образцов страниц из моих книг в самом начале.

        Формула квадратичная функция: Построение графика квадратичной функции — урок. Алгебра, 8 класс.

        404 — Страница не найдена

        Страницы

        Партнеры сайта

        _________________________________


        404: Запрошенная страница с адресом [http://primer. by/algebra/funkcii/kvadratichnaja-funkcija] не найдена.

        Если Вы уверены, что набрали ссылку корректно, напишите, пожалуйста, об этом на:

        меню пользователя

        Новости


        30.11.16 


        17.03.15 


        25.03.14 


        29.08.13 


        05. 05.13 



        primer. by 2013-2016

        Квадратичная функция График квадратичного уравнения рабочего листа функции, формула функции, угол, текст, треугольник png

        Квадратичная функция График квадратичного уравнения рабочего листа функции, формула функции, угол, текст, треугольник png

        теги

        • угол,
        • текст,
        • треугольник,
        • другие,
        • график функции,
        • учитель,
        • параллель,
        • парабола,
        • квадратное уравнение,
        • квадратичная функция,
        • лист,
        • алгебра,
        • линия,
        • диаграмма,
        • уравнение,
        • fx,
        • f X 0,
        • функция,
        • формула функции,
        • график,
        • х 0,
        • png,
        • прозрачный,
        • бесплатная загрузка

        Об этом PNG

        Размер изображения
        1700x689px
        Размер файла
        43. 6KB
        MIME тип
        Image/png
        Скачать PNG ( 43.6KB )

        изменить размер PNG

        ширина(px)

        высота(px)

        Лицензия

        Некоммерческое использование, DMCA Contact Us

        • Квадратичное уравнение Квадратичная формула Квадратичная функция Математика, формула, угол, текст, прямоугольник png 1500x673px 7.22KB
        • Кривая параболы Квадратичная функция График функции Математика, Математика, угол, треугольник, график функции png 1024x620px 15.48KB
        • Квадратичное уравнение Квадратичная формула Алгебра Квадратичная функция, формула функции, угол, текст, прямоугольник png 1280x395px 13.9KB
        • org/ImageObject»> Квадратичное уравнение Квадратичная функция График функции Ноль функции, OneNote, синий, угол, текст png 2000x1714px 101.53KB
        • График функции Родительская функция Квадратичная функция Экспоненциальная функция, Математика, угол, текст, прямоугольник png 2000x2000px 75.49KB
        • Квадратичное уравнение Квадратичная функция Квадратичная формула Завершая квадрат, формула, угол, текст, прямоугольник png 2000x617px 30.38KB
        • Квадратичная функция Квадратичное уравнение Парабола Алгебра, Математика, угол, текст, симметрия png 700x750px 58.63KB
        • Математика геометрия формула евклидово уравнение, математические заметки, угол, текст, треугольник png 6354x6354px 911. 07KB
        • Квадратичная функциональная линия, Квадратичное уравнение, Степень, Полиномиальная функция, График функции, График, Парабола, Коэффициент, угол, площадь, круг png 1630x1553px 75.08KB
        • Квадратичное уравнение Квадратичная формула Квадратичная функция Ноль функции, Математика, угол, белый, текст png 1705x586px 10.08KB
        • математические уравнения, математические формулы, математические обозначения, cdr, угол, текст png 1080x763px 356.8KB
        • Формула Математика Евклидова, математическая формула, угол, текст, монохромный png 3500x3313px 875.77KB
        • Математика Геометрия Формула Тригонометрия Куб, Математика, угол, треугольник, монохромный png 1920x1308px 921. 05KB
        • математические уравнения, математическое уравнение евклидовой формулы, математический набросок материала, угол, текст, цифровой png 918x670px 147.15KB
        • Абсолютное значение Квадратичная функция Максимумы и минимумы Экспоненциальная функция, Математика, угол, текст, треугольник png 1200x1326px 37.18KB
        • Математические уравнения, Формула Математика Функция Евклида, Оси математических функций, синий, угол, текст png 800x800px 366.32KB
        • Система уравнений Математика Квадратичное уравнение Решение уравнений, рукописная математическая формула, угол, текст, число png 1920x2010px 152.49KB
        • org/ImageObject»> Квадратное уравнение Квадратный корень из 3-го корня Формула, математический вопрос, угол, текст, логотип png 1000x1000px 7.85KB
        • Квадрантная декартова система координат График функции Квадратичная функция Математика, 12 бис, угол, текст, прямоугольник png 907x907px 30.58KB
        • Квадратичная функция Формула квадратичного уравнения Parabola, Mc logo, оранжевый, другие, число png 1412x1071px 332.1KB
        • Декартова система координат График функции Диаграмма бумаги Плоскость, др., угол, прямоугольник, треугольник png 800x800px 30.81KB
        • Математика евклидова геометрия формула, математика, угол, текст, треугольник png 4050x4050px 420. 75KB
        • иллюстрация в черно-серой рамке, Диаграммная бумага, текстура, угол, белый png 1501x1501px 14.69KB
        • График функции Экспоненциальная функция Обратная функция Экспоненциальный рост, Математика, угол, текст, треугольник png 617x617px 9.15KB
        • Система линейных уравнений Математика, Математика, синий, угол, текст png 597x599px 16.58KB
        • Система линейных уравнений Система уравнений Решение уравнений, др., разное, угол, белый png 2266x1200px 31.69KB
        • Математика, математика, евклидова формула Компьютерный файл, рукописная математическая формула, угол, текст, класс png 4520x3161px 338. 87KB
        • Алгебра Математика Решение уравнений с переменными, угол, текст, логотип png 1050x1024px 68.67KB
        • Квадратичное уравнение Квадратичная функция Математика Квадратичная формула, Математика, синий, угол, текст png 1500x1125px 39.77KB
        • Квадратичное уравнение Квадратичная функция Квадратичная формула, формула, угол, текст, логотип png 2211x557px 27.73KB
        • График функции Линия Парабола Вершина, линия, угол, текст, треугольник png 1024x1024px 37.52KB
        • Линейная функция График функции Линейное уравнение, линейный график, угол, текст, треугольник png 700x446px 18. 01KB
        • График функции Математическое производное уравнение, математическое уравнение, угол, белый, текст png 1600x625px 8.41KB
        • Математическая формула Алгебра Евклидова, Математическая формула, угол, текст, монохромный png 2244x2244px 134.04KB
        • Математика евклидова формула бумаги, математические различные формулы, угол, текст, монохромный png 4050x4050px 627.53KB
        • Решение уравнений Математика Математическая запись Формула, администратор, текст, число, математик png 792x658px 240.21KB
        • Сюжет Квадратичная функция Квадратичное уравнение параболы, круг, угол, текст, треугольник png 2000x1211px 57. 78KB
        • красный х иллюстрация, красный х письмо компьютерные иконки, красный х, разное, угол, текст png 512x512px 10.3KB
        • Квадратичная функция Квадратичное уравнение Квадратичная формула, Математика, угол, белый, текст png 1512x661px 8.6KB
        • Теорема Пифагора Угол Числовая линия, Угол, угол, текст, прямоугольник png 2400x2384px 40.82KB
        • гистограмма, гистограмма, график функции, шаблон альбома роста, разное, угол, текст png 1436x1111px 458.07KB
        • Круг Коническое сечение Гипербола График функции Эллипс, круг, угол, текст, треугольник png 1572x1551px 140. 41KB
        • Точка симметрии уравнения гиперболической функции, Математика, угол, текст, треугольник png 1333x1014px 18.66KB
        • Parabola Normalparabel Математика Коническое сечение Функция, Математика, угол, текст, треугольник png 668x732px 21.69KB
        • Квадратичное уравнение Квадратичная функция Квадратичная формула Ноль функции, топор, угол, белый, текст png 1600x565px 24.22KB
        • Математика Число Точка Уравнение Геометрия, математическая формула, угол, текст, прямоугольник png 1546x646px 8.84KB
        • Формула Математика Алгебраическое уравнение Число, управляющий, угол, текст, параллель png 1427x1096px 64. 32KB
        • Квадратный корень n-й корень математика квадратное число ноль функции, математика, угол, текст, прямоугольник png 500x549px 9.46KB
        • Гистограмма Компьютерные иконки График функции, другие, разное, угол, текст png 980x736px 14.39KB
        • Квадратичная функция Математика Элемент финитарного отношения, Математика, угол, белый, текст png 1019x854px 74.57KB

        Квадратичная функция

        Общая форма квадратичной функции: ф ( Икс ) «=» а Икс 2 + б Икс + с . График квадратичной функции представляет собой парабола , тип 2 -мерная кривая.

        «Основная» парабола, у «=» Икс 2 , выглядит так:

        Функция коэффициента а в общем уравнении состоит в том, чтобы сделать параболу «шире» или «тоньше» или перевернуть ее вверх дном (если отрицательно):

        Если коэффициент Икс 2 положителен, парабола раскрывается; в противном случае он открывается вниз.

        Вершина

        вершина параболы – это точка в нижней части « U » форма (или вершина, если парабола направлена ​​вниз).

        Уравнение параболы также можно записать в «вершинной форме»:

        у «=» а ( Икс − час ) 2 + к

        В этом уравнении вершиной параболы является точка ( час , к ) .

        Вы можете увидеть, как это соотносится со стандартным уравнением, перемножив его:

        у «=» а ( Икс − час ) ( Икс − час ) + к

        у «=» а Икс 2 − 2 а час Икс + а час 2 + к

        Коэффициент Икс вот − 2 а час . Это означает, что в стандартной форме у «=» а Икс 2 + б Икс + с , выражение

        − б 2 а

        дает Икс -координата вершины.

        Пример:

        Найдите вершину параболы.

        у «=» 3 Икс 2 + 12 Икс − 12

        Здесь, а «=» 3 и б «=» 12 . Итак Икс -координата вершины:

        − 12 2 ( 3 ) «=» − 2

        Подставив в исходное уравнение, чтобы получить у -координата, получаем:

        у «=» 3 ( − 2 ) 2 + 12 ( − 2 ) − 12

        «=» − 24

        Итак, вершина параболы находится в точке ( − 2 , − 24 ) .

        Ось симметрии

        Ось симметрии параболы — это вертикальная линия, проходящая через вершину. Для параболы стандартной формы у «=» а Икс 2 + б Икс + с , ось симметрии имеет уравнение

        Икс «=» − б 2 а

        Обратите внимание, что − б 2 а также является Икс -координата вершины параболы.

        Пример:

        Найдите ось симметрии.

        у «=» 2 Икс 2 + Икс − 1

        Здесь, а «=» 2 и б «=» 1 . Итак, осью симметрии является вертикальная линия

        Икс «=» − 1 4

        Перехваты

        Вы можете найти у -перехват параболы простым вводом 0 для Икс . Если уравнение находится в стандартной форме, то вы можете просто взять с как у -перехват. Например, в приведенном выше примере:

        у «=» 2 ( 0 ) 2 + ( 0 ) − 1 «=» − 1

        Итак у -перехват − 1 .

        Икс -перехваты немного сложнее. Вы можете использовать факторинг , или завершение квадрата , или квадратичная формула найти их (если они существуют!).

        Домен и диапазон

        Как и любая функция, домен квадратичной функции ф ( Икс ) это набор Икс -значения, для которых определена функция, и диапазон множество всех выходных значений (значений ф ).

        Квадратичные функции обычно имеют областью определения всю действительную прямую: любая Икс является законным входом. Диапазон ограничен теми точками, которые больше или равны у -координата вершины (или меньше или равна, в зависимости от того, открывается парабола вверх или вниз).

        Квадратичная функция — стандартная форма, формула, примеры

        Квадратичные функции используются в различных областях техники и науки для получения значений различных параметров. Графически они изображаются параболой. В зависимости от коэффициента высшей степени решается направление кривой. Слово «Квадрат» происходит от слова «Квадрат», что означает квадрат. Другими словами, квадратичная функция — это «полиномиальная функция степени 2». Существует множество сценариев, в которых используются квадратичные функции. Знаете ли вы, что при запуске ракеты ее траектория описывается квадратичной функцией?

        В этой статье мы исследуем мир квадратичных функций в математике. Вы узнаете о графиках квадратичных функций, формулах квадратичных функций и других интересных фактах по теме. Мы также будем решать примеры на основе концепции для лучшего понимания.

        1. Что такое квадратичная функция?
        2. Стандартная форма квадратичной функции
        3. Формула квадратичных функций
        4. Различные формы квадратичной функции
        5. Область и диапазон квадратичной функции
        6. График квадратичной функции
        7. Максимумы и минимумы квадратичной функции
        8. Часто задаваемые вопросы о квадратичной функции

        Что такое квадратичная функция?

        Квадратичная функция — это полиномиальная функция с одной или несколькими переменными, в которой старший показатель переменной равен двум. Поскольку высший член квадратичной функции имеет вторую степень, поэтому его также называют многочленом степени 2. Квадратичная функция имеет минимум одного члена второй степени. Это алгебраическая функция.

        Родительская квадратичная функция имеет вид f(x) = x 2 и соединяет точки, координаты которых имеют вид (число, число 2 ). К этой функции можно применить преобразования, на которых она обычно имеет вид f(x) = a (x — h) 2 + k, а далее ее можно преобразовать в вид f(x) = ax 2 + бх + в. Давайте подробно изучим каждый из них в следующих разделах.

        Стандартная форма квадратичной функции

        Стандартная форма квадратичной функции имеет вид f(x) = ax 2 + bx + c, где a, b и c — действительные числа с a ≠ 0.

        Примеры квадратичных функций

        Уравнение квадратичной функции имеет вид f(x) = ax 2 + bx + c, где a ≠ 0. Рассмотрим несколько примеров квадратичных функций:

        • f(x) = 2x 2 + 4x — 5; Здесь а = 2, Ь = 4, с = -5
        • f(x) = 3x 2 — 9; Здесь а = 3, б = 0, с = -9
        • f(x) = x 2 — x; Здесь а = 1, Ь = -1, с = 0

        Теперь рассмотрим f(x) = 4x-11; Здесь a = 0, поэтому f(x) НЕ является квадратичной функцией.

        Вершина квадратичной функции

        Вершина квадратичной функции (в форме буквы U) находится там, где функция имеет максимальное или минимальное значение. Ось симметрии квадратичной функции пересекает функцию (параболу) в вершине.

        Формула квадратичных функций

        Квадратичную функцию всегда можно разложить на множители, но процесс факторизации может быть затруднен, если нули выражения являются нецелыми действительными числами или недействительными числами. В таких случаях мы можем использовать квадратную формулу для определения нулей выражения. Общая форма квадратичной функции задается следующим образом: f(x) = ax 2 + bx + c, где a, b и c — действительные числа с a ≠ 0. Корни квадратичной функции f(x) можно рассчитать по формуле квадратичной функции:

        • х = [-b ± √(b 2 — 4ac)] / 2a

        Различные формы квадратичной функции

        Квадратичная функция может иметь различные формы: стандартную форму, форму вершины и форму пересечения. Вот общие формы каждого из них:

        • Стандартная форма: f(x) = ax 2 + bx + c, где a ≠ 0,
        • Форма вершины: f(x) = a(x — h) 2 + k, где a ≠ 0, а (h, k) — вершина параболы, представляющая квадратичную функцию.
        • Форма точки пересечения: f(x) = a(x — p)(x — q), где a ≠ 0, а (p, 0) и (q, 0) — точки пересечения по оси x параболы, представляющей квадратичную функцию.

        Парабола открывается вверх или вниз в зависимости от значения ‘a’ меняется:

        • Если a > 0, то парабола открывается вверх.
        • Если a < 0, то парабола направлена ​​вниз.

        Мы всегда можем преобразовать одну форму в другую. Мы можем легко преобразовать форму вершины или форму перехвата в стандартную форму, просто упростив алгебраические выражения. Давайте посмотрим, как преобразовать стандартную форму в форму каждой вершины и форму пересечения.

        Преобразование стандартной формы квадратичной функции в вершинную форму

        Квадратичная функция f(x) = ax 2 + bx + c может быть легко преобразована в вершинную форму f(x) = a (x — h) 2 + k, используя значения h = -b/2a и k = f(-b/2a). Вот пример.

        Пример: Преобразуйте квадратичную функцию f(x) = 2x 2 — 8x + 3 в вершинную форму.

        • Шаг — 1: Сравнивая данную функцию с f(x) = ax 2 + bx + c, получаем a = 2, b = -8 и c = 3.
        • Шаг — 2: Найдите ‘h’ по формуле: h = -b/2a = -(-8)/2(2) = 2.
        • Шаг — 3: Найдите ‘k’ по формуле: k = f(-b/2a) = f(2) = 2(2) 2 — 8(2) + 3 = 8 — 16 + 3 = -5.
        • Шаг — 4: Подставить значения в вершинную форму: f(x) = 2 (x — 2) 2 — 5.

        Преобразование стандартной формы квадратичной функции в форму точки пересечения

        Квадратичная функция f(x) = ax 2 + bx + c может быть легко преобразована в вершинную форму f(x) = a (x — p)(x — q), используя значения p и q (x-отрезки) путем решения квадратного уравнения ax 2 + bx + c = 0.

        Пример: Преобразуйте квадратичную функцию f(x) = x 2 — 5x + 6 в форму пересечения.

        • Шаг — 1: Сравнивая данную функцию с f(x) = ax 2 + bx + c, получаем a = 1,
        • Шаг — 2: Решить квадратное уравнение: x 2 — 5x + 6 = 0
          Разложив на множители левую часть, получим
          . (х — 3) (х — 2) = 0
          х = 3, х = 2
        • Шаг – 3: Подставьте значения в форму вычленения: f(x) = 1 (x – 3)(x – 2).

        Область и диапазон квадратичной функции

        Область определения квадратичной функции — это множество всех значений x, определяющих функцию, а диапазон квадратичной функции — это множество всех значений y, которые функция получает путем замены различных значений x.

        Область квадратичной функции

        Квадратичная функция — это полиномиальная функция, определенная для всех действительных значений x. Итак, область определения квадратичной функции — это множество действительных чисел, то есть R. В интервальной записи областью определения любой квадратичной функции является (-∞, ∞).

        Диапазон квадратичной функции

        Диапазон квадратичной функции зависит от открытой стороны и вершины графика. Итак, найдите самое нижнее и самое верхнее значения f(x) на графике функции, чтобы определить диапазон квадратичной функции. Область значений любой квадратичной функции с вершиной (h, k) и уравнением f(x) = a(x — h) 2 + k равно:

        • y ≥ k (или) [k, ∞), когда a > 0 (поскольку парабола раскрывается, когда a > 0).
        • y ≤ k (или) (-∞, k], когда a < 0 (поскольку парабола раскрывается вниз, когда a < 0).

        График квадратичной функции

        График квадратичной функции представляет собой параболу. т. е. открывается вверх или вниз в форме буквы U. Вот шаги для построения графика квадратичной функции.

        • Шаг — 1: Найдите вершину.
        • Шаг — 2: Вычислите таблицу квадратичных функций с двумя столбцами x и y с 5 строками (мы также можем взять больше строк) с вершиной в одной из точек и возьмите два случайных значения по обе стороны от нее.
        • Шаг — 3: Найдите соответствующие значения y, подставив каждое значение x в заданную квадратичную функцию.
        • Шаг — 4: Теперь у нас есть две точки по обе стороны от вершины, поэтому, нанеся их на координатную плоскость и соединив их кривой, мы можем получить идеальную форму. Кроме того, расширьте график с обеих сторон. Вот график квадратичной функции.

        Пример: Нарисуйте график квадратичной функции f(x) = 2x 2 — 8x + 3.

        Решение:

        Сравнивая это с f(x) = ax 2 90 174 + бх + в, получаем a = 2, b = -8 и c = 3.

        • Шаг — 1: Найдем вершину.
          x-координата вершины = -b/2a = 8/4 = 2
          y-координата вершины = f(-b/2a) = 2(2) 2 — 8(2) + 3 = 8 — 16 + 3 = -5.
          Следовательно, вершина = (2, -5).
        • Шаг — 2: Создайте таблицу с вершиной, записанной в средней строке.
          х и
             
             
          2 -5
             
             
        • Шаг — 3: Заполните первый столбец двумя случайными числами по обе стороны от 2.
          х и
          0  
          1  
          2 -5
          3  
          4  
        • Шаг — 4: Найдите y, подставив каждое значение x в заданную квадратичную функцию. Например, когда x = 0, y = 2(0) 2 — 8(0) + 3 = 3.
          х и
          0 3
          1 -3
          2 -5
          3 -3
          4 3
        • Шаг — 5: Просто нанесите указанные выше точки и соедините их плавной кривой.

        Примечание: Мы можем построить точки пересечения по осям x и y квадратичной функции, чтобы получить более аккуратную форму графика.

        График квадратичных функций также можно получить с помощью калькулятора квадратичных функций.

        Максимумы и минимумы квадратичной функции

        Максимумы или минимумы квадратичных функций находятся в его вершине. Его также можно найти с помощью дифференцирования. Чтобы лучше понять концепцию, давайте рассмотрим пример и решим его. Возьмем пример квадратичной функции f(x) = 3x 2 + 4x + 7.

        Дифференцируя функцию,

        ⇒f'(x) = 6x + 4

        Приравнивание к нулю,

        ⇒6x + 4 = 0

        ⇒ x = -2/3

        Двойное дифференцирование функции,

        9000 2 ⇒f»( x) = 6 > 0

        Поскольку двойная производная функции больше нуля, у нас будут минимумы при x = -2/3 (по тесту второй производной), а парабола направлена ​​вверх.

        Аналогично, если двойная производная в стационарной точке меньше нуля, то функция будет иметь максимумы. Следовательно, используя дифференцирование, мы можем найти минимум или максимум квадратичной функции.

        Статьи по теме

        • Калькулятор квадратных уравнений
        • Калькулятор корней квадратного уравнения

        Важные замечания по квадратичной функции:

        • Стандартная форма квадратичной функции: f(x) = ax 2 +bx+c, где a ≠ 0,
        • График квадратичной функции имеет форму параболы.
        • Квадратная формула используется для решения квадратного уравнения ax 2 + bx + c = 0 и определяется как x = [-b ± √(b 2 — 4ac)] / 2a.
        • Дискриминант квадратного уравнения ax 2 + bx + c = 0 равен b 2 -4ac. Это используется для определения характера нулей квадратичной функции.

         

        Примеры квадратичной функции

        1. Пример 1: Определить вершину квадратичной функции f(x) = 2(x+3) 2 — 2,

          Решение: Имеем f(x) = 2(x+3) 2 — 2, что можно записать как f(x) = 2(x-(-3)) 2 + (-2 )

          Сравнивая данную квадратичную функцию с вершинной формой квадратичной функции f(x) = a(x-h) 2 + k, где (h,k) вершина параболы, имеем

          h = — 3, k = -2

          Следовательно, вершина f(x) равна (-3,-2)

          Ответ: Вершина = (-3,-2)

        2. Пример 2: Найдите нули квадратичной функции f(x) = x 2 + 3x — 4, используя формулу квадратичной функции.

          Решение: Квадратичная функция f(x) = x 2 + 3x — 4. Сравнивая f(x) с общей формой ax 2 + bx + c, получаем a = 1, b = 3, c = -4

          Нули квадратичной функции получаются путем решения f(x) = 0.

          Для этого используем квадратичную формулу: x = [ -b ± √(b 2 — 4ac) ] / 2а

          х = [ -3 ± √{3 2 — 4(1)(-4)}] / 2(1) = [ -3 ± √(9 + 16) ] / 2 = [ -3 ± √25 ] / 2,

          х = [ — 3 + 5 ] / 2, [ -3 — 5 ] / 2

          = 1, -4

          Ответ: Корни f(x) = x 2 + 3x — 4 равны 1 и -4

        3. Пример 3: Запишите квадратичную функцию f(x) = (x-12)(x+3) в общем виде ax 2 + bx + c.

          Решение: У нас есть квадратичная функция f(x) = (x-12)(x+3). Мы просто расширим (умножим биномы) его, чтобы записать в общем виде.

          f(x) = (x-12)(x+3)

          = x(x+3) — 12(x+3)

          = x 2 + 3x — 12x — 36

          = x 2 — 9x — 36

          Ответ: x 2 — 9x — 36

        перейти к слайдуперейти к слайдуперейти к слайду

        Разбивайте сложные концепции с помощью простых визуальных средств.

        Математика больше не будет сложным предметом, особенно когда вы понимаете концепции с помощью визуализаций.

        Забронировать бесплатный пробный урок

        Практические вопросы по квадратичной функции

         

        перейти к слайдуперейти к слайду

        Часто задаваемые вопросы о квадратичной функции

        Что такое квадратичная функция в математике?

        Квадратичная функция — это полиномиальная функция с одной или несколькими переменными, в которой старший показатель переменной равен двум. Другими словами, квадратичная функция — это «полиномиальная функция степени 2».

        Почему название квадратичной функции?

        Слово «quad» означает «квадрат». Следовательно, полиномиальная функция степени 2 называется квадратичной функцией.

        Что такое квадратичное функциональное уравнение?

        Квадратичная функция является многочленом степени 2, поэтому уравнение квадратичной функции имеет вид f(x) = ax 2 + bx + c, где «a» — ненулевое число; a, b и c — действительные числа.

        Что такое вершина квадратичной функции?

        Вершина квадратичной функции — это точка, в которой парабола меняет направление и пересекает ось симметрии. Это точка, в которой парабола меняется с возрастающей на убывающую или с убывающей на возрастающую. В этой точке производная квадратичной функции равна 0,

        Что такое нули квадратичной функции?

        Нули квадратичной функции — это точки, в которых график функции пересекает ось x. В нулях функции координата y равна 0, а координата x представляет собой нули квадратичной полиномиальной функции. Нули квадратичной функции также называют корнями функции.

        Что такое таблица квадратичных функций?

        Таблица квадратичных функций — это таблица, в которой мы определяем значения координат y, соответствующие каждой координате x, и наоборот. Таблица состоит из координат графика квадратичных функций. Мы обычно записываем вершину квадратичной функции в квадратичной функции в одной из строк таблицы.

        Как рисовать квадратичный график?

        График квадратичной функции представляет собой параболу. Его можно нарисовать, нанеся координаты на график. Мы подставляем значения x и получаем соответствующие значения y, таким образом получая координаты графика. После нанесения координат на график, свободной рукой соединяем точки, чтобы получить график квадратичных функций. Нахождение вершины помогает в построении квадратичного графа.

        Как найти точку пересечения по оси x квадратичной функции?

        X-пересечение квадратичной функции можно найти, рассматривая квадратичную функцию f(x) = 0 и затем определяя значение x. Другими словами, точка пересечения с осью x есть не что иное, как нуль квадратного уравнения.

        Является ли парабола квадратичной функцией?

        Парабола — это график квадратичной функции. Квадратичная функция имеет вид f(x) = ax 2 + bx + c, где a не равно 0. Парабола — это U-образный или перевернутый U-образный график квадратичной функции.

        Как найти обратную квадратичную функцию?

        Обратную квадратичную функцию f(x) можно найти, заменив f(x) на y.

        Предел функции тригонометрической: Как решать пределы с тригонометрическими функциями, примеры

        Примеры решения пределов тригонометрических функций с ответами

        Простое объяснение принципов решения пределов тригонометрических функций и 10 наглядных примеров. В каждом примере поэтапный ход решения и ответ.

        Алгоритм решения пределов тригонометрических функций

        Теорема

        Для тригонометрических функций существует много разных пределов, но как правило, все они вычисляются, опираясь на первый замечательный предел и его следствия.

        Первый замечательный предел выглядит следующим образом:

           

        Следствия первого замечательного предела

        Главным следствием первого замечательного предела считают:

           

        Также следствиями являются:

           

           

           

        Нужна помощь в написании работы?

        Мы — биржа профессиональных авторов (преподавателей и доцентов вузов). Наша система гарантирует сдачу работы к сроку без плагиата. Правки вносим бесплатно.

        Заказать работу

        Примеры решения пределов тригонометрических функций

        Пример 1

        Задание

        Найти предел функции:

           

        Решение

        Заменим значение х на число, к которому стремится функция:

           

        Так как мы пришли на неопределённость вида 0/0, преобразуем синус так, чтобы он стал вида первого замечательного предела:

           

        Мы знаем, что первый замечательный предел равен единице, следовательно

           

        Таким образом найдём предел функции:

           

        Пример 2

        Задание

        Найти предел функции:

           

        Решение

        При замене х на число, к которому он стремится, снова получаем неопределённость

           

        Данную задачу можно решить, применив правило Лопиталя.

        Найдём производные числителя и знаменателя функции и решим задачу:

           

        Пример 3

        Задание

        Найти предел функции:

           

        Решение

        При подстановке нуля получим неопределённость типа 0/0:

           

        Воспользуемся свойством

           

        Преобразуем функцию и упростим её:

           

        Вынесем константу ½ за лимит и, пользуюсь свойством первого замечательного предела, найдём передел данной функции:

           

        Пример 4

        Задание

        Найти предел функции:

           

        Решение

        Если заменить x на число, придём к неопределённости 0/0:

           

        Для решения данного примера применим правило Лопиталя и заменим х на число в производных:

           

        Пример 5

        Задание

        Вычислить предел функции:

           

        Решение

        Для решения данного примера воспользуемся свойством разности косинусов:

           

        и получим

           

        Вынесем минус за лимит, дабы не потерять и продолжим решение. Для решения задачи приведём функцию к виду первого замечательного предела. Для этого нужно разделить дробь на множители и добавить в знаменатель коэффициент, равный коэффициенту в числителе. А потом упростим выражение:

           

        Снова вынесем константы за лимит и получим вид первого замечательного предела, с помощью которого приходим к искомому решению:

           

        Пример 6

        Задание

        Вычислить предел функции:

           

        Решение

        При подстановке х снова получаем неопределённость

           

        Значит будем искать передел путём приведения к виду первого замечательного предела.

        Представим тангенс в виде частного синуса х и косинуса х

           

        Приведём к общему знаменателю и разделим выражение на множители следующим образом:

           

        Мы видим первый замечательный предел, а значит, можем упростить до:

           

        Далее снова приведём числитель к общему знаменателю:

           

        Вновь разделим на множители и подставим значение х во второй косинус:

           

        Таким образом нам остаётся разобраться с первым числителем. Поменяем местами 1 и косинус и вынесем минус за лимит.

        Далее воспользуемся формулой понижения степени и найдём решение:

           

        Пример 7

        Задание

        Вычислить предел функции:

           

        Решение

        При простом вычислении получаем неопределённость

           

        Следовательно, будем вычислять предел, опираясь на правило первого замечательного предела. Приведём тангенс к виду частного синуса и косинуса:

           

        Разделим пример на множители.

        Приведём синусы к виду первого замечательного предела и получим ответ:

           

        Пример 8

        Задание

        Найти предел функции:

           

        Решение

        При подставлении числа на место х приходим к неопределённости типа 0/0:

           

        Преобразуем tg, приведем выражение к общему знаменателю cos x, вынесем общий множитель – sin x за скобку:

           

           

        Используя следствие из первого замечательного предела, преобразим выражение и избавимся от тангенса.

        Затем вновь приведем функцию к следствию первого замечательного предела и найдем ответ:

           

           

        Пример 9

        Задание

        Найти предел функции:

           

        Решение

        При подстановке числа видим неопределённость.

           

        Следовательно, искать предел будем, опираясь на правило первого замечательного предела. Для этого заменим переменную, которая будет стремиться к нулю:

           

           

        Подставим в функцию:

           

        Опираясь на свойства тригонометрии, заменим тангенс.

           

        Зная, что предел косинуса нуля = 1, преобразуем пример и приведём к виду первого замечательного предела.

        Найдём ответ.

           

        Пример 10

        Задание

        Вычислить предел функции:

           

        Решение

        Здесь так же получим неопределённость:

           

        Значит, введём новую переменную t:

           

           

           

        Подставим получившиеся значения в пример и найдём предел:

           

        Средняя оценка 2. 5 / 5. Количество оценок: 6

        Поставьте вашу оценку

        Сожалеем, что вы поставили низкую оценку!

        Позвольте нам стать лучше!

        Расскажите, как нам стать лучше?

        29876

        Закажите помощь с работой

        Не отобразилась форма расчета стоимости? Переходи по ссылке

        Не отобразилась форма расчета стоимости? Переходи по ссылке

        Полезно

        Непосредственное вычисление пределов, таблица пределов функций

        Определение непрерывности функции  в точке  и передела функции на бесконечности  и на использовании свойств предела непрерывной функции способствует непосредственному вычислению пределов.

        Определение 1

        Значение предела в точке непрерывности определено значением функции в этой точке.

        При опоре на свойства основные элементарные функции имеют предел в любой точке из области определения, вычисляется как значение соответствующей функции в этих точках.

        Пример 1

        Произвести вычисление предела функции limx→5arctg35·x

        Решение

        Функция арктангенса отличается непрерывностью на всей своей области определения. Отсюда получим, что в точке x0=5 функция является непрерывной. Из определения имеем, что для нахождения предела является значением этой же функции. Тогда необходимо произвести подстановку. Получим, что

        limx→5arctg35·x=arctg35·5=arctg3=π3

        Ответ: π3.

        Для вычисления односторонних пределов необходимо использовать значения точек границ предела. У акрксинуса и акрккосинуса  имеются такие значения x0=-1 или x0=1.

        При x→+∞ или x→-∞ вычисляются пределы функции, заданные на бесконечностях.

        Для упрощения выражений применяют свойства пределов:

        Определение 2
        1. limx→x0(k·f(x))=k·limx→x0f(x), k является коэффициентом.
        2. limx→x0(f(x)·g(x))=limx→x0f(x)·limx→x0g(x), применяемое при получении неопределенности предела.
        3. limx→x0(f(g(x)))=flimx→x0gx,используемое для непрерывных функций, где знак функции и предельного перехода можно менять местами.

        Для того, чтобы научиться вычислять переделы, необходимо знать и разбираться в основных элементарных функциях. Ниже приведена таблица, в которой имеются переделы этих функций с приведенными разъяснениями и подробным решением. Для вычисления необходимо основываться на определении предела функции в точке и на бесконечности.

        Таблица пределов функции

        Для упрощения  и решения пределов используется данная таблица основных пределов.

        Функция корень n-ой степени

        y=xn, где n=2, 4, 6 …

        limx→∞xn=+∞n=+∞

        Для любых x0 из опрелеления 

        limx→x0xn=x0n

        Функция корень n-ой степени

        y=xn, где n=3, 5, 7 … 

        limx→∞xn=+∞n=+∞limx→∞xn=-∞n=-∞

        limx→x0xn=x0n

        Степенная функция y=xa , a>0

        1. Для любого положительного числа a
          limx→∞xa=+∞a=+∞
        2. Если a=2, 4, 6 …, то
          limx→∞xa=-∞a=+∞
        3. Если a=1, 3, 5, …, то
          limx→∞xa=-∞a=-∞
        4. Для любых x0, из области определния
          limx→x0xa=(x0)a

        Степенная функция y=xa, a<0

        1. Для любого отрицательного числа a
          limx→∞xa=(+∞)a=+0limx→0+0=(0+0)a=+∞
        2. Если a=-2, -4, -4, . .., то
          limx→∞xa=-∞a=+0limx→0-0xa=(0-0)a=+∞
        3. Если a=-1, -3, -5, …, то
          limx→∞xa=-∞a=-0limx→0-0xa=(0-0)a=-∞
        4. Для любых x0 из области определения
          limx→x0xa=(x0)a

        Показательная функия

        y=ax, 0<a<1

        limx→∞ax=a-∞=+∞limx→∞ax=a+∞=+0

        Для любых x0 из области опреления limx→x0ax=ax0

        Показательная функия

        y=ax, a>1limx→∞ax=a-∞=+0limx→x0ax=a+∞=+∞

        Для любых знвчений x0 из област опредения limx→x0ax=ax0

        Логарифмическая функция

        y=loga(x), 0<a<1

        limx→0+0logax=loga(0+0)=+∞limx→∞logax=loga(+∞)=-∞

        Для любых x0 из области опрелеленияlimx→x0logax=logax0

        Логарифмическая функция

        y=loga(x), a>1

        limx→0+0logax=loga(0+0)=-∞limx→∞logax=loga(+∞)=+∞

        Для любых x0 из области опрелеления

        limx→x0logax=logax0

        Тригонометрические функции

        • Синус
          limx→∞ sin x не существует
          Для любых x0 из области опрелеления
          limx→x0sin x=sin x0
        • Тангненсlimx→π2-0+π·ktg x=tgπ2-0+π·k=+∞limx→π2+0+π·ktg x=tgπ2+0+π·k=-∞

        limx→∞tg x не существует

        Для любых x0 из области опрелеления

        limx→x0tg x=tg x0

        Тригонометрические функции

        • Косинус
          limx→∞cos x не существует 
          Для любых x0 из области опрелеления
          limx→x0cos x=cos x0
        • Котангенсlimx→-0+π·kctg x=ctg(-0+π·k)=-∞limx→+0+π·kctg x=ctg(+0+π·k)=+∞

        limx→∞ctg x не существует

        Для любых x0 из области опрелеления
        limx→x0сtg x=сtg x0

        Обратные тригонометрические функции

        • Арксинус
          limx→-1+0arcsin x=-π2limx→1-0arcsin x=π2

        Для любых x0 из области опрелеления

        limx→x0arcsin x=arcsin x0

        • Арккосинус
          limx→-1+0arccos (x)=πlimx→1-0arccos (x)=0

        Для любых x0 из области опрелеления

        limx→x0arccis x=arccos x0

        Обратные тригонометрические функции

        • Арктангес
          limx→-∞ arctg (x)=-π2limx→+∞ arctg (x)=π2

        Для любых x0 из области опрелеления

        limx→x0arctg x=arctg x0

        • Арккотангенс
          limx→-∞arcctg (x)=πlimx→+∞arcctg (x)=0

        Для любых x0 из области опрелеления

        limx→x0arcctg x=arcctg x0

        Пример 2

        Произвести вычисление предела limx→1×3+3x-1×5+3.

        Решение

        Для решения необходимо подставить значение х=1. Получаем, что

        limx→1×3+3x-1×5+3=13+3·1-115+3=34=32

        Ответ: limx→1×3+3x-1×5+3=32

        Пример 3

        Произвести вычисление предела функции limx→0(x2+2,5)1×2

        Решение

        Для того, чтобы раскрыть предел, необходимо подставить значение х, к которому стремится предел функции. В данном случае нужно произвести подстановку х=0. Подставляем числовое значение и получаем:

        x2+2.5x=0=02+2.5=2.5

        Предел записывается в виде limx→0(x2+2.5)1×2=limx→02.51×2. Далее необходимо заняться значением показателя. Он является степенной функцией 1×2=x-2. В таблице пределов, предоставленной выше, имеем, что limx→0+01×2=limx→0+0x-2=+∞ и limx→0+01×2=limx→0+0x-2=+∞, значит, имеем право записать как limx→01×2=limx→0x-2=+∞

        Теперь вычислим предел. Получит вид limx→0(x2+2.5)1×2=limx→02.51×2=2.5+∞

        По таблице пределов с показательными функциями, имеющими основание больше 1 получаем, что

        limx→0(x2+2. 5)1×2=limx→02.51×22.5+∞=+∞

        Ответ: limx→0(x2+2.5)1×2=+∞

        Когда задан более сложный предел, то при помощи таблицы не всегда получится получать целое или конкретное значение. Чаще получаются разные виды неопределенностей, для разрешения которых необходимо применять правила.

        Рассмотрим графическое разъяснение приведенной выше таблицы пределов основных элементарных функций.

        Предел константы

         

        Из рисунка видно, что функция у=С имеет предел на бесконечности. Такой же предел при аргументе, который стремится к х0. Он равняется числу C.

        Предел функции корень n-ой степени

        Четные показатели корня применимы для limx→+∞xn=+∞n=+∞, а нечетные, равные больше, чем значение 1, – для limx→+∞xn=+∞n=+∞, limx→-∞xn=-∞n=-∞.  Область определения может принимать абсолютно любое значение х предела заданной функции корня n-ой степени, равного значению функции  в заданной точке.

        Предел степенной функции

        Необходимо разделить все степенные функции по группам, где имеются одинаковые значения пределов, исходя из показателя степени.

        1. Когда a является положительным числом, тогда limx→+∞xa=+∞a=+∞ и limx→-∞xa=-∞a=-∞. Когда x принимает любое значение, тогда предел степенной функции равняется значению функции в точке. Иначе это записывается как limx→∞xa=(∞)a=∞.

        1. Когда a является положительным четным числом, тогда получаем limx→+∞xa=(+∞)a=+∞ и limx→-∞xa=(-∞)a=+∞, причем x из данной области определения  является пределом степенной функции и равняется значением функции в этой точке. Предел имеет вид limx→∞xa=∞a=+∞.

        1. Когда a имеет другие значения, тогда limx→+∞xa=(+∞)a=+∞, а область определения x способствует определению предела функции в заданной точке.

        1. Когда a имеет значение отрицательных чисел, тогда получаем limx→+∞xa=+∞a=+0, limx→-∞xa=(-∞)a=-0, limx→0-0xa=(0-0)a=-∞,limx→0+0xa=0+0a=+∞, а значения x может быть любым из заданной области определения и равняется функции в заданной точке. Получаем, что limx→∞xa=∞a=0 иlimx→0xa=0a=∞.

        1. Когда a является отрицательным четным числом, тогда получаем limx→+∞xa=(+∞)a=+0, limx→-∞xa=-∞a=+0, limx→0-0(0-0)a=+∞, limx→0+0xa=(0+0)a=+∞, а любое значение x на области определения дает результат предела степенной функции равным значению функции в точке. Запишем как limx→∞xa=(∞)a=+0 и limx→0xa=(0)a=+∞.

        1. Когда значение a имеет другие действительные отрицательные числа, тогда получим limx→+∞xa=+∞a=+0 и limx→0+0xa=0+0a=+∞, когда x принимает любое значение из своей области определения, тогда предел степенной функции равняется значению функции в этой точке.

        Предел показательной функции

        Когда 0<a<1, имеем, что limx→-∞ax=a-∞=+∞, limx→+∞ax=(a)+∞=+∞, любое значение x из области определения дает пределу показательной функции значению функции в точке.

        Когда a>1, тогда limx→-∞ax=(a)-∞=+0, limx→+∞ax=(a)+∞=+∞, а любое значение x из области определения дает предел функции равный значению этой функции в точке.

        Предел логарифмической функции

        Когда имеем 0<a<1, тогда limx→0+0logax=loga(0+0)=+∞, limx→+∞logax=loga(+∞)=-∞ ,  для всех остальных значений x из заданной области определения предел показательной функции равняется значению заданной функции в точках.

        Когда a>1, получаем limx→0+0logax=loga(0+0)=-∞, limx→+∞logax=loga(+∞)=+∞,остальные значения x в заданной области определения дают решение предела показательной функции равному ее значению в точках.

        Предел тригонометрических функций

        Предел бесконечности не существует для таких функций как y=sin x, y=cos x. Любое значение x, входящее в область определения, равняется значению функции в точке.

         

        Функция тангенса имеет предел вида limx→π2-0+π·ktg(x)=+∞, limx→π2+π·ktg(x)=∞ или limx→π2+π·ktg(x)=∞, тогда остальные значения x, принадлежащие области определения тангенса, равняется значению функции в этих точках.

        Для функции y=ctg x получаем limx→-0+π·kctg(x)=-∞, limx→+0+π·kctg(x)=+∞ или limx→π·kctg (x)=∞, тогда остальные значения x, принадлежащие области определения, дают предел котангенса, равный значению функции в этих точках.

        Предел обратных тригонометрических функций

        Функция арксинус имеет предел вида limx→-1+0arcsin(x)=-π2 и limx→1-0arcsin (x)=π2, остальные значения x из области определения равняются значению функции в заданной точке.

         

        Функция арккосинус имеет предел вида limx→-1+0arccos(x)=π и limx→1-0arccos(x)=0, когда остальные значения x, принадлежащие области определения, имеют предел арккосинуса, равного значению функции в этой точке.

        Функция арктангенс имеет предел вида limx→-∞arctg(x)=-π2 и limx→+∞arctg(x)=π2, причем другие значения x, входящие в область определения, равняется значению функции  в имеющихся точках.

         

        Функция котангенса имеет предел вида limx→-∞arcctg(x)=π и limx→+∞arctg(x)=0, где x принимает любое значение из своей заданной области определения, где получаем предел арккотангенса, равного значению функции в имеющихся точках.

        Все имеющееся значения пределов применяются в решении для нахождения предела любой из элементарных функций.

        Объяснение урока: Пределы тригонометрических функций

        В этом объяснителе мы узнаем, как вычислять пределы тригонометрических функций.

        Пределы — полезный инструмент, помогающий нам понять форму функции вокруг значения; это один из фундаментальных строительных блоков исчисления. Мы можем найти предел любой тригонометрической функции, используя прямую замену.

        Определение: оценка предела тригонометрических функций

        Если 𝑎 находится в области определения тригонометрической функции, то мы можем вычислить ее предел в 𝑎 прямой подстановкой. В частности, для любого 𝑎∈ℝ

        • limsinsin→𝑥=𝑎,
        • limcoscos→𝑥=𝑎.

        Для любого 𝑎 в области tan𝑥,

        • limtantan→𝑥=𝑎.

        Эти результаты позволяют оценить предел многих тригонометрических выражений. Однако есть примеры, которые мы не можем оценить. Например, рассмотрим limsin→𝑥𝑥, где 𝑥 измеряется в радианах. Если мы попытаемся вычислить этот предел с помощью прямой подстановки, sin00=00, мы обнаружим неопределенную форму, а это означает, что нам нужно вычислить этот предел другим способом. Один из способов сделать это — нарисовать график 𝑦=𝑥𝑥sin.

        На схеме видно, что по мере того, как значения 𝑥 приближаются к 0 с любой стороны, выходы функции приближаются к 1. Следовательно, схема показывает, что limsin→𝑥𝑥=1. Мы также можем увидеть это, построив таблицу.

        𝑥 −0,1 −0,01 −0,001 0 9 0031 0,001 0,01 0,1
        sin𝑥𝑥 0,99833 0,99998 0,99999 0,99999 0,99998 0,99833

        Один раз опять же, таблица предполагает, что по мере того, как значения 𝑥 приближаются к 0 с любой стороны, выходы функции приближаются к 1. Стоит отметив, что мы можем показать аналогичный результат, когда 𝑥 измеряется в градусах; однако при определении пределов мы почти всегда используем радианы. Итак, если не оговорено иное, будем считать, что предел любой тригонометрической функции включает в себя углы, измеряемые в радианах. Это дает нам следующий результат.

        Теорема: предел тригонометрического выражения

        Если 𝑥 измеряется в радианах, то limsin→𝑥𝑥=1.

        Мы можем использовать этот результат, чтобы показать еще более общий результат. Пусть 𝑎∈ℝ−{0}. Подставляем 𝜃=𝑎𝑥 в предельный результат limsin→𝜃𝜃=1. Обратите внимание, что при 𝜃→0 и 𝑎𝑥→0, и 𝑥→0. Это дает нам 1 = 𝜃𝜃 = 𝑎𝑥𝑎𝑥.limsinlimsin→→

        . Вынесение множителя 1𝑎 за пределы этого предела и перестановка дает нам limsin→𝑎𝑥𝑥=𝑎.

        Стоит отметить, что этот результат справедлив и при 𝑎=0. Мы можем резюмировать это следующим образом.

        Теорема: предел тригонометрического выражения

        Если 𝑥 измеряется в радианах и 𝑎∈ℝ, то limsin→𝑎𝑥𝑥=𝑎.

        Давайте рассмотрим пример использования этого результата для вычисления предела тригонометрического выражения.

        Пример 1. Нахождение пределов с использованием тригонометрических функций

        Оценить лимсинсин→𝑥.

        Ответ

        Поскольку этот предел включает частное тригонометрических функций, мы можем попытаться вычислить этот предел прямой подстановкой sin0=00. 

        Это дает нам неопределенную форму, что означает, что мы не можем оценить этот предел прямой подстановкой. Вместо этого мы будем использовать тот факт, что если 𝑥 измеряется в радианах, а 𝑎 является вещественной константой, то limsin→𝑎𝑥𝑥=𝑎. Хотя в вопросе не говорится, что 𝑥 измеряется в радианах, при определении пределов мы почти всегда работаем в радианах, поэтому мы будем исходить из этого для вопроса. Мы можем переписать предел следующим образом:

        Предполагая, что оба предела существуют, мы можем записать это как произведение двух пределов:  .

        Возьмем обратное значение второго предела, используя правило степени для пределов, чтобы получить 𝑥𝑥 ×𝑥, если предел существует и не равен нулю. Затем мы можем оценить оба этих предела, используя наш предельный результат, limsin→𝑎𝑥𝑥=𝑎.

        В первом пределе 𝑎=1, а во втором 𝑎=12. Следовательно, limsinlimsin→→𝑥𝑥×𝑥=1×12=2.

        Есть еще два полезных предельных результата, связанных с тригонометрическими функциями, которые мы можем найти, исследуя их график или используя таблицу. Рассмотрим следующие наброски tan𝑥𝑥 и 1−𝑥𝑥cos, где 𝑥 измеряется в радианах.

        На первой диаграмме мы видим, что по мере того, как значения 𝑥 приближаются к 0, выходы приближаются к 1. Итак, набросок предполагает limtan→𝑥𝑥=1. Точно так же на второй диаграмме, когда значения 𝑥 приближаются к 0, мы видим, что выходы приближаются к 0. Таким образом, набросок предполагает, что limcos→1−𝑥𝑥=0. Это дает нам следующие результаты.

        Теорема: предел тригонометрического выражения

        Если 𝑥 измеряется в радианах, тогда .

        Как и в случае с предельным результатом, включающим синус, мы можем использовать подстановку, чтобы найти предельный результат, где аргумент является постоянным кратным. Если 𝑎∈ℝ, используя 𝜃=𝑎𝑥, мы имеем 1=𝜃𝜃=𝑎𝑥𝑎𝑥.limtanlimtan→→

        Удаление постоянного множителя 1𝑎 и перестановка дает limtan→𝑎𝑥 𝑥=𝑎.

        Аналогично, если 𝑎∈ℝ, используя 𝜃=𝑎𝑥, мы имеем 0=1−𝜃𝜃=1−𝑎𝑥𝑎𝑥.limcoslimcos→→

        Убираем постоянный множитель 1𝑎 и переставляем ing дает limcos→1− 𝑎𝑥𝑥=0.

        Мы можем подытожить это следующим образом.

        Теорема: предел тригонометрического выражения. →1−𝑎𝑥𝑥=0.

        Давайте рассмотрим пример того, как мы можем применить эти предельные результаты для вычисления предела тригонометрического выражения.

        Пример 2. Нахождение пределов с использованием тригонометрических функций

        Определить limcos→9−97𝑥3𝑥.

        Ответ

        Поскольку этот предел включает тригонометрическую функцию, мы можем попытаться вычислить этот предел путем прямой подстановки: 9−9(7×0)3(0)=00.cos

        Это дает нам неопределенную форму, что означает, что мы не можем вычислить этот предел прямой подстановкой. Вместо этого мы будем использовать тот факт, что если 𝑥 измеряется в радианах и 𝑎∈ℝ, то limcos→1−𝑎𝑥𝑥=0.

        Чтобы применить этот результат, мы упростим наш предел следующим образом: 1−7𝑥)𝑥=3×0=0.

        Следовательно, limcos→9−97𝑥3𝑥=0.

        В нашем следующем примере мы будем использовать предельный результат, включающий функции тангенса и синуса, для вычисления предела тригонометрической функции.

        Пример 3. Нахождение пределов с использованием тригонометрических функций

        Найти лимсинтан→7𝑥+33𝑥8𝑥.

        Ответ

        Поскольку это предел тригонометрического и алгебраического выражения, мы можем попытаться вычислить этот предел прямой подстановкой: sintan(7(0))+3(3(0))8(0) =00.

        Поскольку это неопределенная форма, мы не можем определить значение этого предела из прямой подстановки. Вместо этого мы перепишем этот предел в терминах пределов, которые мы можем оценить. А именно, для любой реальной постоянной 𝑎 и 𝑥, измеряемой в радианах, limtanandlimsin→→𝑎𝑥𝑥=𝑎𝑎𝑥𝑥=𝑎.

        Мы можем переписать предел в вопросе следующим образом: →→ →7𝑥+33𝑥8𝑥=7𝑥8𝑥+33𝑥8𝑥=7𝑥8𝑥+33𝑥8𝑥=187𝑥𝑥+383 𝑥𝑥=187𝑥𝑥+383𝑥𝑥.

        Мы можем оценить каждый из этих пределов отдельно. Во-первых, напомним, что если 𝑥 измеряется в радианах и константа 𝑎∈ℝ, то limsin→𝑎𝑥𝑥=𝑎. Используя этот результат, мы имеем limsin→7𝑥𝑥=7.

        Далее напомним, что если 𝑥 измеряется в радианах и 𝑎∈ℝ, то limtan→𝑎𝑥𝑥=𝑎.

        Следовательно, лимтан→3𝑥𝑥=3.

        Подстановка значений этих пределов в уравнение дает нам 3𝑥𝑥=18( 7)+38(3)=498+278=768=192.

        Следовательно, лимсинтан→7𝑥+33𝑥8𝑥=192.

        В нашем следующем примере мы объединим тригонометрическое тождество с предельными результатами тригонометрических функций для вычисления предела.

        Пример 4. Нахождение пределов с использованием тригонометрических функций

        Найти limsin→2−2𝑥4𝑥−2𝜋.

        Ответ

        Так как это предел тригонометрического и алгебраического выражения, мы можем попытаться вычислить этот предел прямой подстановкой: 2−24−2𝜋=00.sin является неопределенной формой, мы не можем определить значение этого предела из прямой подстановки. Вместо этого мы перепишем этот предел в терминах пределов, которые мы можем оценить. Перепишем предел следующим образом: 2 𝑥−.

        Чтобы оценить этот предел, мы будем использовать замену 𝜃=𝑥−𝜋2. Когда 𝑥 приближается к 𝜋2, 𝜃 приближается к 0. Это дает нам limsinlimsin→→1−𝑥2𝑥−=1−𝜃+2𝜃.

        Напомним, что sincos𝜃+𝜋2≡𝜃. Мы можем использовать это, чтобы переписать предел следующим образом:

        Наконец, напомним, что limcos→1−𝑎𝑥𝑥=0.

        Следовательно, 121−𝜃𝜃=12×0=0.limcos→

        Следовательно, limsin→2−2𝑥4𝑥−2𝜋=0.

        В нашем последнем примере мы будем использовать эти предельные результаты для оценки предела обратного тригонометрического выражения.

        Пример 5. Нахождение пределов с использованием тригонометрических функций

        Найти limcotcsc→6𝑥4𝑥8𝑥.

        Ответ

        Поскольку это предел тригонометрического и алгебраического выражения, мы можем попытаться вычислить этот предел прямой подстановкой. Однако 0 не находится в области определения этой функции. Вместо этого мы перепишем предел, сначала используя взаимные тригонометрические тождества: 𝑥8𝑥4𝑥.tansin

        Затем мы можем переписать это с точки зрения предельных результатов. Если 𝑥 измеряется в радианах и 𝑎∈ℝ, limsinlimtan→→𝑎𝑥𝑥=𝑎,𝑎𝑥𝑥=𝑎.

        Таким образом, у нас есть →→6𝑥8𝑥4𝑥 =6𝑥8𝑥𝑥4𝑥=6𝑥4𝑥8𝑥𝑥=6𝑥4𝑥8𝑥𝑥=6𝑥4𝑥8𝑥 𝑥=64𝑥𝑥8𝑥𝑥.

        Применяя предельные результаты, заключаем, что → 6𝑥4𝑥8𝑥=3.

        Давайте закончим повторением некоторых важных моментов из этого объяснения.

        Ключевые моменты

        • Мы можем вычислить предел любой тригонометрической функции в точке 𝑥=𝑎 прямой подстановкой, если a находится в ее области определения.
        • Если 𝑥 измеряется в радианах, мы имеем следующие тригонометрические предельные результаты: 1−𝑥𝑥=0.
      1. Если 𝑥 измеряется в радианах и 𝑎∈ℝ, мы имеем следующие результаты тригонометрического предела: 𝑎𝑥𝑥=𝑎,
      2. лимузин→1 −𝑎𝑥𝑥=0.
      3. Мы можем использовать эти результаты для оценки предела тригонометрических функций.
      4. Тригонометрические пределы Проблемы и решения

        • Математические сомнения
        • Проблемы
        • Ограничения

        Проблемы пределов, связанные с тригонометрическими функциями, появляются в исчислении.

        © 2015 - 2019 Муниципальное казённое общеобразовательное учреждение «Таловская средняя школа»

        Карта сайта